You are on page 1of 184

1

Describe the structure and functional significance of the excitatory, conductive and contractile elements of the heart
(Structure see next question) Functional significance of excitatory & conductive elements: Functional significance of contractile elements:

describe the anatomy of the heart and pericardium


Size.The heart, in the adult, measures about 12 cm. in length, 8 to 9 cm. in breadth at the broadest part, and 6 cm. in thickness. Its weight, in the male, varies from 280 to 340 grams; in the female, from 230 to 280 grams. The heart continues to increase in weight and size up to an advanced period of life; this increase is more marked in men than in women.
Cardiac Chambers
Right atrium The SVC & IVC open into the posterior wall. The coronary sinus drains coronary venous blood into the anteroinferior portion. The thebesian valve is located at the orifice of the coronary sinus. On the medial wall, the limbus of the fossa ovalis circumscribes the septum primum of the fossa ovalis anteriorly, posteriorly, and superiorly. The right auricle is separated from the right atrium internally, by a vertical crest ie, the crista terminalis. The crista terminalis separates the right atrium into trabeculated and nontrabeculated portions. The right atrium is larger than the left, but its walls are somewhat thinner, measuring about 2 mm.; its cavity is capable of containing about 57 c.c. Left atrium The 4 pulmonary veins drain into the left atrium. The flap valve of the fossa ovalis is located on the septal surface. The appendage of the left atrium is consistently narrow and long & is the only trabeculated structure in the left atrium. Right ventricle Tricuspid valve is located in the large anterolateral portion (sinus) of the right ventricle. Pulmonic (semilunar) valve is located in the outflow tract (infundibulum). Internally, both the sinus area and infundibulum contain coarse trabeculations. The septal portion of the right ventricle has 3 components: (1) the inflow tract, which supports the tricuspid valve; (2) the trabecular wall, which typifies the internal appearance of the right ventricle; and (3) the outflow tract. The tricuspid valve is supported by a large anterior papillary muscle, which arises from the anterior free wall and the moderator band, and by several small posterior papillary muscles. The wall of the right ventricle is thinner than that of the left, the proportion between them being as 1 to 3; it is thickest at the base, and gradually becomes thinner toward the apex. The cavity equals in size that of the left ventricle, and is capable of containing about 85 c.c. Left ventricle

2
The left ventricle can be divided into 2 primary portions, namely, the large sinus portion containing the mitral valve and the small outflow tract that supports the aortic (semilunar) valve. The free wall and apical half of the septum contain fine internal trabeculations. The septal surface is divided into a trabeculated portion (sinus) and a smooth portion (outflow). The outflow tract is located anterior to the anterior mitral leaflet and is part of the atrioventricular (AV) septum. The mitral valve is supported by 2 large papillary muscles (ie, anterolateral, posteromedial) attached to the free wall. The anterior papillary muscle is attached to the anterior portion of the left ventricular wall, and the posterior papillary muscle arises more posteriorly from the ventricle's inferior wall.1

Septi
Ventricular septum The ventricular septum is divided into a muscular section (inferior) and a membranous section (superior). The muscular portion comprises the left and right ventricular walls. The membranous septum, also termed the pars membranacea, is a fibrous structure partially separating the left ventricular outflow tract from the right atrium and ventricle. Atrioventricular septum The atrioventricular (AV) septum, located behind the right atrium and left ventricle, is divided into 2 portions: a superior portion (membranous) and an inferior portion (muscular). Inside the left ventricle, the muscular component comprises part of the outlet septum. The AV node lies in the atrial septum, juxtaposed to the membranous and muscular portions of the AV septum.

Conduction System
The conduction system is composed for the most part of modified cardiac muscle that has fewer striations and indistinct boundaries. The SA node and, to a lesser extent, the AV node, also contain small round cells with few organelles, which are connected by gap junctions. These are probably the actual pacemaker cells, and therefore they are called P cells. Sinus node The sinoatrial (SA) node occupies a 1-cm2 area on the lateral surface of the junction of the superior vena cava and right atrium near the crista terminalis. Internodal pathways The spread of electrical activation from the sinus node extends toward the atrioventricular (AV) node via Purkinje like pale cells in atrial muscle bundles. There are three bundles: the anterior internodal tract of Bachman, the middle internodal tract of Wenckebach, and the posterior internodal tract of Thorel. Conduction also occurs through atrial myocytes, but it is more rapid in these bundles. Atrioventricular node The AV node is situated directly on the right atrial side of the central fibrous body in the muscular portion of the AV septum, just superior and anterior to the ostium of the coronary sinus. Measuring approximately 0.1 cm X 0.3 cm X 0.6 cm.

His bundle and bundle branches The AV node continues onto the His bundle which follows a course along the inferior border of the membranous septum and, near the aortic valve, gives off fibers that form the left bundle branch. The left bundle branch divides into an anterior fascicle and a posterior fascicle. The branches and fascicles run subendocardially down either side of the septum and come into contact with the Purkinje system, whose fibers spread to all parts of the ventricular myocardium.

Cardiac Valves

3
Mitral valve bicuspid AV valve of the left ventricle. The AV valve has a large anterior leaflet (septal or aortic) and a smaller posterior leaflet (mural or ventricular). The anterior leaflet is triangular with a smooth texture. The posterior leaflet has a scalloped appearance. The chordae tendineae to the mitral valve originate from the 2 large papillary muscles of the left ventricle and insert primarily on the leaflet's free edge. Tricuspid valve The AV valve of the right ventricle has anterior, posterior, and septal leaflets. The orifice is larger than the mitral orifice and is triangular. The tricuspid valve leaflets and chordae are more fragile than those of the mitral valve. The anterior leaflet, largest of the 3 leaflets, often has notches. The posterior leaflet, smallest of the 3 leaflets, is usually scalloped. The septal leaflet usually attaches to the membranous and muscular portions of the ventricular septum. The right atrioventricular orifice is the large oval aperture of communication between the right atrium and ventricle. Situated at the base of the ventricle, it measures about 4 cm. in diameter and is surrounded by a fibrous ring. Aortic valve The aortic valve has 3 leaflets composed of fragile cusps and the sinuses of Valsalva. Thus, the valve apparatus is composed of 3 cuplike structures that are in continuity with the membranous septum and the mitral anterior leaflet. The aortic sinuses of Valsalva are 3 dilations of the aortic root that arise from the 3 closing cusps of the aortic valve. The right and left sinuses give rise to the right and left coronary arteries; the noncoronary sinus has no coronary artery. The sinus of Valsalva walls are much thinner than the aortic wall, which is a factor of surgical significance; therefore, aortotomies are typically performed away from this region. Pulmonary valve As with the aortic valve, the pulmonary valve has 3 cusps, with a midpoint nodule at the free end and lunulae on either side; a sinus is located behind each cusp.

Coronary Arteries
4 main arteries: the left main, the left anterior descending, and the left circumflex (LCX) arteries (which are all branches of the left coronary artery) and the right coronary artery (RCA). The RCA and LCXs form a circle around the atrioventricular (AV) sulci. The left anterior descending and posterior descending arteries form a loop at right angles to this circle; these arteries feed the ventricular septum. The LCX gives off several parallel, obtuse, marginal arteries that supply the posterior left ventricle. The diagonal branches of the left anterior descending artery supply the anterior portion of the left ventricle. The term dominance is used to refer to the origin of the posterior descending artery (PDA). When the PDA is formed from the terminal branch of the RCA (>85% of patients), it is termed a right-dominant heart. A leftdominant heart receives its PDA blood supply from a left coronary branch, usually the LCX. This is often referred to as a left posterolateral branch (LPL). Left main coronary artery Typically is 1-2 cm in length. When it reaches the left AV groove, the LCA bifurcates into the left anterior descending (LAD) and the LCX branches. The LCA supplies most of the left atrium, left ventricle, interventricular septum, and AV bundles. Left anterior descending artery runs along the anterior interventricular sulcus and supplies the apical portion of both ventricles. Gives rise to 4-6 perpendicular septal branches which supply the interventricular septum. toward the apex, it turns sharply to anastomose with the posterior interventricular branch of the RCA. As the LAD artery courses anteriorly along the ventricular septum, it sends off diagonal branches to the lateral wall of the left ventricle. Left circumflex artery

4
The LCX artery courses in the coronary groove around the left border of the heart to the posterior surface of the heart to anastomose to the end of the RCA. The atrial circumflex artery, the first branch off the LCX artery, supplies the left atrium. The LCX artery gives off an obtuse marginal (OM) branch at the left border of the heart near the base of the left atrial appendage to supply the posterolateral surface of the left ventricle. In fewer than 40% of patients, the sinus node artery may originate from the LCX artery. Right coronary artery The RCA is a single large artery that courses along the right AV groove. The RCA supplies the right atrium, right ventricle, interventricular septum, and the SA and AV nodes. In 60% of patients, the first branch of the RCA is the sinus node artery. As the RCA passes toward the inferior border of the heart, it gives off a right marginal branch that supplies the apex of the heart. After this branching, the RCA turns left to enter the posterior interventricular groove to give off the PDA, which supplies both ventricles. The AV node artery arises from the "U-turn" of the RCA at the crux (ie, the junction of the AV septum with the AV groove). Terminal branches of the RCA supply the posteromedial papillary muscle of the left ventricle. (The LAD artery supplies the anterolateral papillary muscle of the right ventricle.)

Cardiac innervations: The SA node develops from structures on the right side of the embryo and the AV node from structures on the left. This is why in the adult the right vagus is distributed mainly to the SA node and the left vagus mainly to the AV node. Similarly, the sympathetic innervation on the right side is distributed primarily to the SA node and the sympathetic innervation on the left side primarily to the AV node. On each side, most sympathetic fibers come from the stellate ganglion. Noradrenergic fibers are epicardial, whereas the vagal fibers are endocardial. However, connections exist for reciprocal inhibitory effects of the sympathetic and parasympathetic innervation of the heart on each other. Thus, acetylcholine acts presynaptically to reduce norepinephrine release from the sympathetic nerves, and conversely, neuropeptide Y released from noradrenergic endings may inhibit the release of acetylcholine. The parasympathetic system acts via M2 receptors & decreases cAMP while sympathetic system acts via 1 receptors & increases cAMP. The parasympathetic nerves are distributed mainly to nodes, to a lesser extent to atria and very little directly to the ventricles. The sympathetic nerves are distributed to all parts of the heart. Pericardium The heart is separated from the rest of the thoracic viscera by the pericardium. The myocardium itself is covered by the fibrous epicardium. The pericardial sac normally contains 5-30 mL of clear fluid, which lubricates the heart and permits it to contract with minimal friction. The pericardium is a conical fibro-serous sac, in which the heart and the roots of the great vessels are contained. It is placed behind the sternum and the cartilages of the third, fourth, fifth, sixth, and seventh ribs of the left side, in the mediastinal cavity. In front, it is separated from the anterior wall of the thorax, in the greater part of its extent, by the lungs and pleur; but a small area, somewhat variable in size, and usually corresponding with the left half of the lower portion of the body of the sternum and the medial ends of the cartilages of the fourth and fifth ribs of the left side, comes into direct relationship with the chest wall. The lower

5 extremity of the thymus, in the child, is in contact with the front of the upper part of the pericardium. Behind, it rests upon the bronchi, the esophagus, the descending thoracic aorta, and the posterior part of the mediastinal surface of each lung. Laterally, it is covered by the pleur, and is in relation with the mediastinal surfaces of the lungs; the phrenic nerve, with its accompanying vessels, descends between the pericardium and pleura on either side. Although the pericardium is usually described as a single sac, an examination of its structure shows that it consists essentially of two sacs intimately connected with one another, but totally different in structure. The outer sac, known as the fibrous pericardium, consists of fibrous tissue. The inner sac, or serous pericardium, is a delicate membrane which lies within the fibrous sac and lines its walls; it is composed of a single layer of flattened cells resting on loose connective tissue. The heart invaginates the wall of the serous sac from above and behind, and practically obliterates its cavity, the space being merely a potential one. The fibrous pericardium forms a flask-shaped bag, the neck of which is closed by its fusion with the external coats of the great vessels, while its base is attached to the central tendon and to the muscular fibers of the left side of the diaphragm. Above, the fibrous pericardium not only blends with the external coats of the great vessels, but is continuous with the pretracheal layer of the deep cervical fascia. By means of these upper and lower connections it is securely anchored within the thoracic cavity. It is also attached to the posterior surface of the sternum by the superior and inferior sternopericardiac ligaments; the upper passing to the manubrium, and the lower to the xiphoid process. The serous pericardium is, as already stated, a closed sac which lines the fibrous pericardium and is invaginated by the heart; it therefore consists of a visceral and a parietal portion. The visceral portion, or epicardium, covers the heart and the great vessels, and from the latter is continuous with the parietal layer which lines the fibrous pericardium. The portion which covers the vessels is arranged in the form of two tubes. The aorta and pulmonary artery are enclosed in one tube, the arterial mesocardium. The superior and inferior ven cav and the four pulmonary veins are enclosed in a second tube, the venous mesocardium, the attachment of which to the parietal layer presents the shape of an inverted U. The cul-de-sac enclosed between the limbs of the U lies behind the left atrium and is known as the oblique sinus, while the passage between the venous and arterial mesocardiai.e., between the aorta and pulmonary artery in front and the atria behindis termed the transverse sinus.

describe the normal pressure and flow patterns (including velocity profiles) of the cardiac cycle

The figure shows electrical and mechanical events of cardiac cycle:

The cardiac events that occur from the beginning of one heartbeat to the beginning of the next are called the cardiac cycle. Duration of one cardiac cycle is usually 0.8 sec, & consist systole & diastole which are further divided as follows: Phases of systole: 1. Isovolumetric contraction (0.05 sec) lasts from mitral valve closure until opening of aortic valve. Immediately after ventricular contraction begins, the ventricular pressure rises abruptly, causing the A-V valves to close. Then an additional 0.02 to 0.03 second is required for the ventricle to build up sufficient pressure to push the semilunar (aortic and pulmonary) valves open against the

7
pressures in the aorta and pulmonary artery. Therefore, during this period, contraction is occurring in the ventricles, but there is no emptying. 1. Isotonic contraction (0.18 sec) separated into phase of rapid ejection (0.1) & phase of reduced ejection (0.08 sec). When the left ventricular pressure rises slightly above 80 mm Hg (and the right ventricular pressure slightly above 8 mm Hg), the ventricular pressures push the semilunar valves open. Immediately, blood begins to pour out of the ventricles, with about 70 per cent of the blood emptying occurring during the first third of the period of ejection and the remaining 30 per cent emptying during the next two thirds. Therefore, the first third is called the period of rapid ejection, and the last two thirds, the period of slow ejection. 2. Protodiastole (0.04 sec) ejection has finished & pressure starts to fall until aortic valve closes Phases of diastole: 1. Isovolumetric relaxation (0.03-0.06): lasts from closure of the aortic valve until the mitral valve opens 2. Rapid filling phase: blood flows rapidly into the ventricles. The period of rapid filling lasts for about the first third of diastole. 3. Distasis (reduced filling): During the middle third of diastole, only a small amount of blood normally flows into the ventricles; this is blood that continues to empty into the atria from the veins and passes through the atria directly into the ventricles. 4. Atrial systole: During the last third of diastole, the atria contract and give an additional thrust to the inflow of blood into the ventricles; this accounts for about 20 per cent of the filling of the ventricles during each heart cycle.
Pressure Changes in the Atria:

The a wave: caused by atrial contraction. Ordinarily, the right atrial pressure increases 4 to 6 mm Hg during atrial contraction, and the left atrial pressure increases about 7 to 8 mm Hg. The c wave: occurs when the ventricles begin to contract; it is caused partly by slight backflow of blood into the atria at the onset of ventricular contraction but mainly by bulging of the A-V valves backward toward the atria because of increasing pressure in the ventricles. Pressure drop after c wave is x descent. The v wave: results from slow flow of blood into the atria from the veins while the A-V valves are closed during ventricular contraction. Then, when ventricular contraction is over, the A-V valves open, allowing this stored atrial blood to flow rapidly into the ventricles and causing the v wave to disappear. Pressure drop after c wave is y descent.
Pressure Changes in the Aorta:

Opening of semilunar valves allows blood to immediately flow out of the ventricle hence the pressure in the ventricle rises much less rapidly. The entry of blood into the arteries causes the walls of these arteries to stretch and the pressure to increase to about 120 mm Hg. Next, at the end of systole, after the left ventricle stops ejecting blood and the aortic valve closes, the elastic walls of the arteries maintain a high pressure in the arteries, even during diastole. A so-called incisura occurs in the aortic pressure curve when the aortic valve closes. This is caused by a short period of backward flow of blood immediately before closure of the valve, followed by sudden cessation of the backflow. After the aortic valve has closed, the pressure in the aorta decreases slowly throughout diastole because the blood stored in the distended elastic arteries flows continually through the peripheral vessels back to the veins. Before the ventricle contracts again, the aortic pressure usually has fallen to about 80 mm Hg (diastolic pressure). The pressure curves in the right ventricle and pulmonary artery are similar to those in the aorta, except that the pressures are only about one sixth as great.

Relationship of the Electrocardiogram to the Cardiac Cycle The P wave is caused by spread of depolarization through the atria, and this is
followed by atrial contraction, which causes a slight rise in the atrial pressure curve immediately after the electrocardiographic P wave.

8
About 0.16 second after the onset of the P wave, the QRS waves appear as a result of electrical depolarization of the ventricles, which initiates contraction of the ventricles and causes the ventricular pressure to begin rising, as also shown in the figure. Therefore, the QRS complex begins slightly before the onset of ventricular systole. ventricular T wave: This represents the stage of repolarization of the ventricles when the ventricular muscle fibers begin to relax. Therefore, the T wave occurs slightly before the end of ventricular contraction.

Explain the ionic basis of spontaneous electrical activity of cardiac muscle cells (automaticity) Cardiac action potential (normal duration 250 ms): Figure shows the cardiac action potential with 4 phases & ionic currents:

PHASE 0: Resting membrane has potential is close to -90mv. Depolarization to threshold voltage results in opening of the activation gates of the sodium channels. These are now active. Na diffuses in rapidly across electrochemical gradient and membrane rapidly reaches the sodium equilibrium potential Ena (about +70 MV). The sodium current is brief as the opening of activation gate is followed by the closure of the inactivation gate. Inactivation gate (h) have voltage dependent function. They begin to close between -70 to - 55 mv and begin to recover from -55 to -70 mv. PHASE 1 & 2: The action potential plateau (phases 1 and 2) reflects the turning off of most of the sodium current, the waxing and waning of calcium current, and the slow development of a repolarizing potassium current. Most calcium channels become

activated and inactivated in what appears to be the same way as sodium channels, but in the case of the most common type of cardiac calcium channel (the "L" type), the transitions occur more slowly and at more positive potentials. PHASE 3: (repolarization phase) results from completion of sodium and calcium channel inactivation and the growth of potassium permeability, so that the membrane potential once again approaches the potassium equilibrium potential close to -90mv. The major potassium currents involved in phase 3 repolarization include a rapidly activating potassium current (IKr) and a slowly activating potassium current (IKs). These two potassium currents are sometimes discussed together as "IK. PHASE 4: is the resting membrane potential. This is the period that the cell remains in until it is stimulated by an external electrical stimulus (typically an adjacent cell). This phase of the action potential is associated with diastole of the chamber of the heart. In addition to stimulus from adjacent cells, certain cells of the heart have the ability to undergo spontaneous depolarization, in which an action potential is generated without any influence from nearby cells

There are two types of action potential seen in heart: (Also see comparison table in Kerry pg 88) Fast response fibres ( cardiac muscle & HIS purkinje system) Slow response fibres (SA & AV nodes)

SA node and AV node have resting membrane potential in the range of 50 to -70 mv hence all na channels are inactivated. Such depolarized cells exhibit slow responses slow upstroke velocity and slow conduction which depends on calcium inward current. Other relatively depolarized cells exhibiting slow depolarization & conduction include cells exposed to hyperkalemia, sodium pump blockade, or ischemic cells.

10

Pacemaker Potentials: Rhythmically discharging cells (eg SA & AV nodes)have a membrane potential that, after each impulse, declines to the firing level. Thus, this prepotential or pacemaker potential triggers the next impulse. At the peak of each impulse, IK begins and brings about repolarization. IK then declines, and as K+ efflux decreases, the membrane begins to depolarize, forming the first part of the prepotential. Ca2+ channels then open. These are of two types in the heart, the T (for transient) channels and the L (for long-lasting) channels. The calcium current (ICa) due to opening of T channels completes the prepotential, and ICa due to opening of L channels produces the impulse. Describe the normal and abnormal processes of cardiac excitation Normal process of cardiac excitation: SA node is the normal pacemaker of heart. It has the steepest phase 4 pacemaker current & hence depolarizes first. Depolarization initiated in the SA node spreads radially through the atria, then converges on the AV node. Atrial depolarization is complete in about 0.1 s. Because conduction in the AV node is slow, there is a delay of about 0.1 s (AV nodal delay) before excitation spreads to the ventricles. This delay is shortened by stimulation of the sympathetic nerves to the heart and lengthened by stimulation of the vagi. From the top of the septum, the wave of depolarization spreads in the rapidly conducting Purkinje fibers to all parts of the ventricles in the 0.08-0.1 s. In humans, depolarization of the ventricular muscle starts at the left side of the interventricular septum and moves first to the right across the midportion of the septum. The wave of depolarization then spreads down the septum to the apex of the heart. It returns along the ventricular walls to the AV groove, proceeding from the endocardial to the epicardial surface. The last parts of the heart to be depolarized are the posterobasal portion of the left ventricle, the pulmonary conus, and the uppermost portion of the septum. The velocity of conduction of the excitatory action potential signal along both atrial and ventricular muscle fibers is about 0.3 to 0.5 m/sec. The velocity of conduction in the Purkinje fibersis as great as 4 m/sec. The conduction velocity across SA & AV node is 0.05m/sec. The normal refractory period of the ventricle is 0.25 to 0.30 second, which is about the duration of the prolonged plateau action potential. There is an additional relative refractory period of about 0.05 second during which the muscle is more difficult than normal to excite but nevertheless can be excited by a very strong excitatory signal. Abnormal process of cardiac excitation: (Arrythmias)

11

Abnormal automaticity: Non-pacemaker cells begin to spontaneously and abnormally initiate an impulse, believed to be the result of reduced (more positive) RMP bringing it closer to the threshold potential. Eg. Ischemia and electrolyte imbalances Acceleration of pacemaker discharge, brought about by increased phase 4 depolarization slope. Eg. hypokalemia, stimulation, positive chronotropic drugs, fibre stretch, acidosis and partial depolarization by currents of injury.

After depolarization (or triggered activity): Spontaneous depolarizations requiring a preceding impulse (a triggering beat) Early after depolarizations (EAD): After depolarizations originating during phase 2 or 3 of the AP. Seen with prolonged action potential eg. Prolongation of QT interval (repolarization) by inhibition of delayed rectifier potassium current (sotalol, quinidine, dofetilide and procainamide). Torsade de pointe (TdP), a potentially lethal polymorphic ventricular arrhythmia, is an example of EAD, precipitated by K+channel blockers

12 Delayed afterdepolarization (DAD): After depolarizations originating during phase 4 of AP. Ventricular arrhythmias secondary to digoxin toxicity is an example of delayed afterdepolarization. Digoxin mediated increased intracellular Ca++ is believed to be the mechanism of this type of arrhythmia.

Disorders of impulse conduction: Most common mechanism of arrhythmias. conduction block: 1st , 2nd & 3rd degree heart block reentry: o o Impulse recirculates in the heart and cause repititive activation Pre-requisites: Propagating impulse encounters electrophysiologically inhomogeneous tissue with unidirectional block allowing retrograde conduction retrograde conducting impulse encounters excitable tissue

Examples of reentrant arrhythmias: AV nodal reentrant tachycardia (AVNRT), Atrioventricular reentrant tachycardia (AVRT), Atrial flutter, Atrial fibrillation, Ventricular tachycardia.

To explain the physiological basis of the electrocardiograph in normal and common pathological states
Normal electrocardiograph:

13

Feature

Description

Duration

P wave

During normal atrial depolarization, the main electrical vector is directed from the SA node towards the AV node, and spreads from the right atrium to the left atrium. This turns into the P wave on the ECG.

80ms

PR segment

The PR segment connects the P wave and the QRS complex.

150 to 200ms

QRS The QRS complex is a recording of a single heartbeat on the ECG that corresponds to the 70 to complex depolarization of the right and left ventricles. 110ms

ST segment

The ST segment connects the QRS complex and the T wave. Is iso-electric. Represents period when ventricles are depolarized.

80 to 120ms

T wave

The T wave represents the repolarization (or recovery) of the ventricles. The interval from the beginning of the QRS complex to the apex of the T wave is referred to as the absolute refractory period. The last half of the T wave is referred to as the relative refractory period (or vulnerable period).

160ms

PR interval

The PR interval is measured from the beginning of the P wave to the beginning of the QRS complex. Represents AV nodal conduction delay.

120 to 200ms

ST interval

The ST interval is measured from the J point to the end of the T wave.

320ms

QT

The QT interval is measured from the beginning of the QRS complex to the end of the T 300 to

14

interval

wave. Represents the entire period of depolarization & repolarization of the ventricles.

440ms

U wave

The U wave is not always seen. It is typically small, and, by definition, follows the T wave.

ECG in common pathological states Pathological state Abnormal potential Pericardial effusion Tamponade Old infarcts Emphysema
Axis Deviation

ECG finding Low voltage complexes

Mechanism Fluid decreases conduction Less myocardial mass Parenchymal Air decreases conduction

Left ventricle hypertrophy LBBB Right ventricle hypertrophy RBBB Abnormal P wave Left atrial hypertrophy Right atrial hypertrophy Abnormal PR interval

Left axis deviation Right axis deviation

1. Larger

myocardium increased action potential 2. Greater time to depolarize Mechanisms similar to ventricular hypertrophy

P mitrale (biphasic p wave) P pulmonale (tall p wave)

Abnormal QRS complex Hypertrophic/dilated Broad QRS heart Purkinje block Broad, bizarre QRS syndromes Ventricular ectopics Abnormal QT Abnormal T wave Chronic progressive ischemia Hyperkalemia Digitalis overdose Current of injury Mechanical trauma Inverted t waves Earliest sign: biphasic t waves ST changes

Increased mass/increased path of depolarization Normal pattern of depolarization lost

Slow conduction, abnormal repolarization Damaged myocardium

15

Infectious process Ischemia

NSTEMI STEMI Anterior wall MI Lateral wall MI Inferior wall MI Electrolyte disorder Hyperkalemia

ST depression ST elevation V1-V4 ST changes I, aVL, V5, V6 ST changes II, III, aVF ST changes ECG changes

remains partially or totally depolarized, resulting in flow of current (current of injury) to adjacent myocardium when it begins repolarizing.

Hypokalemia

Hypercalcemia

Hypocalcemia

Tall peaked T waves Flattening p-waves. In extreme hyperkalemia p-waves may disappear altogether. Prolonged depolarization leading to QRS widening (nonspecific intraventricular conduction defect) sometimes > 0.20 seconds At concentrations > 7.5 mmol/L atrial and ventricular fibrillation can occur ST depression and flattening of the T wave Negative T waves A U-wave may be visible mild: broad based tall peaking T waves severe: extremely wide QRS, low R wave, disappearance of p waves, tall peaking T waves. narrowing of the QRS complex reduced PR interval T wave flattening and inversion prolongation of the QT-interval prominent U-wave prolonged ST and ST-depression

drugs Quinidine, phenothiazines and tricyclic antidepressants

Lidocaine

ecg Low voltage T waves (or T wave inversion) ST segment depression Prolonged Q-T interval Increased height of U wave Widening and notching of P waves Toxic doses of quinidine may cause widened QRS complexes, heart block, VT and VF. No effects at therapeutic doses. Toxic doses may cause sinus tachycardia, sinus arrest and AV block.

16

Diphenylhydantoi n (Phenytoin)

Amiodarone Verapamil

Digoxin

No noticeable changes occur at normal doses. Occasionally, an increase in PR interval may be seen. With pre-existing severe myocardial disease, the drug has been associated with bradycardia, A-V block, asystole or VF. Prolongation of the Q-T interval and increase in the height of the U waves occurs. This correlates with its effect of prolongation of the action potential. slowing of the sinus rate and AV conduction (hence, a prolonged PR interval). The effects of verapamil on the SA and AV nodes are additive with beta-blocking drugs. The use of these two together can give rise to catastrophic bradycardias. Digoxin can induce direct and indirect changes on the heart. The direct changes are due to inhibition of the normal active process of sodium ion transport (and also potassium ion transport) across the membranes of myocardial and pacemaker cells. Digoxin induces indirect changes by increasing the vagal tone. Therapeutic doses produce ECG changes in a patient taking digitalis. These changes are referred to as the "digoxin effect". These changes are: 1) 2) 3) 4) Decreased T wave amplitude ST segment depression Increase in U wave amplitude Shortening of the Q-T interval

One of the earliest and commonest changes is reduction in T wave voltage. Occasionally, biphasic or inverted T waves may be seen. ST segment changes are seen as a downward sloping ST segment depression, which is often associated with T wave flattening. This is called the "reversed tick" phenomenon (resembles the tick made by a lefthanded person). Digoxin toxicity: The following arrhythmias are seen commonly: 1) Ventricular premature beats (including coupled and multifocal VPCs) 2) Junctional tachycardia 3) Sinus bradycardia 4) Atrial tachycardia with A-V block 5) Heart blocks (1st degree, 2nd degree Mobitz Type I and 3rd degree)

17

6) 7) 8) 9)

Multifocal atrial premature beats Atrial fibrillation and flutter SA block and sinus arrest VF and VT

Drugs causing prolonged QT: Antibiotics Anaesthetics azithromycin halothane clarithromycin erythromycin Antiarrhythmics roxithromycin disopyramide metronidazole procainamide (with alcohol) quinidine Some fluroquinolone amiodarone sotalol Antifungals fluconazole Antidepressants (in cirrhosis) amitriptyline ketoconazole clomipramine imipramine Antivirals dothiepin nelfinavir doxepin Antimalarials chloroquine mefloquine

Antipsychotics risperidone fluphenazine haloperidol clozapine thioridiazine ziprasidone pimozide droperidol Antihistamines terfenadine* astemizole* Other probucol cisapride

Describe the factors that may influence cardiac electrical activity


Factors affecting electrical activity: 1. Nervous system 2. Electrolytes 3. Drugs 4. Temperature 5. Anatomical alterations: a. Myocardial ischemia/infarction b. Aberrant pathway c. Bundle branch blocks Nervous system: (Also read the nerve supply under anatomy question)

18

Nodal tissue especially SA node is heavily innervated by both PANS (acetylcholine) & SANS (norepinephrine) fibres activating M2 & b1 receptors respectively Phase 4 slope is increased by an increase in cAMP resulting from b1 receptor activation and slowed by a decrease in cAMP resulting from M2 receptor activation Increase in cAMP will: - Increase upstroke velocity in pacemaker by increase of iCa - Shorten AP duration by increase of IK - Increase HR by increase of Ina, thus increasing slope of phase 4 Decrease in cAMP will: - Does the opposite plus produces a K+ current (IKIACh), which produces hyperpolarization and slows the rate of diastolic depolarization and thus decreases HR by both SA & AV nodal depression. Strong vagal stimulation can cause complete heart block with ventricular escape. - Beta blockers prevent cAMP formation, with primary effects on SA & AV nodal tissues. Electrolyte disorder Hyperkalemia Effect on cardiac electrical activity The faster repolarization of the cardiac action potential causes the tenting of the T waves, and the inactivation of sodium channels causes a sluggish conduction of the electrical wave around the heart, which leads to smaller P waves and widening of the QRS complex. Bradycardia can occur. In the heart, it causes myocytes to become hyperexcitable. Lower membrane potentials in the atrium may cause arrhythmias because of more complete recovery from sodium-channel inactivation, making the triggering of an action potential more likely. In addition, the reduced extracellular potassium (paradoxically) inhibits the activity of the IKr potassium current and delays ventricular repolarization. This delayed repolarization may promote reentrant arrythmias.
prolongs phase 2 of the action potential. See ecg changes in previous question

Hypokalemia

Hypocalcemia Hypercalcemia Drugs Class I A (Quinidine, procainamide, disopyramide) Class I B (lidocaine,

See ecg changes in previous question Effect on action potential Slow phase 0 depolarization, prolong APD Shorten phase 3 repolarization & Effect on ECG Supress AV conduction, prolong PR, QRS, QT Usually no ECG changes

19

tocainide, mexiletine) Class IC (Flecainide, propafenone) B blockers Class III (sotalol, amiodarone) CCB verapamil & diltiazem

decrease APD Markedly slow phase 0 depolarization. variable effect on APD Slow phase 4 depolarization Prolong phase 3 repolarization Slow phase 4 spontaneous depolarization Markedly delays conduction, prolong PR, broaden QRS Prolong PR, decreased HR Prolonged QT Prolonged PR

Effect of temperature: Increased body temperature, causes a greatly increased heart rate, sometimes to as fast as double normal. Decreased temperature causes a greatly decreased heart rate. These effects presumably result from the fact that heat increases the permeability of the cardiac muscle membrane to ions that control heart rate, resulting in acceleration of the selfexcitation process. Anatomical alterations: a. Myocardial ischemia/infarction: results in current of injury. Reperfusion injury produces increased automaticity and ectopics. Re-entry arrhythmias. Scarred myocardium can delay in conduction or can lead to re-entry circuits. b. Aberrant pathway: eg VPW syndrome can cause AV reentrant tachycardias c. Bundle branch blocks: primarily result in delayed conduction.
Explain the Frank-Starling mechanism and its relationship to excitation-contraction coupling

Frank-Starling law: "energy of contraction is proportional to the initial length of the cardiac muscle fiber." For the heart, the length of the muscle fibers (ie, the extent of the preload) is proportionate to the end- diastolic volume. Increased venous return increases the end-diastolic volume and therefore preload, which is the initial stretching of the cardiac myocytes prior to contraction. Myocyte stretching increases the sarcomere length, which causes an increase in force generation. Increasing the sarcomere length increases troponin C calcium sensitivity, which increases the rate of crossbridge attachment and detachment, and the amount of tension developed by the muscle fiber. The effect of increased sarcomere length on the contractile proteins is termed length-dependent activation.

20

This ability of the heart to change its force of contraction and therefore stroke volume in response to changes in venous return is called the FrankStarling mechanism. The mechanism is very important for: 1. Rapidly responding to acute changes in venous return 2. Keeping the right and left ventricle outputs exactly equal

The relation between ventricular stroke volume and end-diastolic volume is called the Frank-Starling curve.
Positive inotropic effect

Negative inotropic effect

Frank-Starling curves however, does not show how changes in venous return affect end-diastolic and end-systolic volume. In order to do this, it is necessary to describe ventricular function in terms of pressure-volume diagrams. The increased stroke volume is manifested by an increase in the width of the pressure-volume loop.

Steps in excitation contraction coupling: 1. Action potential spreads from the cell membrane into the T tubules 2. Inward calcium current: During the plateau of action potential, Ca2+ conductance is increased and calcium enters the cell from the extracellular fluid

21 3. Ca2+ induced Ca2+ release: the Ca2+ entry triggers release of

4.
5. 6.

7.

even more Ca2+ from the sarcoplasmic reticulum. The amount of calcium released from SR depends upon the amount of calcium stored and on the size of inward current during the plateau phase Net intracellular calcium increases Ca2+ binds to troponin C: & tropomyosin is moved out of the way, removing the inhibition of actin and myosin binding. Actin and myosin bind: Thick and thin filament slide past each other and myocardial cell contracts. The magnitude of tension is proportional to intracellular calcium Relaxation occurs when Ca2+ is re-accumulated by the SR by an active Ca2+ - ATPase pump

Define preload, afterload and myocardial contractility

Preload: is the load on myocardial cell just before the onset of contraction. The pre-load is said to be initial fiber length. After load: is the impedance to the ejection of blood from the heart into the arterial circulation. Myocardial contractility: is the intrinsic ability of the cardiac muscle to develop force at a given muscle length independent of changes in HR or After-load.
Describe the factors that determine preload, afterload and myocardial contractility

Factors determining preload: Factors that normally increase or decrease the length of ventricular cardiac muscle fibers. Increase Preload (increased venous return) Stronger atrial contractions : Atrial contractions aid ventricular filling Increased total blood volume : An increase in total blood volume increases venous return. Increased venous tone : Constriction of the veins reduces the size of the venous reservoirs, decreasing venous pooling and thus increasing venous return Increased pumping action of skeletal muscle : muscular activity increases it as a result of the pumping action of skeletal muscle. Increased negative intrathoracic pressure : An increase in the normal negative intrathoracic pressure increases the pressure gradient along which blood flows to the heart, whereas a decrease impedes venous return Decrease Preload (decreased venous return) Standing : Standing decreases venous return Increased intrapericardial pressure Decreased ventricular compliance : An increase in ventricular stiffness produced by myocardial infarction, infiltrative disease, and other abnormalities

22

Factors determining Afterload:


Afterload is related to ventricular wall stress (), where

(P, ventricular pressure; r, ventricular radius; h, wall thickness). This relationship is similar to the Law of LaPlace, which states that wall tension (T) is proportionate to the pressure (P) times radius (r) for thin-walled spheres or cylinders. Therefore, wall stress is wall tension divided by wall thickness.

Determinants of after load: Systemic vascular resistance Aortic root impedance Transmural pressure across ventricular wall: Myocardium has to contract against a negative intrathoracic pressure which constantly produces outward pull on myocardium. Ventricular wall thickness: A hypertrophied ventricle (thickened wall) reduces wall stress and afterload. The thicker the wall, the less tension experienced by each sarcomere unit. Ventricular radius: At a given pressure, wall stress and therefore afterload are increased by an increase in ventricular inside radius (ventricular dilation). Factors determining myocardial contractility: Factors that increase contractility: 1. Increased heart rate: Increased HR increased frequency of action potential more Ca2+ enters cellmore Ca2+ released from SR and greater tension is produced. Examples: a. Positive staircase or bowditch staircase. Increased HR increases the force of contraction in a stepwise fashion as the intracellular Ca2+ increases cumulatively over several beats b. Post extrasystolic potentiation: beat after extrasystole has increased force of contraction because extra Ca2+ enters during extrasystole. 2. Sympathetic stimulation via b1 receptors increases contractility by two mechanisms: a. Increases inward Ca2+ current during the plateau of each action potential b. Increases the activity of Ca2+ pump of SR by phosphorylation of phasopholamban resulting in more accumulation and subsequent release of Ca2+

23 3. Cardiac glycosides: inhibits Na K ATPaseintracellular Na+

increasesdiminished Na+ gradientinhibits Na-Ca exchange that extrudes Ca2+ out of cell and depends on Na gradient. 4. Xanthines such as caffeine and theophylline that inhibit the breakdown of cAMP are positively inotropic. Glucagon, which increases the formation of cAMP, is positively inotropic. Factors that decrease contractility: 1. Parasympathetic stimulation: via M2 receptor decreases the force of contraction in the atria by decreasing the inward Ca2+ current during the plateau phase of the action potential. 2. Hypercapnia, hypoxia, acidosis 3. Drugs such as quinidine, procainamide, and barbiturates 4. The contractility of the myocardium is also reduced in heart failure (intrinsic depression). The cause of this depression is not known. Describe myocardial oxygen demand and supply, and the conditions that may alter each

MYOCARDIAL OXYGEN DEMAND The basal myocardial O2 consumption in asystole: 2 mL/100 g/min. O2 consumption by the beating heart is about 9 mL/100 g/min at rest. Cardiac venous O2 tension is low i.e PO2 of 20mmhg , and little additional O2 can be extracted from the blood in the coronaries, so increases in O2 consumption require increases in coronary blood flow. Tension time index: Area under the systolic part of LV pressure time curve. Correlates well with the myocardial oxygen consumption

24

Determinants of myocardial oxygen consumption: o Major determinants: Myocardial wall tension: law of Laplace states that the tension developed in the wall of a hollow viscus is proportionate to the radius of the viscus, and the radius of a dilated heart is increased. Hence increased pre-load also leads to increased O2 consumption. Contractility Heart rate- O2 consumption per unit time increases when the heart rate is increased by sympathetic stimulation because of the increased number of beats and the increased velocity and strength of each contraction. o Minor determinants: Basal energy metabolism (25% of total O2 consumption) External work performed: Work is measured as a product of pressure and volume. The oxygen cost of myocardial work depends on the way the work is performed eg pressure work like AS requires higher myocardial oxygen consumption that volume work like AR for the same cardiac output. Energy for electrical activation (1% of total O2 consumption)

MYOCARDIAL OXYGEN SUPPLY Oxygen delivery = coronary blood flow * oxygen content = Coronary blood flow * 1.34*Hb*SO2 Coronary blood flow: Coronary blood flow is 200-250mls/min. This is 5% of CO Coronary perfusion pressure: is the driving force for the coronary blood flow and is calculated as the aortic diastolic pressure the larger of either LV diastolic pressure or the RA pressure (representing the coronary sinus pressure). Usually the perfusion pressure in circulations is just the difference between the arterial and venous pressure, however the circulations like heart, lung and brain are examples of starling resistors where another pressure needs to be considered eg LV pressure when calculating the perfusion pressure. Coronary blood flow variation with the cardiac cycle: o LV: flow predominantly during diastole. Subendocardial flow ceases during systole. o RV: flow during both systole and diastole, as RV pressures are low during both systole and diastole.

25

Blood flow in left coronary

Significant flow only in diastole

Blood flow in right coronary 0 Systo le

Significant flow both in systole & diastole

Diastol e

Most important factors regulating coronary blood flow is vasodilatation produced by the local metabolic factors: hypoxia and adenosine. Sympathetic nerves play a minor role.

Describe and explain cardiac output curves, vascular function curves and their correlation

Cardiac and vascular curves are simultaneous plots of cardiac output and venous return as a function of the right atrial pressure or end diastolic volume 1. The cardiac output or the cardiac function curve a. Depicts the frank starling relationship for the ventricle b. Shows that cardiac output increases as a function of end diastolic volume(primary mechanism) 2. The venous return or vascular function curve a. Depicts the relationship between blood flow through the vascular system and right atrial pressure b. Mean systemic pressure: i. Point at which vascular function curve intersects the X axis ii. Equals the right atrial pressure when there is no flow in the cardiovascular system iii. Increased by increase in blood volume or by decrease in venous compliance and is reflected by a shift of the vascular function curve to the right iv. Decreased by decrease in blood volume or by increase in the venous compliance and is reflected by a shift of the vascular function curve to the left c. Slope of the venous return curve: determined by the resistance of the arterioles

26

i. Clockwise rotation of venous return curve indicates a decrease in total peripheral resistance (TPR). Decrease in TPR increases the venous return ii. Anticlockwise rotation of venous return curve indicates a increase in TPR. Increase in TPR decreases the venous return. 3. Combining cardiac output & venous return curves a. The point at which the two curves intersect is the equilibrium or the steady state point Here the cardiac output equals venous return b. Cardiac output can be changed by altering the cardiac output curve, the venous return curve or both the curves simultaneously. The superimposed curves can be used to predict the direction or magnitude of changes in cardiac output. Eg of such changes are as follows: i. Inotropic agents change the cardiac output curve 1. Positive inotropic agents produce increased cardiac output. The equilibrium point shifts to a higher CO and a correspondingly lower RA pressure. RA pressure decreases because more blood is ejected from the heart on each beat 2. Negative inotropic agents produce decreased contractility and decreased CO ii. Changes in blood volume or venous compliance 1. Increases in blood volume or decreases in venous compliance increases mean systemic pressure, shifting the venous return curve to the right in parallel fashion. A new equilibrium point is established at which both cardiac output and RAP are increased 2. Decreases in blood volume or increases in venous compliance decrease mean systemic pressure and shift the venous return curve to left in parallel fashion. A new equilibrium point is established at which both cardiac output and RAP are decreased. iii. Changes in TPR change both the cardiac output & venous return curve 1. Increases in TPR causes a decrease in both cardiac output and venous return. There is counterclock wise rotation of venous return curve and there is a downward shift of the cardiac output curve. A new equilibrium point is reached at which both CO & venous return are decreased but right atrial pressure is unchanged. 2. Decreasing TPR causes an increase in both CO and venous return. There is clockwise rotation of venous return curve and upward shift of the cardiac output curve. A new equilibrium point is

27

established at which both cardiac output and venous return are increased.

Describe the pressure-volume relationships of the ventricles and their clinical applications

1 2
12- effective arterial elastance

28

In the above figure 10 also represents end systolic elastance Best index of preload LVEDV Best index of afterload slope of the line joining the LVEDV on x axis to the end systolic point. (line no. 12 in the above figure) also known as effective arterial elastance line(Ea) The pressure volume graph can be used to predict the changes in pressurve volume relationship of heart produced by changes in preload, afterload or contractility Increased Preload Referes to increase in end diastolic volume and is the result of increased venous return Causes an increase in the stoke volume based on frank starling mechanism and is reflected by the increased width of the loop. In the figure showing increased preload the afterload lines of the 2 loops are parallel so they have same afterload. Both end systolic points are on the same contractility line so they have same contractility Increased afterload Refers to increased aortic pressure Results in the decreased stroke volume reflected by decreased width of the pressurve volume loop Decrease in the stroke volume results in increased end systolic volume In the figure the preload is same in two loops because the EDV is same and contractility is also same as the end systolic point of the two loops are on the same contractility line Increased contractility The ventricle develop greater than uisual tension during systole causing an increase in the stroke volume represented by the increased width of the PV loop The increase in the stroke vol. decrease the end systolic volume

29

In the figure the slope of the end systolic pressure volume line is increased in for the loop 2 representing the increased contractility. The end systolic point of the two loops are on the same afterload line hence after load is same for the loops. Preload i.e. EDV is same for the two loops

Increased Preload

30

Increased afterload

Increased contractility
Describe the factors Contractilit that determine cardiac output Pre After load load y

Myocardial fiber shortening

Left ventricular size

Heart rate

Stroke volume

Cardiac output

Peripheral resistance

Arterial pressure

31

Interactions between the components that regulate cardiac output and arterial pressure. Solid lines indicate increases, and the dashed line indicates a decrease.

Effect of various conditions on cardiac output. Condition or Factor1 No change Sleep Moderate changes in environmental temperature Increase Anxiety and excitement (50-100%) Eating (30%) Exercise (up to 700%) High environmental temperature Pregnancy Epinephrine Decrease Sitting or standing from lying position (20-30%) Rapid arrhythmias Heart disease 1 Approximate percent changes are shown in parentheses.

Variations in cardiac output can be produced by changes in cardiac rate or stroke volume. The cardiac rate is controlled primarily by the cardiac innervation, sympathetic stimulation increasing the rate and parasympathetic stimulation decreasing it. The stroke volume is also determined in part by neural input, sympathetic stimuli making the myocardial muscle fibers contract with greater strength at any given length and parasympathetic stimuli having the opposite effect. The force of contraction of cardiac muscle is dependent upon its preloading and its afterloading. Preload: is the load on myocardial cell just before the onset of contraction. The pre-load is said to be initial fiber length. After load: is the impedance to the ejection of blood from the heart into the arterial circulation. Relation of Tension to Length in Cardiac Muscle Frank starling law: "energy of contraction is proportional to the initial length of the cardiac muscle fiber." For the heart, the length of the muscle

32

fibers (ie, the extent of the preload) is proportionate to the end- diastolic volume. Regulation of cardiac output as a result of changes in cardiac muscle fiber length is sometimes called heterometric regulation, whereas regulation due to changes in contractility independent of length is sometimes called homometric regulation. Increase Preload (increased venous return) Stronger atrial contractions : Atrial contractions aid ventricular filling Increased total blood volume : An increase in total blood volume increases venous return. Increased venous tone : Constriction of the veins reduces the size of the venous reservoirs, decreasing venous pooling and thus increasing venous return Increased pumping action of skeletal muscle : muscular activity increases it as a result of the pumping action of skeletal muscle. Increased negative intrathoracic pressure : An increase in the normal negative intrathoracic pressure increases the pressure gradient along which blood flows to the heart, whereas a decrease impedes venous return Decrease Preload (decreased venous return) Standing : Standing decreases venous return Increased intrapericardial pressure Decreased ventricular compliance : An increase in ventricular stiffness produced by myocardial infarction, infiltrative disease, and other abnormalities Factors determining Afterload:
Afterload is related to ventricular wall stress (), where

(P, ventricular pressure; r, ventricular radius; h, wall thickness). This relationship is similar to the Law of LaPlace, which states that wall tension (T) is proportionate to the pressure (P) times radius (r) for thin-walled spheres or cylinders. Therefore, wall stress is wall tension divided by wall thickness.

Determinants of after load: Systemic vascular resistance Aortic root impedance Transmural pressure across ventricular wall: Myocardium has to contract against a negative intrathoracic pressure which constantly produces outward pull on myocardium.

33

Ventricular wall thickness: A hypertrophied ventricle (thickened wall) reduces wall stress and afterload. The thicker the wall, the less tension experienced by each sarcomere unit. Ventricular radius: At a given pressure, wall stress and therefore afterload are increased by an increase in ventricular inside radius (ventricular dilation).

Myocardial Contractility The contractility of the myocardium exerts a major influence on stroke volume. Factors that increase contractility: 5. Increased heart rate: Increased HR increased frequency of action potential more Ca2+ enters cellmore Ca2+ released from SR and greater tension is produced. Examples: a. Positive staircase or bowditch staircase. Increased HR increases the force of contraction in a stepwise fashion as the intracellular Ca2+ increases cumulatively over several beats b. Post extrasystolic potentiation: beat after extrasystole has increased force of contraction because extra Ca2+ enters during extrasystole. 6. Sympathetic stimulation via b1 receptors increases contractility by two mechanisms: a. Increases inward Ca2+ current during the plateau of each action potential b. Increases the activity of Ca2+ pump of SR by phosphorylation of phasopholamban resulting in more accumulation and subsequent release of Ca2+ 7. Cardiac glycosides: inhibits Na K ATPaseintracellular Na+ increasesdiminished Na+ gradientinhibits Na-Ca exchange that extrudes Ca2+ out of cell and depends on Na gradient. 8. Xanthines such as caffeine and theophylline that inhibit the breakdown of cAMP are positively inotropic. Glucagon, which increases the formation of cAMP, is positively inotropic. Factors that decrease contractility: 5. Parasympathetic stimulation: via M2 receptor decreases the force of contraction in the atria by decreasing the inward Ca2+ current during the plateau phase of the action potential. 6. Hypercapnia, hypoxia, acidosis 7. Drugs such as quinidine, procainamide, and barbiturates 8. The contractility of the myocardium is also reduced in heart failure (intrinsic depression). The cause of this depression is not known.
Describe the distribution of blood volume and flow in the various regional circulations and to explain the factors that may result in redistribution of blood

34

Circulation (% of resting CO) Coronary (5%) Cerebral (15%)

Local metabolic control Most important mechanism Most important mechanism

Vasoactive metabolites Hypoxia Adenosine CO2 H+

Sympathetic control Least important mechanism Least important mechanism

Mechanical effects Mechanical compressio n during systole Increases in intracranial pressure decrease cerebral blood flow Muscular activity

Muscle (20%)

Most important

Lactate K+

Most important mechanism at

35

mechanism during exercise Skin (5%) Least important mechanism Most important mechanism

Adenosine

Pulmonary (100%) Renal (25%)

Hypoxia vasoconstric ts

rest (vasoconstricti on, vasodilatation) Most important mechanism is temperature regulation Least important mechanism

causes temporary decrease in blood flow

Lung inflation

Mechanisms of Blood Flow Control Local blood flow control can be divided into two phases: (1) acute control : achieved by rapid changes in local vasodilation or vasoconstriction of the arterioles, metarterioles, and precapillary sphincters, (2)long-term control : increase or decrease in the physical sizes and numbers of actual blood vessels supplying the tissues.

Acute Blood Flow Regulation

Local (intrinsic) control of blood flow: 1. Autoregulation: a. Blood flow to the organ remains constant over wide range of perfusion pressure b. Organs with auto regulation: heart, brain, kidney 2. Active hyperemia: a. Blood flow to the organ is proportional to its metabolic demand 3. Reactive hyperemia: a. Increase in blood flow to an organ that occurs after a period of occlusion of flow b. The longer the period of occlusion the greater the increase in blood flow above preocclusion levels Mechanisms that explain local control of blood flow 1. Myogenic hypothesis: a. explains autoregulation but not active or reactive hyperemia b. based on observation that vascular smooth muscle contracts when it is stretched c. increased perfusion pressure causes stretch of the vascular smooth muscle which contracts & produces vasoconstriction to maintain constant flow. 2. Metabolic hypothesis a. Based on observation that tissue supply of oxygen is matched to tissue oxygen demand

36

b. Vasodilator metabolites are produced as a result of metabolic activity in tissue. These vasodilators are CO2, H+, K+, lactate and adenosine. 3. Tissue pressure theory a. Applicabale in encapsulated organs (evidence lacking) b. As the perfusion pressure increasesincreased extravasation of fluidsincreased tissue hydrostataic pressuresmall vessels compressed 4. Mechanism for dilating upstream arteries when microvascular blood flow increases the endothelium-derived relaxing factor (nitric oxide): increase in blood flow increase endothelial stressrelease of EDRF (i.e. NO t1/2 of 6 sec) dilatation of arterioles decrease flow Humoral (extrinsic) control of blood flow: 1. Sympathetic innervations of vascular smooth muscle a. Increased sympathetic tone causes vasoconstriction b. Decreased sympathetic tone causes vasodilatation c. Density of innervations varies: skin has the greatest innervations, while coronary, pulmonary, and cerebral vessels have little innervations 2. Local vasoactive hormones a. Histamine i. Causes arteriolar dilatation and venous constrictionincreased capillary hydrostatic pressureedema ii. Released in reponse to trauma b. Bradykinin: i. Causes arteriolar dilatation and venous constrictionincreased capillary hydrostatic pressureedema c. Serotonin i. Arteriolar constriction ii. Released in reponse to blood vessel trauma iii. Implicated in vascular spasms of migrane d. Prostaglandins: i. Prostacyclin: vasodilator in several vascular beds ii. E series prostaglandins: vasodilators iii. F series prostaglandins: vasoconstrictors iv. Thromboxane A2: vasoconstrictor e. Endothelin: i. A Powerful Vasoconstrictor in Damaged Blood Vessels as large as 5 millimeters. ii. Present in the endothelial cells of all or most blood vessels, released by damage to the endothelium 3. Systemic vasoactive hormones a. Angiotensin II i. powerful potent vasoconstrictor of the small arterioles.

37

Normally acts on many of the arterioles of the body at the same time to increase the total peripheral resistance, thereby increasing the arterial pressure. b. Vasopressin i. Most potent vasoconstrictor ii. Formed in hypothalamus transported to the posterior pituitary gland, where it is finally secreted into the blood. iii. Concentration of circulating blood vasopressin after severe hemorrhage can rise high enough to increase the arterial pressure as much as 60 mm Hg.
ii. Long-Term Blood Flow Regulation:

Over a period of hours, days, and weeks, a long-term type of local blood flow regulation develops in addition to the acute regulation and gives far more complete regulation. Mechanism of long-term local blood flow regulation is principally to change the amount of vascularity of the tissues. If metabolic demand increases vascularization increase and vice-versa. Rapidity & extent of change decreases with age Vascularity Is Determined by Maximum Blood Flow Need Factors promoting vascularization: o O2: Oxygen is important not only for acute control of local blood flow but also for long-term control. Classic eg retrolental fibroplasia of neonate developing after removal of exposure to high O2 o Angiogenic factors: Important ones vascular endothelial growth factor (VEGF), fibroblast growth factor, and angiogenin released by the lack of O2
Explain the factors that determine systemic blood pressure and its regulation

Blood pressure = CO * total peripheral resistance The most important mechanisms for regulating arterial pressure are the fast, neutrally mediated baroreceptor mechanism and the slower hormonally regulated renin agiotensin aldosterone mechanism. Major mechanisms: 1. Baroreceptor reflex (Acute control) 2. Renin-angiotensin-aldosterone system (long term control) 3. Other mechanisms: a. Cerebral ischemia b. Chemoreceptor in carotid and aortic bodies c. Vasopressin d. ANP

38

Baroreceptor reflex 1. Includes fast neural mechanisms 2. Responsible for minute to minute regulation of arterial blood pressure 3. Produces vbasoconstrictor activity tonically which accounts for vasomotor tone 4. Baroreceptors are stretch receptors located within the wall of carotid sinus near the bifurcation of common carotid arteries 5. STEPS: a. Decrease in the arterial pressure decreases the stretch on the wall of carotid sinus. Baroreceptors are more sensitive to the change in pressure rather than the actual pressure. Additional baro-receptors in the aortic arch respond to increases but not to decreases in pressure b. Decreased stretch decreases the firing of the Herings nerve (cranial nerve IX) which carries signal to vasomotor centre in medulla. c. The set point for the mean arterial pressure in the vasomotor centre is about 100mmHg. Therefore if mean arterial pressure is less than 100mmHg a series of autonomic responses is coordinated by the vasomotor centre to correct the pressure d. The vasomotor centre responds to low pressure by decreasing parasympathetic outflow to the heart and increasing the sympathetic outflow to the heart and blood vessel. The results are as follows: i. HR ii. contractility: resulting from the increased sympathetic tone to the heart. Increased contractility + increased HR increased COincreased pressure iii. TPR due to vasoconstriction secondary to sympathetic outflow iv. venoconstriction secondary to sympathetic activity shifts vascular function to right venous return SVCO e. Baroreceptor mechanism is a negative feedback system. As the blood pressure picks up there will be increased stretch on the carotid sinus baroreceptors which will decrease the signals to vasomotor centre 6. Example of baroreceptor reflex mechanism: response to acute blood loss Renin-Angiotensin-aldosterone system 1. Slow hormonal mechanism used in long term pressure regulation by adjusting the blood volume 2. Steps: a. Decrease in the renal perfusion pressure causes the juxtaglomerular cells of the afferent arteriole to secrete rennin.

39

b. Renin catalyzes the conversion of angiotensinogen to angiotensin I in plasma c. ACE converts angiotensin I to physiologically active angiotensin II in lungs d. Angiotensin II has two effects: i. Stimulates the synthesis and secretion of aldosterone from adrenal cortex. Aldosterone increases NaCl reabsorption by the renal distal tubule thereby increasing blood volume and arterial pressure ii. Causes the vasoconstriction of the arterioles thereby increasing TPR and mean arterial pressure Other mechanisms of arterial pressure regulation: Cerebral ischemia 1. When the brain is ischemic the conc. Of CO2 and hydrogen ion in the brain tissue increases 2. Chemoreceptors in the vasomotor centre respond by increasing both sympathetic and parasympathetic outflow: a. Ventricular contractility and TPR are increased but HR is decreased because of overriding parasympathetic influence b. Peripheral vasoconstriction reduces blood flow to other organs to preserve blood flow to brain 3. Cushings reflex is an example of the response to cerebral ischemia. ICP compresses blood vessels and blood flow & cerebral ischemia Chemoreceptors in carotid and aortic bodies: 1. Located near the bifurcation of common carotid arteries and along the aortic arch 2. Have very high rates of O2 consumption and therefore are very sensitive to hypoxia. 3. A decrease in arterial pressure decreases O2 delivery to the chemoreceptors. In turn, information is sent vasomotor centre to increase BP Vasopressin 1. Involved in regulation of blood pressure in response to hemorrhage but not in minute to minute regulation of normal blood pressure 2. Atrial receptors respond to decreased blood pressure and cause the release of vasopressin from posterior pituitary 3. Vasopressin has two affects: a. Potent vasoconstrictor that increases TPR by V1 receptors in the arterioles b. Increases water reabsorption by the renal distal tubules and collecting ducts via V2 receptors ANP

40

1. Released from atria in response to increase atrial pressure 2. Causes relaxation of vascular smooth muscle cells, dilatation of arterioles and decreased TPR 3. Causes increased excretion of salt and water by the kidney, which reduces blood volume and attempts to bring arterial pressure down to normal 4. Inhibits renin secretion
Describe total peripheral vascular resistance and factors that affect it

Vascular resistance is a term used to define the resistance to flow that must be overcome to push blood through the circulatory system. The resistance offered by the peripheral circulation is known as the systemic vascular resistance (SVR) or total peripheral resistance. Units for measuring vascular resistance are dynscm-5 or pascal seconds per cubic metre (Pas/m). Pediatric cardiologists use hybrid reference units (HRU), also known as Wood units Measurement Reference Range Systemic vascular resistance 9001200 dyns/cm5 Pulmonary vascular resistance 100200 dyns/cm5 Factors affecting peripheral resistance: As per Ohms law: Resistance = P/Flow(1) As per poiseuilles law: Flow = Pr4/8L .(2) Combining equation (1) and (2) Resistance = 8 L/ r4 Hence Resistance L/r4

Where, represents viscosity L represents length of vessel r represents radius In other words, resistance is directly proportional to both the fluid viscosity and the structures length, and inversely proportional to the fourth power of the structures radius. Blood viscosity is not fixed but increases as hematocrit increases, and changes in hematocrit, therefore,

41

can have significant effects on the resistance to flow in certain situations. Under most physiological conditions, however, the hematocrit and, hence, viscosity of blood is relatively constant and does not play a role in the control of resistance. Similarly, since the lengths of the blood vessels remain constant in the body, length is also not a factor in the control of resistance along these vessels. In contrast, the radii of the blood vessels do not remain constant, and so vessel radius is the most important determinant of changes in resistance along the blood vessels. Decreasing the radius of a tube twofold increases its resistance sixteenfold. Sympathetic nervous system would cause generalized vasoconstriction and hence would increase TPR. There is a parallel arrangement of organs and their circulation. This is beneficial as parallel arrangement decreases total vascular resistance. This is because in parallel circuit, R=1 / [(1/R1) + (1/R2) + (1/R3)] Small arteries and arterioles are primary site of resistance. Describe the essential features of the micro-circulation including fluid exchange (Starling forces) and control mechanisms present in the pre- and post-capillary sphincters

Microcirculation is the term used to refer to the smallest blood vessels and include : smallest arterioles, metarterioles, the precapillary sphincters, the capillaries and the small venules. Structural aspects: Metarterioles branch into capillary beds. At the junction of the arterioles and capillaries is a smooth muscle band called the pre-capillary sphincter True capillaries do not have smooth muscle and consist only of a single layer of endothelial cell surrounded by basement membrane Systemic capillaries contain only about 5% of blood volume Clefts (pores) between the endothelial cells allow passage of water solube substances. Cleft represents very small fraction fraction of surface area (<0.1%) Blood flow through the capillaries is regulated by contraction and relaxation of the arterioles and precapillary sphincters Vasomotion: the smooth muscle of metarterioles and precapillary sphincters contract and relax regularly causing intermittent flow in the capillaries called vasomotion Functional aspects: Resistance: Arterioles are the major site of resistance and contribute to TPR

42

Reserve: in many tissues particularly in skeletal muscles, many capillaries remain collapsed. They provide reserve capacity and can open up when additional flow is required Regulation of temperature: microcirculation of skin has AV shunts which are under control of nervous system. Meant for heat regulation. Exchange: capillaries are the site where intravascular volume is in contact with the interstitial fluid. The transfer or exchange of substances is the principal function of microcirculation. This includes heat transfer.

Capillary fluid exchanges: Two processes- diffusion and filtration Diffusion: o high total diffusional turnover of 80000L/day however there is negligible net movement of water o bidirectional along the whole length of capillaries o net movement governed by concentration gradient o starling forces are not involved o this is the process responsible for net movement of gases, nutrients and wastes Filtration: o Actually an ultrafiltration as proteins can not cross the capillary membrane o Fluid movement is bi-directional o Net movement is governed by the starling forces o This process is not important for net movement of gases, nutrients and wastes but movement of water is important Passage of substances across capillary walls 1. Lipid soluble substances a. Cross capillary membrane by simple diffusion b. Include o2 and co2 2. Small water soluble substances a. Cross via water filled clefts between the endothelial cells b. Include water, glucose, and aminoacids c. Generally, protein molecules are too large to pass freely through the clefts d. In the brain the clefts between endothelial cells are exceptionally tight (blood brain barrier) e. In the liver and intestine, the clefts are exceptionally wide and allow passage of proteins. These capillaries are called sinusoids. 3. Large water soluble substances a. Can cross by pinocytosis

43

Fluid exchange across capillaries 1. The starling equation Jv= Kf[(Pc-Pi)-(c-i)] Jv = fluid movement (ml/min) Kf = hydraulic conductance or filtration co-efficient (ml/min*mmHg) Pc =capillary hydrostatic pressure Pi = interstitial hydrostatic pressure c =capillary oncotic pressure i =interistitial oncotic pressure =reflection coefficient Jv is fluid flow it is positive when the net fluid movement is out of capillary i.e filtration and is negative when the net fluid movement is into the capillary i.e. absorption Pc or capillary hydrostatic pressure: o Increase in Pc favors filtration out of capillary o Determined by arterial and venous pressures and resistance o Increase in arterial or venous pressure increases in Pc o Pc is higher at the arteriolar end of capillary than at the venous end

Pi is interstitial hydrostatic pressure o Increase in Pi opposes filtration out of the capillaries o Usually close to 0mmHg c Capillary oncotic pressure o An increase in capillary oncotic pressure opposes filtration out of capillaries o c is increased by the increase in the protein conc. In the blood & vice-versa o small solutes do not contribute to c i is the interstitial fluid oncotic pressure o an increase in the i favours filtration out of capillary o i is dependent on the protein concentration of interstitial fluid which is normally quiet low because very little protein is filtered Filtration coefficient: o It is the constant of proportionality in the flux equation o It is product of these two component: o K= Area*hydraulic conductivity

44

o A leaky capillary will have high filtration coefficient eg glomerular capillaries o Filtration coefficient measures leakiness of capillary membrane for water Reflection coefficient: o It is the correction applied to the measured oncotic pressure gradient across capillary walls to account for ineffectiveness of some of the measured oncotic pressure gradient o Value ranges from 0 to 1. o Eg CSF and glomerular fluid has very low proteins and the coefficient is close to 1 where is in hepatic sinusoids which are very leaky for proteins the coefficient is close to 0 o Measures leakiness of membrane for protein. Typical values for starling forces: Pc Pi c c Arteriolar end 25 -6 25 5 Venous end 10 -6 25 5

Lymph Function of lymph o Normally there is a net filtration of fluid out of capillaries which is returned to circulation via the lymph o Also returns any filtered protein Unidirectional flow of lymph o One way flap valves permit interstitial fluid to enter, but not leave, the lymph vessels o Flow through larger lymphatic vessels is also unidirectional and is aided by one way valves and skeletal muscle contraction. Edema o Occurs when the volume of interstitial fluid exceeds the capacity of the lymphatics to return it to the circulation o Can be caused by excess filtration or blocked lymphatics
Describe the anatomy relevant to central venous cannulation (including femoral, internal jugular, external jugular and subclavian veins)

Internal-External Jugular Approach Anatomy

45

The IJV emerges from the base of the skull through the jugular foramen and enters the carotid sheath dorsally with the internal carotid artery (ICA). It then courses posterolaterally to the artery and runs beneath the sternocleidomastoid (SCM) muscle. The vein lies medial to the anterior portion of the SCM muscle superiorly, then runs beneath the triangle formed by the two heads of the muscle in its medial portion before entering the SV near the medial border of the anterior scalene muscle at the sternal border of the clavicle. The junction of the right IJV (which averages 2 to 3 cm in diameter) with the right SV forming the innominate vein follows a straight path to the SVC. As a result, malpositions and looping of a catheter inserted through the right IJV are unusual. In contrast, a catheter passed through the left IJV must negotiate a sharp turn at the left jugulosubclavian junction, which results in a greater percentage of catheter malpositions. The ICA runs medial to the IJV but, rarely, may lie directly posterior or anterior. Behind the ICA, just outside the sheath, lie the stellate ganglion and the cervical sympathetic trunk. The dome of the pleura, which is higher on the left, lies caudal to the junction of the IJV and SV. Posteriorly, at the root of the neck, course the phrenic and vagus nerves. The thoracic duct lies behind the left IJV and enters the superior margin of the SV near the jugulosubclavian junction. The right lymphatic duct has the same anatomical relationship but is much smaller, and chylous effusions typically occur only with left-sided IJV cannulations. External Jugular Vein Approach Anatomy The EJV is formed anterior and caudal to the ear at the angle of the mandible by the union of the posterior auricular and retromandibular veins. It courses obliquely across the anterior surface of the SCM, then pierces the deep fascia just posterior to the SCM and joins the SV behind the medial third of the clavicle. In 5% to 15% of patients, the EJV is not a distinct structure but a venous plexus, in which case it may receive the ipsilateral cephalic vein. The EJV varies in size and contains valves throughout its course. Its junction with the SV may be at a severe, narrow angle that can be difficult for a catheter to traverse. Femoral Vein Approach Anatomy The FV is a direct continuation of the popliteal vein and becomes the external iliac vein at the inguinal ligament. At the inguinal ligament the FV lies within the femoral sheath a few centimeters from the skin surface. The FV lies medial to the femoral artery, which in turn lies medial to the femoral branch of the genitofemoral nerve. The medial compartment contains lymphatic channels and Cloquet's node. The external iliac vein courses cephalad from the inguinal ligament along the anterior surface of the iliopsoas muscle to join its counterpart from the other leg and form the (IVC) anterior to and to the right of the fifth lumbar vertebra Subclavian Vein Approach Anatomy The SV is a direct continuation of the axillary vein, beginning at the lateral border of the first rib, extending 3 to 4 cm along the undersurface of the

46

clavicle and becoming the brachiocephalic vein where it joins the ipsilateral IJV at Pirogoff's confluence behind the sternoclavicular articulation. The vein is 1 to 2 cm in diameter, contains a single set of valves just distal to the EJV junction, and is fixed in position directly beneath the clavicle by its fibrous attachments. These attachments prevent collapse of the vein, even with severe volume depletion. Anterior to the vein throughout its course lie the subclavius muscle, clavicle, costoclavicular ligament, pectoralis muscles, and epidermis. Posteriorly, the SV is separated from the subclavian artery and brachial plexus by the anterior scalenus muscle, which is 10 to 15 mm thick in the adult. Posterior to the medial portion of the SV are the phrenic nerve and internal mammary artery as they pass into the thorax. Superiorly, the relationships are the skin, platysma, and superficial aponeurosis. Inferiorly, the vein rests on the first rib, Sibson's fascia, the cupola of the pleura (0.5 cm behind the vein), and the pulmonary apex. The thoracic duct on the left and right lymphatic duct cross the anterior scalene muscle to join the superior aspect of the SV near its union with the IJV.

47

Describe the anatomy relevant to the insertion of an arterial line into a brachial, axillary, posterior tibial, dorsalis pedis, radial or femoral artery
The brachial artery The variations in the anatomy of the brachial artery are of great importance to the anaesthetist. They include: 1 The artery may bifurcate high in the arm, even at axillary level, into a main trunk (which continues into the forearm as the common interosseous artery) and a common stem, termed the superficial brachial artery, which divides at a variable level into its radial and ulnar branches (1% of cases). 2 A superficial radial artery may be given off in the upper arm; an anomaly of little practical importance because this vessel continues its course in a manner identical with a normal radial artery (14% of cases). 3 A superficial ulnar artery is given off in the upper arm in 2% of cases and descends nearly always superficially to the common origin of the forearm flexors. It then sweeps downwards to the lateral border of flexor carpi ulnaris to take up, in the distal forearm, the relationships of a normal ulnar artery. This superficial ulnar artery may lie beneath the deep fascia throughout its course or may lie subcutaneously, either at the elbow or in the upper forearm. It is the anomalous ulnar artery in this superficial and subcutaneous position, immediately beneath the median cubital vein and without the protection of the bicipital aponeurosis, which is at greatest risk of accidental puncture when the antecubital vein is selected for an intravenous injection. Fortunately, the superficial ulnar artery (unlike the normal ulnar artery) usually does not give rise to the common interosseous artery; in such cases the latter is derived from the radial artery. An inadvertent intra-arterial injection into the superficial ulnar artery will therefore usually spare this common interosseous branch and tend to preserve undamaged the main source of arterial blood supply to the forearm muscles.

48

Explain Integrated cardiovascular responses to pregnancy

Cardiac output Stroke volume Heart rate SVR PVR BP

Venous pressure Colloid oncotic pressure

Change +40% +30% +20% -20% -35% Unaffected or may be mid pregnancy drop of diastolic pressure by 5 10 mmHg +100% -15%

Cardiac output starts to increase from 5th week of preganancy, reaches its peak at about 30-34 week. Thereafter CO remains stable till term

49

Cardiac output is lowest in the sitting or supine position and highest in the right or left lateral or knee chest position. Cardiac output further increases by (+50%) during pregnancy and immediately after delivery (+70%) from the pre-pregnancy values There is squeezing out of blood from the uterus into the maternal circulation during labor and immediate post partum CO returns to pre-labour values by one hour following delivery and to the pre-pregnancy levels by another 4 weeks time. Increases in CO are caused by: 1. Increased blood volume 2. Increased metabolic activity Venous pressure: Antecubital venous pressure remains unaffected Femoral venous pressure is markedly raised specially in the later months, due to pressure of gravid uterus on common iliac viens more on the right due to dextro rotation of uterus more incidence of development of edema, varicose veins, piles, supine hypotension syndrome & DVT. Central hemodynamics: No significant change in the CVP, MAP and PCWP because of significant fall in SVR, PVR and colloid osmotic pressure

Explain the integrated cardiovascular responses to central neural blockade

Cardiovascular effects similar to the combined effects of the IV alpha and beta adrenergic blockers decreased heart rate and arterial blood pressure Sympathectomy extends two to six dermatomes above the sensory level with spinal anaesthesia and at the same level with epidural Vasomotor tone is primarily determined by sympathetic fibers arising from T5 to L1, innervating arterial and venous smooth muscle. Both arterial and venodilatation occurs, however venodilatation effects pre-dominate because: o 75% of blood volume is in venous capacitance system o Smooth muscle in arterial system helps maintain vascular tone Decreases in preload may initiate reflexes that cause severe bradycardia. Three such reflexes have been suggested: o Pacemaker stretch. The rate of firing of these cells within the myocardium is proportional to the degree of stretch. Decreased venous return results in decreased stretch and a slower heart rate. o firing of low-pressure baroreceptors in the right atrium and vena cava.

50 Bezold-Jarisch reflex: receptors in the left ventricle are stimulated by a decrease in ventricular volume and cause bradycardia. This reflex slowing should allow time for more complete filling of the heart. After neuraxial block induced sympathectomy, if normal CO is maintained, the TPR decreases by 15-18% in normovolemic subjects In elderly pts, TPR may fall by 25%, CO may decrease by 10% Heart rate may also decrease due to decreased right atrial stretch Effects will be more pronounced in: o Elderly (physiological reserve) o Fixed cardiac output states risk of critical cardiac event o blood vol (hypovolaemic) Level of block o Brainstem block inhibition of vasomotor centre unable to activate SNS response profound MAP life threatening o High block T1-T4 cardio-acceleratory centre blockade unable to HR/contractility with SNS stimulation profound MAP o Mid-thoracic / Renal level block GFR activation of RAA system by afferent arteriolar stretch o Sacral block nil sympathetic chain blockade (only parasympathetic fibres) minimal effect on peripheral vascular tone o

To describe the features of the coronary circulation and to explain the clinical significance of these
Anatomic Considerations The two coronary arteries that supply the myocardium arise from the sinuses behind two of the cusps of the aortic valve at the root of the aorta.

51 The right coronary artery has a greater flow in 50% of individuals, the left has a greater flow in 20%, and the flow is equal in 30%. Most of the venous blood returns to the heart through the coronary sinus and anterior cardiac veins, which drain into the right atrium.

Pressure Gradients & Flow in the Coronary Vessels Basal coronary blood flow 200-250ml/min

Coronary blood flow variation with the cardiac cycle: o LV: The pressure inside the left ventricle is slightly higher than in the aorta during systole. Flow predominantly during diastole. Subendocardial flow ceases during systole. o RV: flow during both systole and diastole, as RV pressures are low during both systole and diastole.

Blood flow in left coronary

Significant flow only in diastole

Blood flow in right coronary 0 Systo Clinical significance of above variation: le

Significant flow both in systole & diastole

Diastol e

52 Since diastole is shorter when the heart rate is high, left ventricular coronary flow is reduced during tachycardia. Because there is no blood flow during systole in the subendocardial portion of the left ventricle, this region is prone to ischemic damage and is the most common site of myocardial infarction. Blood flow to the left ventricle is decreased in patients with stenotic aortic valves because the pressure in the left ventricle must be much higher than that in the aorta to eject the blood. Also there is increased myocardial work. Coronary flow is also decreased when the aortic diastolic pressure is low. The rise in venous pressure in conditions such as congestive heart failure reduces coronary flow because it decreases effective coronary perfusion pressure.

Variations in Coronary Flow At rest, the heart extracts 70-80% of the O2 from each unit of blood delivered to it (coronary sinus po2 is 20mmhg). O2 consumption can be increased significantly only by increasing blood flow. The caliber of the coronary vessels, and consequently the rate of coronary blood flow, is influenced not only by pressure changes in the aorta but also by chemical and neural factors. The coronary circulation shows considerable autoregulation. Chemical Factors Factors suspected of playing this role include O2 lack and increased local concentrations of CO2, H+, K+, lactate, prostaglandins, adenine nucleotides, and adenosine. hypoxia increase coronary blood flow 200-300% in denervated as well as intact hearts. A similar increase in flow is produced in the area supplied by a coronary artery if the artery is occluded and then released. There is evidence that in the heart it is due to release of adenosine. Adenosine also known to decrease reperfusion induced injury. Neural Factors -adrenergic receptors mediate vasoconstriction -adrenergic receptorsmediate vasodilation. Activity in the noradrenergic nerves to the heart and injections of norepinephrine cause coronary vasodilation. However, norepinephrine increases the heart rate and the force of cardiac contraction, and the vasodilation is due to production of vasodilator metabolites in the myocardium secondary to the increase in its activity. When the inotropic and chronotropic effects of noradrenergic discharge are blocked by a -adrenergic blocking drug, stimulation of the noradrenergic

53 nerves or injection of norepinephrine in unanesthetized animals elicits coronary vasoconstriction. Thus, the direct effect of nor-adrenergic stimulation is constriction rather than dilation of the coronary vessels. Stimulation of vagal fibers to the heart dilates the coronaries. When the systemic blood pressure falls, the overall effect of the reflex increase in noradrenergic discharge is increased coronary blood flow secondary to the metabolic changes in the myocardium at a time when the cutaneous, renal, and splanchnic vessels are constricted. In this way the circulation of the heart, like that of the brain, is preserved when flow to other organs is compromised.

describe the cerebral circulation, autoregulation in the cerebral circulation and the factors that may affect it

(answer starts next page)

54

CBF = 750ml/min Cerebral metabolic rate of oxygen consumption = 50ml/min (i.e. 20% of basal o2 consumption) CBF = CPP/CVR Hence CBF depends upon CPP and CVR In spite of the marked local fluctuations in brain blood flow with neural activity, the cerebral circulation is regulated in such a way that total blood flow remains relatively constant. The factors involved in regulating the flow are summarized

55

Factors affecting CVR: Metabolic autoregulation Pressure autoregulation Chemical factors (PO2 PCO2) Nervous system

Metabolic autoregulation: Extremely important for moment to moment regulation of regional blood flow to meet metabolic demands Because of release of local vasoactive substances Exaples: o o Hig h Cerebr al blood flow Lo w Decrease in CBF after thiopentone induced decrease in metabolism Increase in CBF during seizures

Flow metabolism coupling

Lo w Level of cerebral metabolic activity (global or regional)

Hig h

Effect of changes in cerebral metabolic activity on cerebral blood flow

56

Pressure autoregulation: Ability to mentain a constant CBF despite changes In the MAP from 50 150mmhg Mechnaism-myogenic theory: arteriolar pressure smooth muscle stretch resting tone radius resistance decreases flow to normal Causes of right shift of pressure autoregulation curve: o o Chronic hypertension Acute sympathetic stimulation

Right shift causes intolerance to large drop in blood pressure which leads to decrease in the CBF In neonates the curve is shifted to left and plateau is narrower

Cerebr 200 al blood 15 flow 0 (as % 10 of 0 normal ) 50

50

10 0

15 0

200

Mean arterial blood pressure (mmHg) Effect of changes in mean BP on cerebral blood flow

57 Effect of pCO2 The CBF increases linearly as the CO2 rises from 20 to 80mmhg. The curve then flattens out at extremes. The normal CO2 is at the midpoint of the steep part of curve and hence CBF is very sensitive to the pCO2 The curve flattens out at low co2 because the vasoconstriction is counterbalanced by the vasodilatation by the concomitant hypoxia CBF change is 4% per mmHg pCO2 change Effect is shirt lived i.e. 4-6hrs as the bicarbonate decreases in the brain ISF returning pH to normal and so does the CBF. Now if the CO2 is increased to its normal level there is excessive rise in CBF.

Cerebr 200 al blood 15 flow 0 (as % 10 of 0 normal ) 50

20

40

60 Arterial pCo2

80

100

Effect of changes in arterial pCo2 on cerebral blood flow Cerebr Effect 200 al blood 15 flow 0 (as % 10 of 0 normal ) 50 of PO2 There is no change in the CBF until the pO2 decrreases to 50% below which there is increase in the CBF. There is no change in the CBF until PO2 of 50 because as per the ODC the oxygen content does not decrease significantly unless the po2 decreaes below the 50-60 mmHg mark (the steep part of ODC)

50

10 0

15 0

200

Arterial pO2 (mmHg) Effect of changes in arterial PO2 on cerebral blood flow

58

Role of nervous system: Changes in the ANS can at best alter CVR a small amount (5-10%) However the effects are via effects on CPP. Drop in BP initiates carotid sinus refles decrease neuronal firinf increased sympathetic outflow increased BP CPP is mentained along with the intrinsic pressure auto-regulation Cushings reflex: under conditions of extreme hypoxia maximal sympathetic stimulation occurs through out inorder to increase CP and retore CPP. Last ditch maneuver prognosis poor

CPP: Mean arterial pressure minua the greater of the cerebral venous pressure or the ICP. Hence to determine the CPP requires the measurement of all the three factors. This is because like heart and lung, brain is also a starling resistor Monro-kellie doctrine: Bony skull is rigid and volume is constant. Fluid is incompressible. Therefore any increase In the intracranial content will result in large increase in pressure Intracranial content: brain 85%, CSF 10% and blood 5%. Initially the pressure does not rise abruptly as the CSF is translocated out. But once this buffer mechanism iks exhausted the pressure rises abruptly Maneuvers decreasing ICP: Head tilt up Ensuring no obstruction to venous drainage Hyperventilation Mannitol Controlled hypotension Hypertonic urea solutions

59

Describe the renal circulation and to explain its significance in maintaining renal function
Anatomy The afferent arterioles are short, straight branches of the interlobular arteries. Each divides into multiple capillary branches to form the tuft of vessels in the glomerulus. The capillaries coalesce to form the efferent arteriole, which in turn breaks up into capillaries that supply the tubules (peritubular capillaries) before draining into the interlobular veins. The arterial segments between glomeruli and tubules are thus technically a portal system, and the glomerular capillaries are the only capillaries in the body that drain into arterioles. However, there is relatively little smooth muscle in the efferent arterioles. The capillaries draining the tubules of the cortical nephrons form a peritubular network, whereas the efferent arterioles from the juxtamedullary glomeruli drain not only into a peritubular network but also into vessels that form hairpin loops (the vasa recta) that dip into the medullary pyramids. The descending vasa recta have a nonfenestrated endothelium that contains a facilitated transporter for urea, and the ascending vasa recta have a fenestrated endothelium, consistent with their function in conserving solute. Regional Blood Flow & Oxygen Consumption Renal blood flow: 1200-1300 ml/min ( 25% of CO) 95% of flow goes to cortex and 5% to medulla. Renal oxygen consumption = 18ml/min (7% of total body oxygen consumption) Main function of the renal cortex is glomerular filtration, hence cortical blood flow is high (5 mL/gm/min) despite of low oxygen extraction. The PO2 of the cortex is about 50 mm Hg. Main function of medulla is maintenance of the osmotic gradient, hence has relatively low blood flow (2.5 mL/g/min in the outer medulla and 0.6 mL/g/min in the inner medulla). However the metabolic work is high particularly to reabsorb Na+ in the thick ascending limb of henle. Therefore the oxygen extraction is high and PO2 of the medulla is about 15 mm Hg. This makes the medulla vulnerable to hypoxia if flow is reduced further. NO, prostaglandins, and many cardiovascular peptides in this region function in a paracrine fashion to maintain the balance between low blood flow and metabolic needs.

Measurement of renal blood flow: Renal plasma flow measured as the clearance of PAH. Renal blood flow can be calculated of the hematocrit is known. RBF=RPF(1/1-hct)

60 Ficks principle: the uptale of the substance by the tissue must be equal to the difference between the amount entering the tissue (=flow*arterial concentration) and the amount leaving the tissue (=flow*venous conc. ) i.e. Flow= uptake/(arterio-venous conc. Difference) For the kidney Renal plasma flow = PAH uptake/ (renal arterial PAH-renal vien PAH) PAH is almost entirely cleared hence renal vien PAH = 0 and renal arterial PAH=systemic arterial PAH=systemic venous PAH Hence, Effective Renal plasma flow = urinary PAH*volume of urine/peripheral venous PAH Not all of PAH is removed from the plasma, hence a correction factor is used to convert effective renal plasma flow to renal plasma flow. Correction factor is 0.9. RPF=ERPF/0.9 Renal blood flow auto regulation: Auto-regulation occurs between pressures 75mmHg and 160mmHg Major mechanisms for auto regulation: Myogenic mechanism in the glomerular arterioles. NO may also be involved. Tubuloglomerular feedback important in maintaining a constant GFR. Too much fluid and electrolyte reaching early distal tubule is sensed by the macula densa and a feedback signal results in the vasoconstriction of afferent glomerular arteriole and vice-versa. The signal sensed by the macula densa cells is probably change in Na+ flux across their membranes. At low perfusion pressures, angiotensin II also appears to play a role by constricting the efferent arterioles, thus maintaining the glomerular filtration rate. This is believed to be the explanation of the renal failure that sometimes develops in patients with poor renal perfusion who are treated with drugs which inhibit angiotensin-converting enzyme.

Effect of humoral factors on rbf Norepinephrine constricts the renal vessels, with the greatest effect of injected norepinephrine being exerted on the interlobular arteries and the afferent arterioles.

61 Dopamine is made in the kidney and causes renal vasodilation and natriuresis. Angiotensin II exerts a greater constrictor effect on the efferent arterioles than on the afferent. Prostaglandins increase blood flow in the renal cortex and decrease blood flow in the renal medulla. Acetylcholine also produces renal vasodilation. A high-protein diet raises glomerular capillary pressure and increases renal blood flow.

Effect of ANS on rbf Strong stimulation of the sympathetic noradrenergic nerves to the kidneys causes a marked decrease in renal blood flow. This effect is mediated by 1adrenergic receptors and to a lesser extent by postsynaptic 2-adrenergic receptors. When systemic blood pressure falls, the vasoconstrictor response produced by decreased discharge in the baroreceptor nerves includes renal vasoconstriction. Renal blood flow is decreased during exercise and, to a lesser extent, on rising from the supine position.

describe the hepatic and splanchnic circulation

The blood from the intestines, pancreas, and spleen drains via the hepatic portal vein to the liver and from the liver via the hepatic veins to the inferior vena cava. The viscera and the liver receive about 30% of the cardiac output via the celiac, superior mesenteric, and inferior mesenteric arteries.

62 The liver receives about 1000 mL/min from the portal vein and 500 mL/min from the hepatic artery.

Intestinal Circulation The intestines are supplied by a series of parallel circulations via the branches of the superior and inferior mesenteric arteries. There are extensive anastomoses between these vessels. The blood flow to the mucosa is greater than that to the rest of the intestinal wall, and it responds to changes in metabolic activity. Thus, blood flow to the small intestine (and hence blood flow in the portal vein) doubles after a meal, and the increase lasts up to 3 hours. Hepatic Circulation Derived from both the hepatic artery and portal vien. Total liver blood flow is approx 1.5L/min or 25% CO Hepatic artery: 1. High pressure, high resistance system 2. Delivers 30% of total hepatic blood flow 3. Contributes 50% of total hepatic oxygen supply 4. Owing to high resistance in hepatic arterioles the pressure drops to 35 mmHg and the ratio of pre-sinusoidal to postsinusoidal resistance results in low hepatic sinusoidal pressure of 2mmHg 5. Average flow velocity = 18cm/s

Portal vien

63 1. Low pressure (5-10mmHg), low resistance and low velocity system 2. Delivers 70% of total hepatic blood flow 3. Contributes 50% of total hepatic oxygen supply 4. Portal vien is a valve less viemn draining blood from intestines, stomach, spllen, pancreas and GB to liver. 5. In fasting state the oxygen saturation of portal venous blood is approx 85% which decreases with increased gut activity 6. Average flow velocity = 18cm/s

Hepatic viens: 1. Venous blood from the liver returns into the IVC via right and left hepatic viens 2. Various stimuli may cause hepatic venoconstriction and these include norepinephrine, angiotensin, hepatic nerve stimulation and histamine 3. Hepatic venous pressure is also influenced by external factors: IPPV, intraabdominal pressure, gravity and gut wall activity

Hepatic microvasculature 1. Hepatic triad: Small vessels of portal vien + hepatic artery + biliary canaliculli 2. Vessels of triad anastomose to form sinusoids which drain from the periphery of acinus into hepatic veins 3. Blood flows from the center of this functional unit to the terminal branches of the hepatic veins at the periphery. This is why the central portion of the acinus, sometimes called zone 1, is well oxygenated, the intermediate zone (zone 2) is moderately well oxygenated, and the peripheral zone (zone 3) is least well oxygenated and most susceptible to anoxic injury. Capacitance function 1. Liver has about 450ml of blood, 50% of which can be mobilized in response to hypovolemia 2. Portal blood can bypass the sinusoids as blood is shunted from portal venules to hepatic venules

64 3. Catecholamines mobilize blood from sinusoids 4. Liver can also buffer against an increase in blood volume. Hepatic compliance increases with increase

Regulation of hepatic blood flow Given the high blood flow the high o2 demand is met by increased extraction rather than increase in flow Intrinsic control: o When the hepatic arterial pressure is reduced, flow is mentained by decrease in arterial resistance until systolic pressure is below 80mmHg Portal venous system has no autoregulation Hepatic arterial buffer: Reduction in portal vien flow is associated with increase in hepatic artery flow between 22 and 100 %, however the decrease in hepatic artery blood flow does not change the portal venous flow significantly. This semi-reciprocal relationship is known as hepatic arterial buffer. Mediated by adenosine induced vasodilatation

o o

Extrinsic control: o Hepatic artery has alpha, beta and dopamine receptors but portal vien has only alpha and dopamine receptors. Epinephrine causes portal vien constriction and in artery intial constriction (alpha) followed by dilatation (beta). Glucagon, VIP and secretin increases hepatic blood flow Angiotensin II and vasopressin decreases blood flow Feeding dramatically increases hepatic blood flow During normal spontaneous breathing, hepatic venous outflow decreases during inspiration and increases during expiration Vigourous exercise causes splanchnic vasoconstriction and reduced hepatic blood flow. PPV decreases HBF Hypocapnia can reduce HBF by 30% mainly due to reduction in portal venous blood flow and vice versa Spinal and epidural anaesthesia reduce total hepatic blood flow.

o o o o

o o

65 o Inhalational agents generall reduce hepatic blood flow, halothane causes greatest reduction Intravenous agents such as thiopentone decreases HBF in dose dependent manner

Measurment of hepatic blood flow: Direct method: electromagnetic flowmeters around vessels at laparotomy Indirect method: o Clearance techniques: o Single bolus technique Continuous infusion technique

Reticuloendothelial cell uptake of radiolabelled colloid particles. Clearance used to estimate HBF Microspheres technique (only animal studies)

Clearance techniques: Indicator used: o o o Indocyanin green (ICG) Sulphobromophthalein Iodine labeled albumin

Single bolus technique o Single bolus of ICG (0.5mg/kg) is injected intravenously and venous samples are collected every 2 min for 14min Clearance = dose/area under conc. Vs time curve Extraction ratio of ICG is 0.74. HBF = clearance/extraction ratio

o o o

Continious infusion technique o After loading dose, infusion of ICG is administered for 20 min to achieve equilibrium Samples are taken simultaneously from artery and hepatic vien HBF = clearance/ extraction ratio

o o

66 o o Clearance = infusion rate/art. Conc. ICG Extraction ratio = (arterio-venous conc difference of ICG)/art. Conc. ICG

Reservoir Function of the Splanchnic Circulation 25-30% of the volume of the liver is accounted for by blood. Contraction of the capacitance vessels in the viscera can pump a liter of blood into the arterial circulation in less than a minute. Other blood reservoirs that contain a large volume of blood at rest are the skin and lungs. During vigorous exercise, constriction of the vessels in these organs and decreased blood "storage" in the liver and other portions of the splanchnic bed, the skin, and the lungs may increase the volume of actively circulating blood perfusing the muscles by as much as 30%.

Describe utero-placental circulation


Concerns with the circulation of the maternal blood through the intervillous space. Mature placenta has a volume of 500ml of blood; 350ml being occupied by the villi system and 150ml lying in the intervillous space. Intervillous blood flow at term is 500-600ml/min. villi depend on the maternal blood for nutrition. The pressure within the intervillous space is 10-15mmhg during uterine relaxation and 30-50mmHg during uterine contraction. In contrast the fetal capillary pressure in the villi is 20-40mmHg.

67 circulation: 120-200 spiral arteries open in the intervillous space by piercing the basal plate randomly at numerous sites. There is cytotrophoblastic invasion of the spiral arteries upto the intra-decidual portion within 12 weeks of pregnancy. Secondary invasion occurs between 12-16 weeks extending upto the radial arteries within the myometrium. Spiral arteries are converted to large bore uteroplacental arteries. The blood in the intervillous space is protected from clotting by some fibrinolytic enzyme activity of the trophoblast.

Describe the responses to changes in posture


CVS response to standing from supine position: When a person stands a significant volume of blood pools in the lower extremities because of the high compliance of the veins. (muscular activity would prevent this pooling) As a result of venous pooling and increased local venous pooling, Pc in the legs increases and fluid is filtered in the interstitium. If net filtration of fluid exceeds the ability of the lymphatics to return it to the circulation, edema will occur Blood volume and venous return decreases stroke volume and CO decrease Initially arterial pressure decreases because of reduction in CO. if CPP becomes low enough fainting may occur Compensatory mechanisms: carotid sinus baroreceptors respond to decrease in BP by decreasing the firing rate of carotid sinus nervesincreased sympathetic outflow from the vasomotor centre to heart and blood vessel HR and TPR increase normalize BP

Describe the cardiovascular changes seen in haemorrhage and hypovolaemia

68

Hemorrhage decreases the mean systemic filling pressures of the circulation and consequently reduces venous return such that CO falls. The physiological effects of haemmorhage depend on the (i) rate of blood loss and (ii) degree of blood loss. Hypovolaemia CO CO CVP CVP

MAP MAP Arterial baroArterial baroreceptors receptors Cardiopulmonary Cardiopulmonary mechanoreceptors mechanoreceptors chemoreceptors chemoreceptors

Vasomotor centre

Parasympathe Parasympathe tic output tic output reduced reduced

Sympathetic Sympathetic output output increased increased

HR

Myocardial contractility

venous constriction

Arteriolar constriction

capillary hydrostatic pressure Fluid absorption

CO

TPR

Arterial BP When the blood loss is <10% the pulse pressure is reduced, mean restored? pressure is same, because of increased in HR and high SVR. Blood flow to

69 the tissues that are highly innervated by the sympathetic nervous system will be reduced. Reduced venous compliance. Renal, coronary and cerebral blood flows are auto-regulated.

When the blood loss > 20%, the compensatory mechanisms become inadequate and the CO and BP start to decrease. When the blood pressure decreases below 50mmHg CNS ischemic response is elicited causing a powerful sympathetic stimulation. Irreversible hypotension can occur with a blood loss greater than 30% of blood volume. Inadequate tissue perfusion leads to increased anaerobic glycolysis with the production of large amounts of lactic acid lactic acidosis depresses the myocardium and reduces catecholamine responsiveness in peripheral vessels.

Loss of blood volume causes the venous return curve to be shifted to the left because of fall in systemic filling pressure, but this is partially restored by the increased sympathetic activity which also shifts the CO curve upwards and consequently a new equilibrium point is established.

70

Hormonal response: decreased venous return decreased right atrium stretch decreased ANF release and stimulation of ADH. Renal vasoconstriction renin release renin-angiotensin system Increased aldosterone from adrenal cortex. ADH and aldosterone mediate Na and water reabsorption Angiotensin and vasopressin system take between 1min to 1 hr to respond completely Increased sympathetic activity causes release of cortisol and catecholamines from adrenal gland

71

Long-Term Compensatory Reactions After a moderate hemorrhage, the circulating plasma volume is restored in 12-72 hours. There is also a rapid entry of preformed albumin from extravascular stores, but most of the tissue fluids that are mobilized are protein-free. They dilute the plasma proteins and cells, but when whole blood is lost, the hematocrit may not fall for several hours after the onset of bleeding. After the initial influx of preformed albumin, the rest of the plasma protein losses are replaced, presumably by hepatic synthesis, over a period of 3-4 days. Erythropoietin appears in the circulation, and the reticulocyte count increases, reaching a peak in 10 days. The red cell mass is restored to normal in 4-8 weeks. However, a low hematocrit is remarkably well tolerated because of various compensatory mechanisms. One of these is an increase in the concentration of 2,3-DPG in the red blood cells, which causes hemoglobin to give more O2 to the tissues. In longstanding anemia in otherwise healthy individuals, exertional dyspnea is not observed until the hemoglobin concentration is about 7.5 g/dL. Weakness becomes appreciable at about 6 g/dL; dyspnea at rest appears at about 3 g/dL; and the heart fails when the hemoglobin level falls to 2 g/dL.

Explain the cardiovascular effects and responses in different forms of shock

Shock is multifactorial syndrome resulting in inadequate tissue perfusion and cellular oxygenation. In 1972 Hinshaw and Cox suggested the following classification: hypovolemic, cardiogenic, distributive and obstructive shock. (CHOD) Type of shock Hypovolemic Pathophysiology Absolute loss of circulating volumedecreased venous return decreased venous filling pressure decreased CO decreased BP decreased perfusion Most common Eg hemorrhagic Decreased stroke volume due to systolic or diastolic failuredecreased CO decreased BP decreased perfusion Eg. Myocardial infarction, cardiomyopathy, arrhythmias, contusio cordis, or cardiac valve problems "relative" hypovolaemia due to dilation of blood vessels which diminishes systemic vascular resistance decreased venous return decreased venous filling pressure

Cardiogenic

Distributive

72 decreased CO decreased BP decreased perfusion Septic shock: Caused by an overwhelming systemic infection resulting in vasodilation Anaphylactic shock: Caused by a severe anaphylactic reaction to an allergen, antigen, drug or foreign protein causing the release of histamine which causes widespread vasodilation, leading to hypotension and increased capillary permeability. Neurogenic shock: trauma to the spinal cord resulting in the sudden loss of autonomic and motor reflexes below the injury level decrease in peripheral vascular resistance, leading to vasodilation and hypotension. Mechanical obstruction to blood flow resulting in either failure of ventricular filling or emptying decreased stroke volume decreased CO decreased BP decreased perfusion. Eg. Cardiac tamponade, Constrictive pericarditis, Tension pneumothorax, Massive pulmonary embolism, Aortic stenosis.

Obstructive

PCWP Hypovolem Low ic Cardiogeni c High

CVP Low

CO Low

SVR High

High

Low

High

Inflammato Low / N ry Neurogenic Low Obstructiv e

Low/N

High

Low

Low

Low low

Low high

Low/high high

To explain the cardiovascular responses accompanying pregnancy, birth, ageing, cardiac failure, and during intermittent positive pressure ventilation, positive end-expiratory pressure, and the Valsalva manoeuvre.
Valsalava manoeuvre Forced expiration against a closed airway is termed valsalva manoeuvre. This results in a rise in the intrathoracic, intra-abdominal and CSF

73 pressure. The CVP rises by about 7mmhg for a 10mmHg rise in mouth pressure. Clinically this can be performed by a person blowing into mercury column to produce a pressure of 40mmhg and holding it for 10-15sec. Normal cardiovascular changes associated with the valsalva manoeuvre may be divided into 4 phases: o Phase I: at the onset of manoeuvre there is a transient small rise in blood pressure with a brief fall in HR. this Is due to the transmission of increased intra-thoracic pressure onto the aorta Phase II: raised intra-thoracic pressure causes a decrease in venous return to the right heart that reduces CO and causes a fall in BP stimulates baroreceptors and compensatory mechanisms. Sympathetic stimulation increases HR and causes vasoconstriction. These changes restore BP o Phase III: immediately after release of positive airway pressure, there is transient fall in blood pressure with a further rise in the HR. this is brought about by the loss of the transmitted raised intrathoracic pressure on the aorta Phase IV: with the intrathoracic pressure returning to baseline, venous return is restored and a normal cardiac output results. The delivery of normal CO into constricted peripheral vascular bed causes overshoot of blood pressure. This rise in BP is sensed by the baro-receptors resulting in reflex bradycardia by vagal action.

Abnormal responses: o Patients with diminished baro-receptor reflexes eg. Quadriplegia and diabetic autonomic neuropathy excessive fall in BP in phase II and an absence of overshoot and bradycardia in phase IV o In CHF: square wave response: BP elevated throughout phase II and there is no overshoot in phase IV and little change in HR.

Valsalva maneuver may be used to assess ANS (using valsalva ratio ratio of minimum HR and maximum HR in phase 4) and also to slow SVT.

74

describe the factors that affect mixed venous oxygen saturation


Sampling of mixed venous blood: True mixed venous blood must be obtained via slow aspiration from a pulmonary artery catheter where blood from SVC, IVC and coronary sinus have fully mixed. Thus, mixed venous pO2 and pCO2 reflect O2 extraction and CO2 addition from the entire body. In the presence of central (ASD,VSD etc) or peripheral (AV fistula) shunt, it may not be possible to obtain a sample of true mixed venous blood. Typical values for a mixed venous blood gas on a person breathing room air are: o o o o PvO2 of 40mmHg PvC02 of 45mmHg Sa02 of 75% Venous 02 content of 15mls/dL for normal ranges of Hb in the blood.

Oxygen content = 1.34*Hb*SO2 + 0.003*PO2 Relationship between PO2 and O2 content in mixed venous blood:

75 o P02 relationship with O2 content dependent on shape of Hb02 dissociation curve. Right shifted Hb02 decreases Hbs affinity for 02 and increases the amount dissolved i.e. P02 factors which right shift are decrease in pH, increase in C02, and increase in temperature and increase in 2,3 DPG levels.

Factors affecting mixed venous oxygen saturation o Modification of the FICK equation: VO2 = (CaO2 - CvO2)*Q gives Cv02 = Ca02 V02/Q This equation demonstrates the inverse relationship between oxygen extraction by tissues and the cardiac output. If cardiac output decreases, oxygen extraction by the tissues increases, causing mixed venous 02 concentration to fall. This fall in concentration causes a decrease in mixed venous Pv02. Cv02 is directly proportional to Ca02. However, clinically, this factor is not as important as the level of cardiac output. This is because relatively large increases in partial pressure of oxygen (eg hyperbaric) are required to substantially increase the 02 content of arterial blood above 20ml/dl as Hb is near fully saturated at a Pa02 of 100mmHg and any additional increase in partial pressure contributes a relatively miniscule 0.003mls of 02/mmHg/ml of blood.

o o

Outline the physics of blood flow

Poiseuilles equation:

Developed by poiseuille based on study of laminar flow of Newtonian (homogenous) fluid in glass capillary tubes. Conceptually can be applied to blood flow. The equation states that Flow=(Pi-Po)r4/8L Where Pi-Po is the hydrostatic pressure gradient acting along the length of the vessel, is the blood viscosity, L is the vessel length, and r is the vessel radius

Viscosity

76

Blood is a non-newtonian fluid and the measured viscosity varies with the flow. Higher the viscosity slower the flow and vice versa. Blood has a relative viscosity of 4-5 when measured at moderate or high shear rates

Hydraulic resistance

Hydraulic resistance = pressure drop/flow Substituting poisseuille equation Resistance = 8L/ r4 Where is blood viscosity, L is the length of vessel and r is the vessel radius The radius is very important as resistance is inversely proportional to the fourth power of radius

Resistance of vessels in series and parallel

Arteries, capillaries and veins lie in series but many individual organ circulations lie in parallel In series R = R1+R2+R3+..+Rn In parallel resistances 1/R = 1/R1+1/R2+1/R3+..+1/Rn In other words when vessels lie in parallel the combined total hydraulic resistance is less than any one of the individual vessel resistance Capillaries are of smaller diameter than arterioles yet the main site of resistance is the arterioles because the no. of capillaries in parallel is so huge that the combined hydraulic resistance is much lower than arterioles.

Physiological deviations from poiseuilles equation Blood vessels are not rigid tubes. The pressure flow relationships of different blood vessels vary considerably but may be considered as being of three types as shown in figure

77

Pulsatile blood flow: one function of the large elastic arteries is to convert the intermittent ventricular output to more continuous pulsatile flow. Because of this flow in the capillaries is relatively steady. Anomalous viscosity of blood: blood viscosity changes with temperature, hematocrit, vessel diameter and flow rate. Blood viscosity is also lower when measured in-vivo than in-vitro, because of laminar flow patterns in small vessels which decreases viscosity. The viscosity of blood falls progressively when the vessel diameter is reduced below 300 m. Turbulent flow: turbulence tends to develop in large diameter vessels with high flow velocity, low fluid viscosity and high fluid density. The Reynolds number can be used to predict the presence of turbulent flow in a long straight tube: Reynolds number (Re)=Dv/ where is fluid density, D is the vessel diameter, v is the mean velocity and is the fluid viscosity. Flow is usually turbulent is Re > 2000 and laminar if Re<1000

Pressure and flow Total fluid energy determines liquid movement between two points in a tube. Total fluid energy comprises three factors: potential energy (pressure energy), kinetic energy and gravitational potential energy. For fluid moving horizontally, gravitational potential energy can be ignored. Potential energy is measured in terms of pressure applied on the lateral walls. The total fluid energy always remains constant, hence

78

when kinetic energy increases the potential energy decreases or the pressure on the lateral wall decreases.
describe the various methods of measuring blood pressure

Intermittent, non-invasive systems require three key components: 1. an inflatable cuff for occluding the arterial supply to the distal limb; 2. a method for determining the point of systolic and diastolic blood pressures; 3. a method for measuring pressure. A cuff is placed circumferentially around the limb. Most commonly the upper arm. The cuff should be 20% wider than the diameter of the part of the limb being used (or cover two-third its length). Cuffs that are too small will lead to overestimation of blood pressure and vice versa. The cuff is inflated to a pressure above that of the arterial systolic pressure. At this point, the walls of the artery are opposed preventing blood flow. The cuff is then deflated below systolic pressure allowing blood flow to resume; this flow can then be detected using various means. Palpation While being easy to perform, this technique has been shown to underestimate a systolic pressure of 120 mm Hg by 25%. Diastolic and mean pressures cannot be determined. Doppler Systolic pressure can also be determined using the Doppler principle. As Doppler is so sensitive, this technique is usually reserved for the measurement of low pressures, e.g. vascular insufficiency. Auscultation Auscultation over the brachial artery while using a cuff.5 phases are observed. The cause of the sounds is uncertain. fifth sound represents diastolic pressure, unless the pulsation continues to be audible on complete deflation of the cuff when the fourth sound should be used. Simple to perform. Operator variability can be significant.

79

Oscillotonometry The Von Recklinghausen Oscillotonometer uses two cuffs and two bellows connected to a measurement gauge. The two cuffs overlap, one occludes the artery (occluding cuff) and the other senses the arterial signal (sensing cuff). Pressure from both cuffs is transmitted to the two bellows which is in turn displayed via a single gauge, alternating between the two bellows using a lever. With the lever in the sensing position, the occlusive cuff is inflated above systolic pressure. The cuff is then deflated using a bleed valve until the needle suddenly starts to move vigorously. The lever is then switched to measure the occluding cuff pressure. This is the systolic blood pressure. With the lever back in the 'sensing cuff' position, the occluding cuff is deflated further. The needle will jump further with maximal oscillations occurring at mean arterial pressure (MAP), as measured by moving the lever once more. Diastolic pressure is the point at which these oscillations reduce. The most accurate measurements taken are those of systolic and mean blood pressures with diastolic measurements being more susceptible to operator variability. However, diastolic pressure can be related to systolic pressure and MAP using the following formula:

80

Liquid Manometers The air in the cuff acts on a liquid forcing it up a manometer. Mercury is used as it has 13.6 times the density of water. (7.5 mm Hg = 10 cm H2O i.e 3/4th ). A sphygmomanometer uses an open manometer and measures gauge pressure, e.g. pressure over atmospheric pressure. Unlike water, the meniscus created by a level of mercury is convex upwards. Measurement is taken from the top of the meniscus. Closed manometers are used in mercury barometers and measure absolute pressure. A meniscus forms below a Torricellian vacuum. (When the height of a mercury column is above atmospheric pressure, i.e. 760 mm Hg, a space forms above it. It is not a true vacuum as it has a pressure equal to that of the saturated vapour pressure of mercury and hence contains mercury vapour. This vacuum is termed a Torricellian vacuum). Blood pressure could be measured using a closed barometer but this would require a large column of mercury as the measurement would include atmospheric pressure, for example 760 mm Hg + 120 mm Hg = 880 mm Hg, in order for a meniscus to be created. Assuming gravity and the density of the mercury remain constant; the height of a column of mercury is only proportional to the force exerted upon that column. Therefore, the width and shape of the manometer has no bearing on the height of the column and therefore no bearing on the measurement. Aneroid Gauge An aneroid gauge commonly replaces the mercury column as it is more robust and avoids the problems associated with mercury toxicity. An increase in pressure expands a bellows, which then moves a pointer along a scale to indicate pressure. They are susceptible to loss of accuracy over time and hence require regular calibration. Electronic Systems A change in air pressure causes movement of a diaphragm. That movement is then detected and displayed electronically. This system is utilized by automated, non-invasive blood pressure measuring systems.

81

Continuous Non-Invasive Blood Pressure Measurement Uses the Pez principle which states 'a force exerted by a body can be determined by measuring an opposing force that prevents physical disruption'. A small cuff is placed around a finger. With each cardiac cycle, the volume of blood within the finger varies and with it, the amount of light absorbed. In order to keep the amount of light absorbed constant, the volume must also be kept constant and so pressure is applied to the finger. The applied pressure waveform correlates to the pressure waveform of the arterial supply to the finger. highly accurate in vasodilated patients and those with normal circulation, it is less accurate in hypotensive patients or those with vascular insufficiency.

Continuous, Invasive Blood Pressure Monitoring This is the gold standard of blood pressure measurement giving accurate beat-to-beat information. avoids the problem of repeated cuff inflation (causing localized tissue damage) and allows repetitive sampling for blood gases and laboratory analysis. The basic principle is to provide a solid column of liquid connecting arterial blood to a pressure transducer (hydraulic coupling) and requires the following components: 1. intra-arterial cannula; 2. tubing 3. transducer; 4. microprocessor and display screen; 5. mechanism for zeroing and calibration. Intra-Arterial Cannula. Teflon or polyurethane is inserted into an artery. Normally, a 20G cannula is used although 22G, and 25G are available for children and neonates. Preferably, a non-end artery, such as radial or dorsalis pedis is cannulated. The collateral supply to the hand can be assessed using Allen's test although this is not 100% reliable. Infusion and Tubing. The cannula is connected to a disposable tubing system, which delivers a constant infusion of plain or heparinized 0.9% saline, delivered at a rate of 2-4 mls/h to avoid thrombotic occlusion. The infusion fluid is kept pressurized to ensure a constant flow into the arterial

82

system. The tubing should be stiff and not contain any bubbles in order to minimize resonance and damping. Transducers. The liquid within the infusion tubing is in contact with a diaphragm that moves in response to the transmitted pressure waveform. The movement is converted to an electrical signal by a transducer. The transducer can do this by acting as part of a capacitor, inductor, or most commonly a strain gauge. Strain gauges use the principle that resistance of a wire increases with increasing length and visa versa. The diaphragm of the transducer moves a small plate that is connected to four strain gauges. With any one movement, two gauges are compressed and the other two stretched. The material used in the strain gauge is important as different materials have different sensitivity to stress and strain and whose resistances will differ according to temperature. By using four gauges, two of which are compressed and two stretched, the effect of a change of temperature (so long as this remains linear) is cancelled out. All four strain gauges form part of a Wheatstone bridge thus increasing the sensitivity four-fold. The transducer needs to be kept horizontally level with the patient; traditionally, the right atrium. A 10 cm change in height will alter the pressure reading by 7.5 mm Hg. Raising the hand will not affect the measurement so long as the transducer is maintained level with the heart, unlike non-invasive methods. Microprocessor and Display Screen. These provide a user-friendly numerical and graphical display allowing beat-to-beat measurement of pressure and also allow analysis of the waveform. Analysis can be clinical (e.g. morphology, determining the position of the dichrotic notch or 'swing' which can give information regarding filling status and cardiac output), or computerized. Calibration: Calibration can be considered in two parts: Static calibration (to ensure static accuracy) Dynamic calibration (to ensure dynamic accuracy) Static calibration involves: Zeroing the transducer Checking and adjusting the gain Checking for time stability. Dynamic calibration involves: Resonant frequency Damping coefficient

83

Zeroing Zeroing the transducer sets the zero reference point for the pressure measurement. The technique is to: Turn off the line to the patient by turning off a three way tap. Position the tap at the level which is to be used as the zero reference point. Open the tap in the transducer dome to air so that the transducer is exposed to atmospheric pressure. Press the 'Zero' button on monitor. (This electronically records the signal from the transducer as representing the zero value). Check that a value of OmmHg is shown. Close the tap on the dome. Open the three way tap so there is a continuous fluid column between the transducer and the arterial lumen. The zero reference point is not at the level of the transducer diaphragm but at the level of the top of the fluid column (air-fluid interface) above (or below) the diaphragm when the tap was open to air. Dynamic caliberation: Catheter-transducer systems can be characterised by three mechanical parameters: elasticity, mass and friction.These three basic parameters determine two parameters: Resonant frequency Damping coefficient The two parameters can be used to specify the Dynamic Response of clinical catheter transducer systems.

Resonance The arterial pressure waveform is made up of many different sine waves (as determined by Fourier Analysis) with each sine wave having a specific frequency. Every system has its own natural oscillatory frequency, or resonant frequency, and if the resonant frequency of the transduction system coincides with one of the frequencies making up the arterial waveform, resonance, and subsequent distortion of the signal will occur. The specific cannulae and infusion tubing used in invasive arterial pressure monitoring are designed to keep its natural frequency above 40 Hz, which is above any of the frequencies making up the arterial waveform, thus minimizing resonance. Damping Some damping is inherent in any system and acts to slow down the rate of change of signal between the patient and pressure transducer. This can

84

be caused by occlusion of the arterial system, a bubble interrupting the saline column, or using a soft cannula and tubing. Some damping is useful as it reduces the resonant frequency of the pressure transducing system. The amount of damping in a system is indicated by the 'damping factor'. 'Damping coefficient' The damping coefficient is a number which quantifies the amount of damping occurring in the system. If a system with elastic properties is disturbed, it tends to oscillate before settling at a new value. Damping is the property that tends to decrease the magnitude of the oscillations. The damping coefficient has a value which depends on how much the oscillations are reduced in magnitude with each oscillation. If the system was not damped at all, the damping coefficient would be zero and the oscillations would continue forever rising to the same height each time. The damping coefficient can be calculated from the ratio of the magnitudes of consecutive oscillations. lf these magnitudes are represented as Dt & D2, then the ratio is D2/Dl and theformulafor damping coefficient (D) is:

Though it is not difficult to perform this calculation on a scientific calculator it is much simpler and quicker to refer to a graph where the Ratio of Magnitudes (D2/D1) is plotted Against the Damping Coefficient. This formula is only valid if there is an overshoot and D2/Dl has meaning. So it cannot be used at D = I or higher as there is no overshoot.] The system can be:

Optimally damped: The system responds rapidly to a change in signal by allowing a small amount of overshoot (Damping factor 0.64). Advantages:
o o o

Amplitude Distortion is minimized The Frequency Response obtained is maximal Phase Distortion is minimised

85

Critically damped: No overshooting occurs but the system may be too slow (Damping factor 1.0). Under-damped: Resonance occurs causing the signal to oscillate and overshoot (Damping factor <0.64). Over-damped: This may be due to soft tubing, a bubble, or a constriction. The signal takes a long time to reach equilibrium but will not overshoot. It may not reach equilibrium in time for a true reading to be given (Damping factor >1.0).

the required resonant frequency is always one quarter of the pulse rate. So in more general terms, the required system is D = 0.64 (always) and Resonant frequency = (Pulse rate /4) Hz. These levels of dynamic accuracy is required only for accurate measurement of systolic and diastolic pressures. Physiologically, for nearly all the tissues in the body it is the mean arterial pressure which is most important for determining perfusion pressure. The mean pressure can be measured very accurately by invasive means as accuracy depends only on the static calibration and is independent of the dynamic calibration of the system. explain the various methods of measuring cardiac output as well as their limitations Cardiac output (CO) is the volume of blood being pumped by the heart per minute and is equal to the heart rate multiplied by the stroke volume. The normal value at rest for a 70 kg male is around 5 L/min. Invasive and non invasive measures. Invasive The Fick principle Fick described the following relationship in the 19th century: Q = M / (V - A) Q volume of blood flowing through an organ in a minute M number of moles of a substance added to the blood by an organ in one minute V are the venous concentrations of that substance A are the arterial concentrations of that substance This principle can be used to measure the blood flow through any organ that adds substances to, or removes substances from, the blood. The heart does not do either of these but the CO equals the pulmonary blood

86

flow, and the lungs add oxygen to the blood and remove carbon dioxide from it.

The concentration of the oxygen in the blood in the pulmonary veins is 200 ml/L and in the pulmonary artery is 150 ml/L, so each litre of blood going through the lungs takes up 50 ml. At rest, the blood takes up 250 ml/min of oxygen from the lungs and this 250 ml must be carried away in 50 ml portions; therefore, the CO must be 250/50 or 5 L/min. Limitations Original method described by Fick difficult to carry out. Accurate collection of the gas is difficult unless the patient has an endotracheal tube Analysis of the gas is straightforward if the inspired gas is air, but if it is oxygen-enriched air there are two problems, (a) the addition of oxygen may fluctuate and produce an error and (b) it is difficult to measure small changes in oxygen concentration at the top end of the scale. The denominator of the equation requires the mixed venous (i.e. pulmonary arterial) oxygen content to be measured and therefore a pulmonary artery catheter is needed to obtain the sample. Complications may arise from these catheters. Not practicable in routine clinical practice. Several variants of the basic method have been devised, but usually their accuracy is less good.

Dilution techniques: Thermodilution Dye dilution (eg. Indocyanin green, lithium) Principle: If a known quantity of an indicator substance is introduced to an unknown flow, assuming no indicator is lost and mixing is thorough the flow will be equal to the amount of indicator injected divided by the average downstream concentration of the indicator.

87

In practice this is done by measuring the downstream concentration of the indicator over time, and then integrating the area under the measured concentration-time curve to obtain the average indicator concentration. This is expressed by the Stewart-Hamilton equation:

where q is the quantity of indicator injected c is indicator concentration dt is change in time. Thermodilution

In order to measure the Cardiac output by this method, a PA floatation catheter is required. Indicator: Mass of cold Sensor: Thermistor

Method: PA catheter inserted through a central vein. It is 50cm long, has a thermistor 4cm from tip. Tripple lumen- air filled lumen for inflation of a small balloon at tip, proximal lumen 20cm from tip which sits in RA, distal lumen which opens at the tip distal to the balloon. A mass of indicator is added via the proximal lumen 20cm from the tip (in the RA). This is mixed with a volume of blood flowing past this point and is diluted with it. The thermistor measures the change in temperature over time. There is a thermistor at the injection port also to measure temp of injectate. Mass injected is mass of cold (derived from vol/ density/ temperature and specific heat capacity of injectate). Dependent also on catheter size and rate of injection. In practice a computer determines the CO by plotting the log of the indicator concentration against time. The initial decline in concentration, linear on a semilog plot, is extrapolated to the axis , giving the time for

88

the first passage of the indicator through the circulation. There is often a recirculation hump (less prominent in thermodilution). Accuracy: 3-13% variability compared with direct electromagnetic flow measurement Larger injectate volume , greater reproducibility. smooth & rapid injection Reject uneven curves. Best of three consecutive measurements Injection ideally less that 4 secs 5% Dex better than saline Clinical situations where result may be invalid TR /IPPV/ R-L shunt/ Rapid infusion into SVC/ RA port lies within sheath Calculation of CO is achieved using the Stewart-Hamilton equation. Application of this equation assumes three major conditions; complete mixing of blood and indicator, no loss of indicator between place of injection and place of detection and constant blood flow. The errors made are primarily related to the violation of these conditions.

The amount of indicator (n) is related to its mean concentration (c), cardiac output (Q) and the time for which it is detected (t2 - t1). Advantages : Indicator is non toxic Does not re-circulate. Repeat measurements are only limited by volume constraints and the time to regain temperature stability between injections The method shows good agreement with the fick and indocyanin green methods. However there is considerable variability. A clinically significant change in CO cannot be diagnosed with certainty unless there is a difference of approximately 15% between the mean of three CO determinations and the previous mean.

Dye dilution: A known amount of dye is injected into the pulmonary artery, and its concentration is measured peripherally.

89

Indocyanine green: Indocyanine green is suitable due to its low toxicity and short half-life. A curve is achieved, which is replotted semi-logarithmically to correct for recirculation of the dye. CO is calculated from the injected dose, the area under the curve (AUC) and its duration. (Short duration indicates high CO). Lithium: has also been used as an alternative to indocyanine green. It is injected via a central venous catheter and measured by a lithium-sensitive electrode incorporated into the radial arterial cannula. Limitations include: o It cant be used in patients recieiving lithium therapy o Electrode drift can occur in the presence of high peak doses of muscle relaxants o Abnormal shunts can result in erroneous cardiac output measurements o Ex vivo analysis requires disposal of sampled blood

Ultrasound-Based Methods for Cardiac Output Monitoring Uses the Doppler principle When ultrasound waves strike moving objects, these waves are reflected back to their source at a different frequency, termed the Doppler shift frequency, that is directly related to the velocity of the moving objects and the angle at which the ultrasound beam strikes these objects. The Doppler equation:

Where
Fd = frequency shift Ft = transmitted frequency V = velocity of flow cos = cosine of the angle of transmitted frequency to flow (normally assumed to be 1) C = velocity of sound through medium (normally assumed to be approx 1560 meters/s)

To measure blood flow velocity, this equation is rearranged to solve for velocity v=f.c/2.f0.cos Where f = Doppler shift frequency v = velocity of red blood cell targets f0 = transmitted ultrasound beam frequency Theta = angle between the ultrasound beam and the vector of red blood cell flow

90

c = velocity of ultrasound in blood (approximately 1570 m/sec) In general, measurement of blood flow velocity requires just a single measurement, the Doppler shift frequency, because the velocity of ultrasound in blood and the transmitted ultrasound frequency are known, and cosine theta is assumed to equal 1 as long as the angle of insonation is small. Hence it is important that the ultrasound beam be oriented as much as possible in a direction that is parallel to blood flow. Once the Doppler shift frequency is measured and blood flow velocity is calculated, stroke volume can be determined from the following Equation. SV = v ET CSA where SV = stroke volume (mL) v = spatial average velocity of blood flow (cm/sec) ET = systolic ejection time (sec) CSA = cross-sectional area of the vessel (cm2) Continious and pulsed wave Doppler are used to measure flow. Pulse wave Doppler allows the site of sampling to be specified, the target sample is usually the central laminar flow. With continious wave Doppler, a piezoelectrical crystal transmits the ultrasound beam while nother measures the frequency of refelected wave. The velocity of all the red cells moving along the path of the ultrasound beam are recorerded.

Oesophageal Doppler monitoring Measures the blood flow in the descending aorta with a Doppler probe usually 4 MHz continuous wave or 5 MHz pulsed wave depending upon device, incorporated in the tip of flexible probe. The probe is positioned in the oesophagus 30-40 cmm from the teeth. At this point the aorta lies parallel to the oesophagus and cross sectional area varies least. Aortic cross sectional area is either measured usuing M mode or calculated based upon nomograms. Advantages: 1. only short period of training is required 2. probe (6mm) are minimally invasive 3. contraindications are few severe agitation, pharyngo oesophageal pathology, aortic balloon counter-pulsation, aotic dissection or severe aortic coarctation

91

Disadvantages: Based on the assumption that descending aortic flow is the 70% of the total CO and nomograms accurately determine cross section area and cross sectional area is constant Flow in the aorta is not always laminar eg in valvular disease or severe anaemia Finding and maintaining optimal probe positioning Probe may be poorly tolerated in the poorly sedated patients

Transthoracic impedance Can be measured across externally applied electrodes. Impedance changes with the cardiac cycle (changes in blood volume). The rate of change of impedance is a reflection of cardiac output. It is thought to be useful in estimating changes but not for absolute measurements.

Limitations

Contraction of the heart produces a cyclical change in transthoracic impedance of about 0.5%, unfortunately giving a rather low signal to noise ratio. Although the method has been reported to give accurate results in normal subjects, several studies have some inaccuracy in critically ill patients,
Explain mechanisms and physiological consequences of alpha 1, alpha 2, beta 1 and beta 2 receptor blockade

Alpha receptor Rank order of potency of agonists Antagonist Effector pathway Adr NA > Iso Phenoxybenzamine IP3/DAG, cAMP , K+ channel

Beta receptor Iso > Adr > Na Propanolol cAMP , Ca 2+ channel

92

Beta 1 Location Heart, Jg cells in kidney

Beta 2 Bronchi, blood vessels, uterus, GIT, urinary tract, eye

Beta 3 Adipose tissue

Alpha 1 Location Postjunctional on effector organs

Alpha2 Prejunctional on nerve endings (2A), also post junctional in brain, pancreatic cells, platelets and extrajunctional in certain blood vessels Inhibition of transmitter release Vasoconstriction Decreased central sympathetic flow Decreased insulin release Platelet aggregation

Function subserved

Smooth muscle contraction Vasoconstriction Gland-secretion Gut relaxation Heart arrythmia

Effector pathway

IP3 /DAG , phospholipase A2

cAMP K channel Ca2+ channel or IP3/DAG

Receptor Antagonism Inhibit vasoconstriction induced by endogenous catecholamines Vasodilation may occur Fall in blood pressure due to decreased peripheral resistance

2 Adrenergic Receptor Antagonism


Activation of presynaptic 2 receptors inhibits the release of norepinephrine and other cotransmitters from peripheral

93 The magnitude of such effects is less in supine than in upright subjects and is particularly marked if there is hypovolemia. The fall in blood pressure is opposed by baroreceptor reflexes that cause increases in HR & CO, as well as fluid retention. These reflexes are exaggerated if the antagonist also blocks 2 receptors on peripheral sympathetic nerve endings, leading to enhanced release of norepinephrine and increased stimulation of postsynaptic 1 receptors in the heart and on juxtaglomerular cells. Blockade of 1 receptors also inhibits vasoconstriction and the increase in blood pressure produced by the administration of a sympathomimetic amine. They block pressor action of Adr which then produces only fall in BP due to beta 2 mediated vasodilatation - vasomotor reversal of dale. Tone of smooth muscle in the bladder trigone, sphincter and prostate is reduced by the blockade of alpha 1 receptors (mostly of the alpha 1A subtype) - urine flow in pts with BHP increases Alpha blockers can inhibit ejaculation; this may manifest as impotence Nasal stuffiness and miosis result from blockade of alpha receptors in the nasal blood vessels and radial muscles of iris respectively intestinal motility is increased due to partial inhibition of relaxant sympathetic influences diarrhea may occur sympathetic nerve endings and leads to a fall in blood pressure; Blockade of 2 receptors can increase sympathetic outflow and potentiate the release of norepinephrine from nerve endings, leading to activation of 1 and 1 receptors in the heart and peripheral vasculature with a consequent rise in blood pressure. The physiological role of vascular 2 receptors in the regulation of blood flow within various vascular beds is uncertain. the effect of blockade of platelet 2 receptors in vivo is not clear. Blockade of pancreatic 2 receptors may facilitate insulin release. Alpha receptor antagonists reduce smooth muscle tone in the prostate and neck of the bladder, thereby decreasing resistance to urine outflow in benign prostatic hypertrophy (see below). Beta 2 antagonism During sympathetic activity peripheral resistance increases as a result of blockade of vascular 2 receptors and activation of vascular receptors.

Beta 1 antagonism receptor antagonists slow the heart rate and decrease myocardial contractility, especially during exercise or stress. However stroke volume is often preserved.

long-term administration of these drugs to hypertensive patients ultimately leads to a fall in peripheral vascular resistance

Blockade of 2 receptors tends to blunt the increase in blood flow to active skeletal muscle during submaximal exercise

94 With long-term use of receptor antagonists, total peripheral resistance returns to initial values or decreases in patients with hypertension 1 Receptor antagonists reduce Sinus rate, slow conduction in atria and AV node and decrease automaticity.

Increased incidence of bronchospasm in obstructive airway disease

glycogenolysis in the human liver is at least partially inhibited after 2-receptor blockade

improved relationship between cardiac oxygen supply and demand; exercise tolerance generally is improved in patients with angina, whose capacity to exercise is limited by the development of chest pain The release of renin from the juxtaglomerular apparatus is stimulated by the sympathetic nervous system via 1 receptors, and this effect is blocked by receptor antagonists

Effects of non-selective betablockade: Metabolic Effects. Nonselective blockers may delay recovery from hypoglycemia in IDDM but infrequently in NIDDM. Block glycogenolysis. During hypoglycemia by blunt the perception of symptoms such as tremor, tachycardia, and nervousness. Attenuate the release of free fatty acids from adipose tissue (important source of energy for exercising muscle) Nonselective receptor antagonists consistently reduce HDL cholesterol, increase LDL cholesterol, and increase triglycerides. In contrast, 1selective antagonists, including celiprolol, carteolol, nebivolol, carvedilol, and bevantolol, reportedly improve the serum lipid profile of dyslipidemic patients.

95 In contrast to classical blockers, which decrease insulin sensitivity, the vasodilating receptor antagonists (e.g., celiprolol, nipradilol, carteolol, carvedilol, and dilevalol) increase insulin sensitivity in patients with insulin resistance.

Eye: Decreased aqueous secretion from the ciliary epithelium

Classify alpha and beta receptor blocking agents according to their pharmacokinetic and pharmacodynamic properties

Classification of alpha blockers: NON-COMPETITIVE ANTAGONISTS

-Haloalkylamines: Phenoxybenzamine

COMPETITIVE ANTAGONISTS Nonselective antagonists Ergot alkaloids Ergotamine, ergotoxin

Hydrogenated ergot alkaloids dihydroergotami ne, dihydroergotoxin Imidazolines Tolazoline, Phentolamine Miscellaneous chlorpromazine,

1 selective antagonists Prazocin Terazocin Doxazosin Tamsulosin

2 selective antagonists Yohimbine

96

ketanserine

Comparative Information About Adrenergic Receptor Antagonists HALOALKYLAMINES IMIDAZOLINES QUINAZOLINES Prototype Phenoxybenzamine Phentolamine Prazosin Antagonism Irreversible Competitive Competitive Selectivity 1 with some 2 Nonselective Selective for 1; does not between 1 and 2 distinguish among 1 subtypes Hemodynamic Decreased PVR and blood Similar to PBZ Decreased PVR and blood effects pressure pressure Venodilation is prominent Veins less susceptible than arteries; thus, postural hypotension less of a problem Cardiac stimulation Cardiac stimulation is less (cardiovascular reflexes and (NE release is not enhanced enhanced NE release due to due to 1 selectivity) 2 antagonism) Actions other Some antagonism of ACh, Cholinomimetic; At high doses some direct than 5-HT, and histamine adrenomimetic; vasodilator action, probably blockade histamine-like due to PDE inhibition actions Blockade of neuronal and Antagonism of 5extraneuronal uptake HT Routes of Intravenous and oral; oral Similar to PZB Oral administration absorption incomplete and erratic Adverse Postural hypotension, Same as PBZ, Some postural hypotension, reactions tachycardia, miosis, nasal plus GI especially with the first stuffiness, failure of disturbances due dose; less of a problem ejaculation to overall than with PBZ or cholinomimetic phentolamine and histaminelike actions Therapeutic Conditions of Same as PBZ Primary hypertension

97

uses catecholamine excess (e.g., Pharmacological/Pharmacokinetic classification/properties of Receptor Blocking Agents pheochromocytoma) DRUG MEMBRANE INTRINSIC LIPID EXTENT OF ORAL t 1/2 Peripheral vascular disease Benign prostatic STABILIZING AGONIST SOLUBILITY ABSORPTION BIOAVAILAB (hours) hypertrophy ACTIVITY ACTIVITY (%) ILITY (%) Classical non-selective blockers: First generation Nadolol 0 0 Low 30 30-50 20-24 Propranolol ++ 0 High <90 30 3-5 Timolol 0 0 Low to Moderate 90 75 4 Penbutolol 0 + High 100 ~100 ~5 Pindolol + +++ Low >95 ~100 3-4 1-Selective blockers: Second generation Acebutolol + + Low 90 20-60 3-4 Atenolol 0 0 Low 90 50-60 6-7 Bisoprolol 0 0 Low 90 80 9-12 Esmolol 0 0 Low NA NA 0.15 * Metoprolol + 0 Moderate ~100 40-50 3-7 Non-selective blockers with additional actions ( blocker): Third generation Carteolol 0 ++ Low 85 85 6 Carvedilol ++ 0 Moderate >90 ~30 7-10 Labetalol + + Low >90 ~33 3-4 1-selective blockers with additional actions ( 2 agonistic): Third generation Betaxolol + 0 Moderate >90 ~80 15 Celiprolol 0 + Low ~74 30-70 5 Beta 2 selective blocker Butoxamine

PROTEIN BINDING (%) 30 90 <10 80-98 40 26 6-16 ~30 55 12 23-30 98 ~50 50 4-5

Classification of beta blockers:

describe the pharmacology of alpha receptor blocking agents and apply this to their clinical use

Phenoxybenzamine
Physicochemical Structure

Haloalkylamine

98

Presentation MOA

10mg cap. And 50mg/ml inj. Pharmacodynamics

Use

Dose

CVS

CNS Respiratory Other Side effects/ adverse effects

blocks 1 and 2 receptors irreversibly Inhibits reuptake of released norepinephrine by presynaptic adrenergic nerve terminals. Blocks histamine (H1), acetylcholine, and serotonin receptors at high doses Treatment of pheochromocytoma almost always used to treat the patient in preparation for surgery. Was used for BPH; not used any more due to side effects. Has been used to control the manifestations of autonomic hyperreflexia in patients with spinal cord transection. A conservative approach is to initiate treatment with phenoxybenzamine (at a dosage of 10 mg twice daily) 1 to 3 weeks before the operation. The usual daily dose of phenoxybenzamine in patients with pheochromocytoma is 40 to 120 mg given in two or three divided portions. progressive decrease in peripheral resistance and reflex increase in CO Marked postural hypotension fatigue, sedation, and nausea Nasal stuffiness due to vasodilatation Smooth muscle relaxation in bladder and vas deference Postural hypotension with reflex tachycardia and other arrhythmias Exaggerated hypotension in volume depleted subjects Reversible inhibition of ejaculation Nasal stuffiness Mutagenic in ames test

Interactions Pharmacokinetics absorbed after oral administration, although bioavailability is low. Administered orally. Chronic administration leads to accumulation in adipose tissue Cyclizes spontaneously into highly active intermediate. half-life of phenoxybenzamine probably is less than 24 hours. However, the duration of its effect is dependent not only on the presence of the drug, but also on the rate of synthesis of receptors. irreversible blockade of long duration (1448 hours or longer). Most of the administered dose is excreted in urine in 24 hrs.

Absorption Distribution Metabolism

Excretion

Prazocin
Physicochemical

99

Structure Presentation MOA

contain a piperazinyl quinazoline nucleus. Structurally similar to terazocin 0.5, 1 and 2 mg tabs Pharmacodynamics

Use

Dose

CVS

CNS Respiratory Other Side effects/ adverse effects Interactions

The major effects result from its potent and selective blockade of 1 receptors in arterioles and veins. 1: 2 1000:1. Also inhibits phosphodiesterase. Hypertension BHP LVF refractory to diuretics and digitalis Raynauds disease HT: The initial dose should be 1 mg at bed time to reduce the risk of syncopal reactions (The first dose effect). the dose is titrated upward depending on the blood pressure. A maximal effect generally is observed with a total daily dose of 20 mg in patients with hypertension in 2 to 3 divided doses. BPH: 1 to 5 mg twice daily typically are used. TPR & venous return CO. usually does not increase heart rate as it has little or no 2 receptor-blocking effect and does not promote the release of norepinephrine from sympathetic nerve endings. depresses baroreflex function in hypertensive patients. suppress sympathetic outflow favorable effects on serum lipids, LDL and triglycerides and HDL 1st dose effect: postural syncope Nasal stuffiness Relaxes bladder smooth muscle

Absorption Distribution

Metabolism

Excretion

Pharmacokinetics Good oral absorption. bioavailability ~ 60%. Peak conc. reached in ~ 2 hrs after oral dose. Tightly bound to plasma proteins (primarily 1-acid glycoprotein) and only 5% of the drug is free Extensively metabolized in the liver. T1/2 ~ 2.5 hrs ( 6 to 8 hours in congestive heart failure). The duration of action: 7 to 10 hours in the treatment of hypertension. Excreted mainly in bile & little unchanged drug is excreted by the kidneys

Phentolamine (Regitine)
Physicochemical Structure Presentation Imidazoline 10mg/ml Inj. Pharmacodynamics

100

MOA

Use

Dose CVS

competitive receptor antagonist that has similar affinities for 1 and 2 receptors Inhibits reuptake of released norepinephrine by presynaptic adrenergic nerve terminals. Phentolamine also can block receptors for 5-HT, and it causes release of histamine from mast cells; phentolamine also blocks K+ channels short-term control of hypertension in patients with pheochromocytoma relieve pseudo-obstruction of the bowel in patients with pheochromocytoma used locally to prevent dermal necrosis after the inadvertent extravasation of an receptor agonist Treatment of hypertensive crises that follow the abrupt withdrawal of clonidine or that may result from the ingestion of tyramine-containing foods during the use of nonselective MAO inhibitors. Direct intracavernous injection of phentolamine (in combination with papaverine) has been proposed as a treatment for male sexual dysfunction. Buccally or orally administered phentolamine may have efficacy in some men with sexual dysfunction 5mg IV repeated as required progressive decrease in peripheral resistance and reflex increase in CO Marked postural hypotension fatigue, sedation, and nausea Nasal stuffiness due to vasodilatation
severe tachycardia, arrhythmias, and myocardial ischemia, especially after intravenous administration. With oral administration, adverse effects include tachycardia, nasal congestion, and headache.

CNS Respiratory Other Side effects/ adverse effects Interactions

Absorption Distribution Metabolism Excretion

Pharmacokinetics limited absorption after oral administration. Its pharmacokinetic properties are not well known; it may reach peak concentrations within an hour after oral administration and has a half-life of 57 hours

Tamsulosin

101

Structure Presentation MOA Use Dose CVS CNS Respiratory Other Side effects/ adverse effects Interactions

Physicochemical Benzenesulfonamide 0.4 mg capsules Pharmacodynamics a1 receptor antagonist with some selectivity for a1A (and a1D) subtypes compared to a1B subtype. This selectivity may favor blockade of a1A receptors in prostate Treatment of BPH with little effect on BP Tamsulosin may be administered at a 0.4-mg starting dose; a dose of 0.8 mg ultimately will be more efficacious in some patients Little effect on BP as compared to other alpha antagonists

Dizziness & retrogade ejaculation

Absorption Distribution Metabolism Excretion

Pharmacokinetics well absorbed orally It is extensively metabolized by CYPs in liver. T1/2 ~ 5 to 10 hours. Biliary excretion

describe the pharmacology of beta blockers with particular reference to propanolol, atenolol, metoprolol, esmolol, carvedilol, sotalol and labetalol

Propanolol
Physicochemical Structure Presentation MOA 10, 40, 80 mg tab, 1mg/ml Inj Pharmacodynamics

Use

Propranolol interacts with 1 and 2 receptors with equal affinity, lacks intrinsic sympathomimetic activity, and does not block receptors. HT Angina Arrythmias: supraventricular arrhythmias/tachycardias, ventricular arrhythmias/tachycardias, premature ventricular

102

Dose CVS

contractions, digitalis-induced tachyarrhythmias myocardial infarction pheochromocytoma prophylaxis of migraine variceal bleeding in portal hypertension generalized anxiety disorder For HT and angina initially 40 80 mg/day titrated upwards.
Heart-decreases HR, force of contraction and CO. Cardiac work and O2 consumption reduced. Decreases automaticity. AV conduction is delayed. Blood vessels: Initially TPR increases and CO decreases, little change in BP. With prolonged administration BP decreases due to decrease in TPR. This is due to (i) reduced NA release from sympathetic terminus (ii) Decreased rennin release from kidney (beta 1) (iii) decreased central sympathetic outflow Forgetfulness, increased dreaming and nightmares, suppresses anxiety Inreases bronchial resisatnce by blocking beta 2 receptors Metabolic: blocks adrenergically induced lipolysis reduced free fatty acids. Plasma TGL and LDL/HDL ratio is increased. Inhibits glycogenolysis. Carbohydrate intolerance during prolonged administration. Potent local anaesthetic Inhibits adrenergically induced tremors Reduce exercise capacity Reduced secretion of aqueous Relaxation of uterus is blocked Accentuates myocardial insufficiency- can ppt CHF and edema Bradycardia Risk of life threatening attack of asthma Exacerbates variant angina Carbohydrate intolerance Altered lipid profile Reduced exercise capacity- inability to increase blood flow Worsening of PVD-cold hands and feet

CNS Respiratory Other

Side effects/ adverse effects

Interactions

The bioavailability of propranolol may be increased by the concomitant ingestion of food and during long-term administration of the drug. Additive depression of Sinus node and AV conduction with digitalis and verapamil Delays recovery from hypoglycaemia Indomethacin and other NSAIDs attenuate anti-hypertensive action Propanolol decreases lignocaine metabolism by decreasing HBF
Pharmacokinetics highly lipophilic and is almost completely absorbed after oral administration. large volume of distribution (4 liters/kg), 90% protein bound.

Absorption Distribution

103

Metabolism

Excretion

Extensive hepatic metabolism systemic bioavailability ~ 25%. great interindividual variation in the clearance of propranolol by the liver; this contributes to enormous variability in plasma concentrations (approximately twentyfold). Metabolism depends upon hepatic blood flow. Prolonged administration decreases HBF and decreases metabolism Metabolites excreted in urine

The features of cardioselective beta blockers (metoprolol, atenolol, acebutolol) lower propensity to cause bronchoconstriction less interference with carbohydrate metabolism less chance of ppt raynauds phenomenon less deleterious effect on lipid profile ineffective in blocking essential tremor less liable to impair exercise tolerance

Atenolol
Physicochemical Structure Presentation MOA Use Dose 25, 50 mg tab Pharmacodynamics

1-selective antagonist that is devoid of intrinsic sympathomimetic and membrane stabilizing activity HT Angina The initial dose of atenolol for the treatment of hypertension usually is 50 mg per day, given once daily may be increased to 100 mg; higher doses are unlikely to provide any greater antihypertensive effect. The drug accumulates in patients with renal failure, and dosage should be adjusted for patients whose creatinine clearance is less than 35 ml/minute.

CVS CNS Respiratory Other Side effects/ adverse effects

104

Interactions Pharmacokinetics Atenolol is incompletely absorbed (about 50%), but most of the absorbed dose reaches the systemic circulation.

Absorption Distribution Metabolism

Excretion

There is relatively little interindividual variation in the plasma concentrations of atenolol; peak concentrations in different patients vary over only a fourfold range., and the elimination half-life is about 5 to 8 hours. The drug is excreted largely unchanged in the urine

Metoprolol
Physicochemical Structure Presentation MOA 25, 50, 100mg tab/ 5mg/ml Inj Pharmacodynamics

1-selective receptor antagonist that is devoid of intrinsic sympathomimetic activity and membrane-stabilizing activity. chronic heart failure. HT MI Tachyarrythmias Usual initial dose is 100 mg per day. Metoprolol generally is used in two divided doses for the treatment of stable angina

Use

Dose CVS CNS Respiratory Other Side effects/ adverse effects Interactions

Absorption Distribution

Pharmacokinetics Completely absorbed after oral administration, but bioavailability is relatively low (about 40%) because of first-pass metabolism.

105

Metabolism

Excretion

Metoprolol is extensively metabolized in the liver, with CYP2D6 the major enzyme involved. Plasma concentrations of the drug vary widely (up to seventeenfold), perhaps because of genetically determined differences in the rate of metabolism. The half-life of metoprolol is 3 to 4 hours, but can increase to 7 to 8 hours in CYP2D6 poor metabolizers. 90 % biliary excretion and 10% unchanged urinary excretion

Esmolol
Physicochemical Structure Presentation MOA 100mg/ml Inj Pharmacodynamics

Use

Dose CVS CNS Respiratory Other Side effects/ adverse effects Interactions

1-selective antagonist with a very short duration of action. It has little if any intrinsic sympathomimetic activity, and it lacks membranestabilizing actions SVT AF MI Hypertension Loading dose 0.5mg/kg followed by 0.05 0.2mg/kg/min

Pharmacokinetics Absorption Distribution Metabolism Excretion Administered IV apparent volume of distribution of approximately 2 liters/kg. Inactivated by blood esterases. T1/2 < 10min. Action last 15-20min after terminating infusion. Metabolite excreted in urine

Carvedilol

106

Physicochemical Structure Presentation MOA Use 6.25, 12.5, 25mg tab Pharmacodynamics 1+ 2+1 blocker + membrane-stabilizing activity

Dose CVS CNS Respiratory Other Side effects/ adverse effects Interactions

HT Angina CHF 6.25mg BD initially, titrate to max 25mg BD


Produces vasoldilatation and has anti-oxidant properties

Absorption Distribution Metabolism Excretion

Pharmacokinetics Oral bioavailability 30%. peak plasma concentrations occurring in 1 to 2 hours. It is highly lipophilic and thus is extensively distributed into extravascular tissues. It is >95% protein bound T1/2 2-8hrs. Extensively metabolized in the liver, predominantly by CYP2D6 and CYP2C9. The half life is 7 to 10 hours. Biliary excretion

Sotalol Structure Presentat ion MOA Physicochemical Formulated as a racemic mixture of d- and l-sotalol. 40, 80mg Tab Pharmacodynamics Both b-adrenergic receptor-blocking (class 2) and action potential prolonging (class 3) actions. All the b-adrenergic blocking activity resides in the lisomer; the d- and l-isomers share action potential

107

prolonging actions. Beta-adrenergic blocking action is not cardioselective and is maximal at doses below those required for action potential prolongation. Use Approved for the treatment of life-threatening ventricular arrhythmias (superior to lidocaine to terminate VT) and the maintenance of sinus rhythm in patients with atrial fibrillation. It is also approved for treatment of supraventricular and ventricular arrhythmias in the pediatric age group. Sotalol decreases the threshold for cardiac defibrillation. IV dose : 0.5 -1.5 mg/kg over 5 20 min. oral therapy initiated at 80mg 12 hrly & increased to 160mg 12 hrly Prolongs APD & ERP in atria, AV node, ventricles, and accessory pathways.

Dose

CVS

CNS Respirato ry Other Side effects/ adverse effects Interactio ns Absorptio n Distributi on Metabolis m

dose-related incidence of torsade de pointes Patients with overt heart failure may experience further depression of left ventricular function rare direct drug interactions because of simple pharmacokinetics Pharmacokinetics Orally bioavailability of approx. 100%.

half-life of approximately 12 hours.

108

Excretion

Excretion is predominantly by the kidneys in the unchanged form

Labetalol
Physicochemical Structure Presentation MOA 50, 100mg tab Pharmacodynamics

Use

competitive antagonists at both 1 and receptors. The potency of the mixture for receptor blockade is fivefold to tenfold that for 1 receptor blockade. The R,R isomer has some intrinsic sympathomimetic activity at 2 adrenergic receptors; this may contribute to vasodilation HT Pheochromocytoma Clonidine withdrawal
Oral: 50mg BD IV: 20-40mg every 10min till desired response is achieved 1 Receptor blockade leads to relaxation of arterial smooth muscle and vasodilation, particularly in the upright position. The 1 blockade also contributes to a fall in blood pressure, in part by blocking reflex sympathetic stimulation of the heart. In addition, the intrinsic sympathomimetic activity of labetalol at 2 receptors may contribute to vasodilation. Relatively high doses decrease CO & TPR

Dose CVS

CNS Respiratory Other Side effects/ adverse effects Interactions Pharmacokinetics Complete absorption from the gut Limb blood flow increases in contrast to propanolol
Labetalol has been associated with hepatic injury in a limited number of patients

Absorption Distribution Metabolism

Extensive first-pass clearance; bioavailability is only about 20% to 40% and is highly variable. Bioavailability may be increased by food intake. The drug is rapidly and extensively metabolized in the liver by oxidative biotransformation and glucuronidation; very little unchanged drug is found in the urine. The rate of metabolism of labetalol is

109

Excretion

sensitive to changes in hepatic blood flow. The elimination half-life of the drug is about 8 hours. The half-life of the R,R isomer of labetalol (dilevalol) is about 15 hours. Mainly biliary excretion

Classify the mechanisms of action of the anti-hypertensive agents


1. ACE inhibitors: Captopril, Enalapril, Lisinopril, Perindopril, Ramipril 2. Angiotensin (AT1) antagonists: Losartan, Candesartan, Irbesartan 3. Calcium channel blockers: Verapamil, Diltiazem, Nifedipine, Felodipine, Amlodipine, Nitrendipine, Lacidipine 4. Diuretic: a. Thiazides: Hydrochlorthiazide, chlorthalidone, indapamide b. High ceiling: Furosemide c. K sparing: Spironolactone, triamterene, Amiloride 5. adrenergic blockers: Metoprolol, propanolol, atenolol etc 6. + adrenergic blockers: labetalol, Carvedilol 7. adrenergic blockers: prazocin, terazocin, phentolamine, phenoxybenzamine 8. Central sympatholytic: clonidine, methyldopa 9. Vasodilator: a. Arterial: hydralazine, minoxidil, diazoxide b. Arterial + Venous: sodium nitroprusside

Describe the pharmacology of centrally acting agents such as clonidine and alpha-methyl dopa

Clonidine
Physicochemical Structure Presentation MOA

Use

Dose CVS

Imidazoline 150g tab, transdermal patch Pharmacodynamics Partial agonist with high affinity and high intrinsic activity at 2 receptors specially 2A receptor present post junctionally in medulla fall in BP and bradycardia. Plasma NA decreases. HT Opioid withdrawal Analgesia Attenuates vasomotor symptoms of menopause Control loose motions due to diabetic neuropathy Clonidine suppression test for pheochromocytoma Started with 100 g OD or BD maximum 300 g TDS When the sympathetic tone to the vasculature is low, the major effect is to reduce both HR and SV When sympathetic outflow to the vasculature is normally increased eg

110

CNS Respiratory Other Side effects/ adverse effects

when upright, these drugs reduce vascular resistance postural hypotension. The decrease in cardiac sympathetic tone leads to a reduction in myocardial contractility and heart rate; this could promote congestive heart failure in susceptible patients. Rapid IV injection raises BP transiently due to activation of peripheral post synaptic vasoconstrictor 2B Therapeutic window phenomenon: optimum lowering of BP occurs between blood levels of 0.2-2.0 ng/ml Decreased sympathetic flow to kidney results in reduced rennin release (see side effects) Sedation Xerostomia, dry nasal mucosa, dry eyes, and parotid gland swelling and pain. Postural hypotension and erectile dysfunction vivid dreams or nightmares, restlessness, and depression. Impotence symptomatic bradycardia and sinus arrest in patients with dysfunction of the sinoatrial node and AV block in patients with AV nodal disease or in patients taking other drugs that depress AV conduction. Sudden discontinuation increased sympathetic discharge withdrawal syndrome consisting of headache, apprehension, tremors, abdominal pain, sweating, tachycardia & HT.

Interactions

Absorption Distribution Metabolism Excretion

Rare Diuretics potentiate the hypotensive effect. Tricyclic antidepressants may inhibit the antihypertensive effect Pharmacokinetics Well absorbed orally, peak occurs in 2-4hrs T1/2 ~ 8-12 hrs. Effect of single dose lasts 6-24 hrs. -2/3rd of oral dose is excreted unchanged in urine rest as metabolite. Dose modification required in renal insufficiency

Methyl Dopa
Physicochemical

111

Structure Presentation MOA

Use Dose CVS

analog of 3,4-dihydroxyphenylalanine (DOPA), 250mg tab Pharmacodynamics Centrally acting antihypertensive agent. It is a prodrug. metabolized by the L-aromatic amino acid decarboxylase in adrenergic neurons to -methyldopamine, which then is converted to methylnorepinephrine. -Methylnorepinephrine is stored in the secretory vesicles of adrenergic neurons, substituting for norepinephrine (NE) itself. Thus, when the adrenergic neuron discharges its neurotransmitter, -methylnorepinephrine is released instead of norepinephrine. Because -methylnorepinephrine is as potent as norepinephrine as a vasoconstrictor, its substitution for norepinephrine in peripheral adrenergic neurosecretory vesicles does not alter the vasoconstrictor response to peripheral adrenergic neurotransmission. Methylnorepinephrine acts as an agonist at presynaptic 2 adrenergic receptors in the brainstem, attenuating NE release and thereby reducing the output of vasoconstrictor adrenergic signals to the peripheral sympathetic nervous system. Preferred drug for treatment of hypertension during pregnancy based on its effectiveness and safety for both mother and fetus. The usual initial dose of methyldopa is 250 mg twice daily, and there is little additional effect with doses above 2 g per day.
It lowers blood pressure chiefly by reducing peripheral vascular resistance, with a variable reduction in heart rate and cardiac output. Most cardiovascular reflexes remain intact after administration of methyldopa Postural (orthostatic) hypotension may occur in volume-depleted patients.

The fall in arterial pressure is maximal 6 to 8 hours after an oral or intravenous dose.

CNS Respiratory Other Side effects/ adverse effects

Methyldopa enters the brain via an aromatic amino acid transporter

overt sedation ( most frequent ) mental lassitude and impaired mental concentration Nightmares, mental depression, vertigo, and extrapyramidal signs Lactation, associated with increased prolactin secretion positive Coombs test, which sometimes makes cross-matching blood for transfusion difficult and rarely is associated with hemolytic anemia, as well as hepatitis and drug fever. The Coombs test may remain positive for as

long as a year after discontinuation of methyldopa, but the hemolytic anemia usually resolves within a matter of weeks.

Interactions Pharmacokinetics

112

Absorption Distribution Metabolism

Absorbed by an active amino acid transporter. Peak concentrations in plasma occur after 2 to 3 hours. small apparent volume (0.4 liter/kg) Eliminated with a half-life of about 2 hours. The usual oral dose of
methyldopa produces its maximal antihypertensive effect in 46 hours, and the effect can persist for up to 24 hours. Because the effect depends on accumulation and storage of a metabolite (-methylnorepinephrine) in the vesicles of nerve endings, the action persists after the parent drug has disappeared from the circulation.

Excretion

Methyldopa is excreted in the urine primarily as the sulfate conjugate (50% to 70%) and as the parent drug (25%). The remaining fraction is excreted as other metabolites

Describe the physiology and pharmacology of the vascular endothelium and smooth muscle with particular reference to nitric oxide

Endothelial Cells and Vascular Smooth Muscle Large number of substances can induce the contraction or relaxation of vascular smooth muscle. Many of these substances do so by acting directly on the arteriolar smooth muscle, but others act indirectly via the endothelial cells adjacent to the smooth muscle. Endothelial cells secrete several paracrine agents that diffuse to the adjacent vascular smooth muscle and induce either vasodilation or vasoconstriction. One very important paracrine vasodilator is nitric oxide which was called endothelium derived relaxing factor (EDRF). Nitric oxide is released continuously in significant amounts by endothelial cells in the arterioles and contributes to arteriolar vasodilation in the basal state. Another vasodilator released by endothelial cells is the eicosanoid prostacyclin (PGI2). There is little basal secretion of PGI2, but secretion can increase markedly in response to various inputs. One of the important vasoconstrictor paracrine agents released by endothelial cells in response to certain mechanical and chemical stimuli is endothelin-1 (ET-1). ET-1 is a member of the endothelin family of peptide paracrine agents secreted by a variety of cells in diverse tissues, including the brain, kidneys, and lungs. Not only does ET-1 serve as a paracrine agent but under certain circumstances it can also achieve high enough concentrations in the blood to serve as a hormone, causing widespread arteriolar vasoconstriction. In response to increased shear stress, arterial endothelium releases PGI2, increased amounts of nitric oxide, and less ET-1. All these changes cause the arterial vascular smooth muscle to relax and the artery to dilate. This flow-induced arterial vasodilation may be important in remodeling of arteries and in optimizing the blood supply to tissues under certain conditions.

113

There are several signal transduction mechanisms that modulate intracellular calcium concentration and therefore the state of vascular tone. Three different mechanisms are: 1) Gs-protein-coupled pathway, 2) phosphatidylinositol pathway, and 3) nitric oxide-cGMP pathway. Gs-Protein Coupled Signal Transduction Gs-protein coupled pathway stimulates adenylyl cyclase (AC), which catalyzes the formation of cAMP. An increase in cAMP in vascular smooth muscle causes relaxation. cAMP inhibits Myosin light chain kinase (MLCK) which phosphorylates myosin and causes contraction The Gs-protein is coupled to several important receptors that bind vasodilator substances:

2-adrenoceptors A2 purinergic receptors IP receptors (bind prostacyclin, PGI2).

IP3- Coupled Signal Transduction

114

The phosphatidylinositol pathway: activated phospholipase C converts phosphatidylinositol (PIP2) to inositol triphosphate (IP3) which then stimulates the sarcoplasmic reticulum (SR) to release calcium. The formation of diacylglycerol (DAG) activates protein kinase C (PK-C), which can also contribute to vascular smooth muscle contraction via protein phosphorylation. This pathway is coupled to following receptors: 1-adrenoceptors (Noradrenaline) AT1 receptors (Angiotensin) ETA receptors (Endothelin) V1 receptors (vasopressin) M3receptors (acetylcholine)

cGMP-Coupled Signal Transduction Very important in regulating vascular smooth muscle tone. Vascular endothelial cells normally produce NO, which diffuses from the endothelial cell to adjacent smooth muscle cells where it activates guanylyl cyclase leading to increased formation of cGMP and vasodilation. Nitric oxide (NO) is produced by many cells in the body; however, its production by vascular endothelium is particularly important in the regulation of blood flow. NO Biosynthesis

115

NO is produced from the amino acid L-arginine by the enzymatic action of nitric oxide synthase (NOS). Two endothelial forms of NOS: o constitutive NOS (cNOS; type III) o inducible NOS (iNOS; type II) Co-factors for NOS: o flavin adenine nucleotides o Oxygen o NADPH o Tetrahydrobiopterin In addition to endothelial NOS, there is a neural NOS (nNOS; type I) that serves as a transmitter in the brain and in different nerves of the peripheral nervous system, such as non-adrenergic, noncholinergic (NANC) autonomic nerves that innervate penile erectile tissues and other specialized tissues in the body to produce vasodilation. Under basal conditions in blood vessels, NO is continually being produced by cNOS. The activity of cNOS is calcium and calmodulin dependent. There are two basic pathways for the stimulation of cNOS, both of which involve release of calcium ions from subsarcolemmal storage sites: o Flow-dependent NO formation: shearing forces generated by blood flow acting on the vascular endothelium causes a release of calcium and subsequent cNOS activation. o Receptor-stimulated NO formation: endothelial receptors for a variety of ligands stimulate calcium release and subsequent NO production. Eg. Receptors for acetylcholine, bradykinin, substance-P, adenosine, and many others vasoactive substances. iNOS differs from cNOS in that its activation is calcium independent. The activity of iNOS is stimulated during inflammation by bacterial endotoxins (e.g., lipopolysaccharide) and cytokines such as tumor necrosis factor (TNF) and interleukins.

116

Intracellular Mechanisms NO has a half-life of only a few seconds, because: o Superoxide anion has a high affinity for NO & reduces NO bioavailability. o Diffuses into blood where it binds to heme moiety of hemoglobin and subsequently broken down o Diffuses into the vascular smooth muscle cells adjacent to the endothelium where it binds to and activates guanylyl cyclase

Activated guanylyl cyclase catalyzes the dephosphorylation of GTP to cGMP, which serves as a second messenger and induces smooth muscle relaxation by:

o increased intracellular cGMP, which inhibits calcium entry into the cell, and decreases intracellular calcium concentrations o activates K+ channels, which leads to hyperpolarization and relaxation o stimulates a cGMP-dependent protein kinase that activates myosin light chain phosphatase, the enzyme that dephosphorylates myosin light chains, which leads to smooth muscle relaxation.

Increased cGMP also has an important anti-platelet, antiaggregatory effect.

Vascular Effects of NO Vascular actions of NO include the following:


Direct vasodilation (flow dependent and receptor mediated) Indirect vasodilation by inhibiting vasoconstrictor influences (e.g., inhibits angiotensin II and sympathetic vasoconstriction) Anti-thrombotic effect - inhibits platelet adhesion to the vascular endothelium Anti-inflammatory effect - inhibits leukocyte adhesion to vascular endothelium; scavenges superoxide anion Anti-proliferative effect - inhibits smooth muscle hyperplasia

Because of the above actions of NO, when its production is impaired or its bioavailability is reduced, the following can result:

117

Vasoconstriction (e.g., coronary vasospasm, elevated systemic vascular resistance, hypertension) Thrombosis due to platelet aggregation and adhesion to vascular endothelium Inflammation due to upregulation of leukocyte and endothelial adhesion molecules Vascular hypertrophy and stenosis

Diseases or Conditions Associated with Abnormal NO Production and Bioavailability Hypertension Obesity Dyslipidemias (particularly hypercholesterolemia and hypertriglyceridemia) Diabetes (both type I and II) Heart failure Atherosclerosis Aging Cigarette smoking NO releasing drugs: o Nitroglycerine o Amylnitrite o Isosorbide dinitrate o Nitroprusside

describe the pharmacology of calcium antagonists with reference to the management of hypertension

Mechanisms of Action. An increased concentration of cytosolic Ca2+ causes increased contraction in cardiac and vascular smooth muscle cells. The entry of

118 extracellular Ca2+ is more important in initiating the contraction of cardiac myocytes (Ca2+-induced Ca2+ release). The release of Ca2+ from intracellular storage sites also contributes to contraction of vascular smooth muscle, particularly in some vascular beds. Cytosolic Ca2+ concentrations may be increased by various contractile stimuli. The Ca2+ channel antagonists produce their effects by binding to the 1 subunit of the L-type Ca2+ channels and reducing Ca2+ flux through the channel.

Pharmacological Properties Cardiovascular Effects. Actions in Vascular Tissue. An increase in cytosolic Ca2+ results in enhanced binding of Ca2+ to calmodulin. The Ca2+-calmodulin complex in turn activates myosin light-chain kinase, with resulting phosphorylation of the myosin light chain. Such phosphorylation promotes interaction between actin and myosin and contraction of smooth muscle. Ca2+ channel antagonists inhibit the voltage-dependent Ca2+ channels in vascular smooth muscle. All Ca2+ channel blockers relax arterial smooth muscle, but they have little effect on most venous beds and hence do not affect cardiac preload significantly. Actions in Cardiac Cells. Within the cardiac myocyte, Ca2+ binds to troponin, relieving the inhibitory effect of troponin on the contractile apparatus and permitting a productive interaction of actin and myosin leading to contraction. Thus Ca2+ channel blockers can produce a negative inotropic effect. Although this is true of all classes of Ca2+ channel blockers, the greater degree of peripheral vasodilation seen with the dihydropyridines is accompanied by a sufficient baroreflex-mediated increase in sympathetic tone to overcome the negative inotropic effect. In the SA and AV nodes, depolarization largely depends on the movement of Ca2+ through the slow channel. The effect of a Ca2+ channel blocker on AV conduction and on the rate of the sinus node pacemaker depends on whether or not the agent delays the recovery of the slow channel. Although nifedipine reduces the slow inward current in a dose-dependent manner, it does not affect the rate of recovery of the slow Ca2+ channel. Nifedipine does not affect conduction through the node. In contrast, verapamil not only reduces the magnitude of the Ca2+ current through the slow channel but also decreases the rate of recovery of the channel. Verapamil and diltiazem depress the rate of the sinus node pacemaker and slow AV conduction; the latter effect is the basis for their use in the treatment of supraventricular tachyarrhythmias. Its electrophysiological properties lead to slowing of the heart rate, prolongation of the AV nodal effective refractory period, and importantly, prolongation of the QTc interval. Particularly in the setting of hypokalemia, the last effect can be associated with torsades de pointes, a potentially lethal ventricular arrhythmia

119

Hemodynamic Effects. All the Ca2+ channel blockers increase coronary blood flow. The dihydropyridines are more potent vasodilators in vivo and in vitro than verapamil, which is more potent than diltiazem. Nifedipine causes selective dilation of arterial resistance vessels. The decrease in arterial blood pressure elicits sympathetic reflexes, with resulting tachycardia and positive inotropy. Nifedipine also has direct negative inotropic effects in vitro. venous tone does not change. The other dihydropyridines amlodipine, felodipine, isradipine, nicardipine, nisoldipine, and nimodipineshare many of the cardiovascular effects of nifedipine. Nimodipine has high lipid solubility and was developed as an agent to relax the cerebral vasculature. It is effective in inhibiting cerebral vasospasm and has been used primarily to treat patients with neurological defects associated with cerebral vasospasm after subarachnoid hemorrhage.

Absorption, Fate, and Excretion. Although the absorption of these agents is nearly complete after oral administration, their bioavailability is reduced, in some cases markedly, by first-pass hepatic metabolism. The effects of these drugs are evident within 30 to 60 minutes of an oral dose, with the exception of the more slowly absorbed and longer-acting agents amlodipine, isradipine, and felodipine. For comparison, peak effects of verapamil occur within 15 minutes of its intravenous administration. These agents all are bound extensively to plasma proteins (70% to 98%); their elimination half-lives vary widely and range from 1.3 to 64 hours. During repeated oral administration, bioavailability and half-life may increase because of saturation of hepatic metabolism. A major metabolite of diltiazem is desacetyldiltiazem, which has about one-half of diltiazem's potency as a vasodilator. N-Demethylation of verapamil results in production of norverapamil, which is biologically active but much less potent than the parent compound. The half-life of norverapamil is about 10 hours. The metabolites of the dihydropyridines are inactive or weakly active. In patients with hepatic cirrhosis, the bioavailabilities and half-lives of the Ca2+ channel blockers may be increased, and dosage should be decreased accordingly. The half-lives of these agents also may be longer in older patients. Toxicity and Untoward Responses. dizziness, hypotension, headache, flushing, ankle edema, digital dysesthesia, and nausea. constipation, peripheral edema, coughing, wheezing, and pulmonary edema. muscle cramps. Less common side effects include rash, somnolence, and occasional minor elevations of liver function tests. Worsened myocardial ischemia has been observed in two studies with the dihydropyridine nifedipine - coronary steal. Although bradycardia, transient asystole, and exacerbation of heart failure have been reported with verapamil, these responses usually have occurred after intravenous administration of verapamil in patients with disease of the SA node

120 or AV nodal conduction disturbances or in the presence of adrenergic receptor blockade. In the setting of hypokalemia and/or bradycardia, polymorphic ventricular tachycardia (torsades de pointes), a potentially lethal arrhythmia, can be seen. Agranulocytosis also has been reported. Interactions: blockers additive depression of conduction Digitalis increased levels Cardiac depressants like quinidine and disopyramide Therapeutic Uses Variant & Exertional Angina antiarrhythmic agents hypertension heart failure Nimodipine has been approved for use in patients with neurological deficits secondary to cerebral vasospasm after the rupture of a congenital intracranial aneurysm. Nifedipine, diltiazem, amlodipine, and felodipine appear to provide symptomatic relief in Raynaud's disease.

121

describe in detail the pharmacodynamics and pharmacokinetics of sodium nitroprusside and glyceryl trinitrate including their adverse effects

Sodium nitroprusside
Physicochemical Structure

Presentation MOA

50mg vial

Pharmacodynamics Nitroprusside undergoes reduction in red cells & endothelial cells to form MetHb and release NO and 5 cyanide molecules. NO activates the guanylyl cyclase cGMP PKG pathway, leading to vasodilation Tolerance does not develop to SNP unlike nitroglycerin
Hypertensive emergencies Short-term reduction of cardiac preload and/or afterload improve cardiac output in congestive heart failure, especially in hypertensive patients with pulmonary edema that does not respond to other treatment. Decrease myocardial oxygen demand after acute myocardial infarction.

Use

122 Controlled hypotension to reduce bleeding In the treatment of acute aortic dissection along with blocker

Dose

Because the compound decomposes in light, only fresh solutions should be used, and the bottle should be covered with an opaque wrapping. Rapid photoreduction occurs to form HCN- which changes colour of solution from browny-red blue. The drug must be administered as a controlled continuous infusion, and the patient must be closely observed. The majority of hypertensive patients respond to an infusion of 0.25 to 1.5 g/kg per minute
Dilates both arterioles and venules: combination of venous pooling and reduced arterial impedance. In subjects with normal left ventricular function, venous pooling predominates and CO decreases, whereas in poor LV function reduced afterload and increased cardiac output predominates. renal blood flow and glomerular filtration are maintained, and plasma renin activity increases. Only a modest increase in HR and an overall reduction in myocardial demand for oxygen unlike minoxidil, hydralazine, diazoxide, and other arteriolar vasodilators

CVS

CNS Respiratory Other Side effects/ adverse effects

Headache and nausea due to raised ICP VQ mismatch


Cyanide toxicity: o Occurs when sodium nitroprusside is infused at a rate greater than 5 g/kg/min or about 2 g/kg per minute for a prolonged period. o Limiting factor in the metabolism of cyanide availability of sulfur-containing substrates in the body (mainly thiosulfate). The concomitant administration of sodium thiosulfate can prevent accumulation of cyanide. o CN- binds inactive cytochrome oxidase inhibiting oxidative phosphorylation anaerobic metabolism lactic acidosis o o O2 utilisation mixed venous O2 content Suspect in Pts with HT resistant to therapy: o HR Diaphoretic MV Can progress CNS Sx inc seizure, coma

risk in hypothermic patients-rate of rhodanase

123 conversion CNo Treatment: Cease infusion O2 available supplemental O2 Correct acidosis NaHCO3 Sulfur donors IV Na thiosulfate (150mg/kg over 15min) CN- binders hydroxycobalbumin (Vit B12a) cyanocobalbumin (Vit B12) metHb Na nitrite 5mg/kg slow IV **if severe**

Thiocyanate toxicity: o Risk increases with infusions > 24 to 48 hrs especially if renal function is impaired o Symptoms - anorexia, nausea, fatigue, disorientation, and toxic psychosis o Plasma conc. of thiocyanate should be monitored and should not be allowed to exceed 0.1 mg/ml o Rarely, excessive concentrations of thiocyanate may cause hypothyroidism by inhibiting iodine uptake by the thyroid gland. In patients with renal failure, thiocyanate can be removed readily by hemodialysis.

MetHb: o Unlikely to accumulate to levels which are toxic, even in Pts with congenital MetHb reductase deficiency o To develop 10% metHb need 10mg/kg SNP (really high dose) o Treatment: methylene blue (1-2mg/kg) BUT not advised as metHb needed for CN- clearance
Worsening of arterial hypoxemia in patients with COPD because the drug interferes with hypoxic pulmonary vasoconstriction ventilation perfusion mismatch. ICP and CPP Platelet aggregation

Interactions Pharmacokinetics

124 Unstable molecule - decomposes under alkaline conditions or when exposed to light Onset of action is within 30 seconds; the peak hypotensive effect occurs within 2 minutes, and when the infusion of the drug is stopped, the effect disappears within 3 minutes. Nitroprusside undergoes reduction in red cells to form MetHb and release NO and 5 cyanide molecules. MetHb binds one cyanide molecule to form non-toxic compound. Cyanide is further metabolized by liver mitochondrial enzyme rhodanase to thiocyanate, which is eliminated almost entirely in the urine. The mean elimination half-time for thiocyanate is 3 days in patients with normal renal function, and it can be much longer in patients with renal insufficiency.

GTN
Physicochemical
Structure

Presentation

0 1 2 3 4 5

S/L spray 400mcg/dose S/L tablets 300-600mcg Buccal tabs 1-5mg Oral tablets 2.6-10mg Patch 5-15mg/24hrs Injection 1-5mg/ml diluted to 100mcg/ml (0.01%)

MOA

Pharmacodynamics Nitroglycerin is denitrated by glutathione S-transferase. Free nitrite ion is released, which is then converted to nitric oxide. A different unknown enzymatic reaction releases nitric oxide directly from the parent drug molecule. NO activates guanylyl cylase cGMP Ca2+ influx into vascular smooth mm / Ca2+ uptake into smooth ER Overall Ca2+ in cytoplasm relaxation smooth mm vasodilatation

Use

Angina CHF & acute LVF MI Biliary colic Esophageal spasm Cyanide poisoning

125

Dose

Drug "Short-acting" Nitroglycerin, sublingual NTG infusion

Dose

Duration of Action 1030 minutes

0.151.2 mg 5g/min - 100g/min

"Long-acting" Nitroglycerin, oral sustained-action Nitroglycerin, 2% ointment, transdermal Nitroglycerin, slowrelease, buccal Nitroglycerin, slowrelease patch, transdermal
CVS

6.513 mg per 68 hours 11.5 inches per 4 hours 12 mg per 4 hours

68 hours 36 hours 36 hours

1025 mg per 24 810 hours hours (one patch per day)

Vessels: - 1 venodilatation 1 tendency for VR 2 preload RV 3 - Vasodilation 4 end-diastolic pressure / vent wall tension afterload Heart: - metabolic O2 requirements 1 2 above factors myocardial work O2 demand 2 - coronary BF 3 2 vent wall tension (afterload), redirecting blood flow to subendocardium 4 2 coronary vasodilatation improve O2 supply 5 - Results in favourable supply:demand ratio

126

- CO o VR CO in normal Pts HF Pts CO 2 SVR and improved myocardial performance Periphery: - Vasodilatation o Orthostatic hypotension o High doses systemic vascular resistance (SVR) CNS Respiratory MAP more pronounced in volume depleted CBF/ICP 2 vasodilatation, headache 1 PVR capacitance of pulmonary vessels favour absorption transudate 2 Release of hypoxic pulmonary vasoconstriction shunt - Uterus 3 uterine tone 4 blood flow risk haemorrhage Haematological 1 Rarely precipitates metHb 2 Platelets cGMP Ca2+ in cytoplasm platelet aggregation Side effects/ adverse effects Methemoglobinemia (rare): Nitrite ion reacts with hemoglobin (which contains ferrous iron) to produce methemoglobin (which contains ferric iron). Because methemoglobin has a very low affinity for oxygen, large doses of nitrites can result in pseudocyanosis, tissue hypoxia, and death. orthostatic hypotension tachycardia Throbbing headache. TOLERANCE: may be caused in part by a decrease in tissue sulfhydryl groups. Increased generation of oxygen free radicals during nitrate therapy may be another important mechanism of tolerance

Other

127

Interaction s Absorption

Sildenafil dangerous hypotension Pharmacokinetics High first pass metabolism oral bioavailability < 10%

Distribution The volume of distribution following intravenous administration is 3.3 L/kg Metabolism Unchanged nitrate compounds have half-lives of only 28 minutes. Partially denitrated metabolites have much longer half-lives (up to 3 hours). Nitroglycerin metabolites - two dinitroglycerins and two mononitro forms. the dinitro derivatives have significant vasodilator efficacy Excretion Excretion, primarily in the form of glucuronide derivatives of the denitrated metabolites, is largely by way of the kidney

Describe the pharmacology of the ACE inhibitors and angiotensin receptor antagonists with reference to the management of hypertension

128

Differences amongst ACE inhibitors are primarily pharmacokinetic, no single drug is superior to others

Captopril
Structure

Physicochemical Sulphydryl containing dipeptide surrogate of proline

Presentation MOA Use

12.5, 25mg tab Pharmacodynamics Inhibits ACE and abolishes pressor action of angiotensin I. Also increases plasma levels of kinins. (see diagram above) HT: advantages no postural hypotension, no CNS effects, safe in asthmatics, prevention of secondary hyperaldosteronism, renal blood flow mentained, reverse left ventricular hypertrophy, no effect on lipid profile, no rebound HT, reduce cardiovascular morbidity and increase life expectancy. CHF: reduce afterload and preload. CO. Accumulated

129

Dose

salt and water is lost because of increased renal perfusion and abolition of mineralocorticoid mediated Na+ retension. Cardiac work (HR*SBP) and exercise capacitance MI Prophylaxis in high risk cardiovascular risk subjects Diabetic nephropathy prevents or delays ESRD Hypertensive nephropathy Scleroderma crisis-BP and renal function is caused by AII. ACE inhibitors Dramatic improvement and life saving Started with 25mg BD gradually increased to 50mg TDS. In CHF start with 6.25mg BD to avoid sudden fall in BP. Should be taken 1 hr before or 2 hr after a meal.
relax smooth muscle and thereby promote vasodilationarterioles dilate and compliance of large arteries TPR increase renal salt and water excretion, reduce plasma volume Both systolic and diastolic BP falls. Magnitude of drop in BP depends on Na+ and renin status. decrease cellular hypertrophy.

CVS

CNS Respiratory Other

CV reflexes intact and no effect on capacitance vessels. Reflex increase in aldosterone is abolished Poor penetration Cough in 15% due to inhibition of bradykinin/substance P
Renal: renal blood flow is mentained. Anti-proliferative effects. in the treatment of patients with diabetes, slowing the development and progression of diabetic glomerulopathy. slowing the progression of other forms of chronic renal disease, such as glomerulosclerosis

Side effects/ adverse effects

Well tolerated if daily dose is < 150mg


Hypotension: especially in those with high pre-treatment renin/angiotensin II eg vol depleted, CHF, cirrhosis, renovascular HT Hyperkalemia - when ACE inhibitors are used with other drugs that can cause K+ retention reduced renal function, including that associated with bilateral renal artery stenosis and stenosis in the artery of a solitary kidney Cough, an adverse effect of ACE inhibitors, is less frequent with

130 angiotensin II receptor antagonists. Angioedema occurs very rarely. Teratogenicity Cough with ace inhibitors Headache, dizziness, nausea and GI upset Rare granulocytopenia and protienuria especially in renal disease

Interaction s

Indomethacin (and other NSAID) attenuate the hypotensive action. Hyperkalemia with K supplements/K sparing diuretics Antaacids reduce bioavailability of captopril Li+ clearance is reduced predisposition to toxicity Pharmacokinetics Absorption 70% oral bioavailability. Food reduces absorption. Distribution Penetration in brain is poor. Metabolism T1/2 ~ 2 hrs. Action lasts 6-12 hrs. Excretion Partly metabolized and partly excreted unchanged in urine. 40% to 50% as captopril and the rest as captopril disulfide dimers and captopril-cysteine disulfide

Pharmacology of AT1 antagonists differ from ACE inhibitors in following ways: 1. No interference with degradation of bradykinin and substance p 2. More complete inhibition of AT1 receptor activation. With ACE inhibitors the alternative pathways lead to formation of some Angiotensin II and hence some ATI receptor activation does occur 3. The AT2 receptor: adrenal medulla, kidney, and in the CNS, and may play a role in vascular development. renin and angiotensin II conc. increased during AT1-receptor antagonism act on uninhibited AT2 receptors elicit antigrowth and antiproliferative responses. 4. The AT1 receptor: vascular and myocardial tissue and also in brain, kidney, and adrenal glomerulosa cells, which secrete aldosterone

Losartan

Physicochemical
Structure

131

Presentation MOA

Use

Dose CVS

25, 50mg tab Pharmacodynamics Competitive antagonist for AII. 10000 times more selective blocker of ATI receptors than ATII receptors. Partially carboxylated in liver to an active metabolite which is 10-30 times more potent non-competitive AT1 antagonist. (see diagram above). Blocks all overt actions of AII vizvasoconstriction, central and peripheral sympathetic stimulation, release of aldosterone from adrenals, renal actions promoting salt and water retention, central actions like thirst, vasopressin release and growth promoting actions on heart and blood vessels. Systemic HT: Max effect in 3-4weeks. advantages no postural hypotension, no CNS effects, safe in asthmatics, prevention of secondary hyperaldosteronism, renal blood flow mentained, reverse left ventricular hypertrophy, no effect on lipid profile, no rebound HT, reduce cardiovascular morbidity and increase life expectancy. Portal HT CHF: reduce afterload and preload. CO. Accumulated salt and water is lost because of increased renal perfusion and abolition of mineralocorticoid mediated Na+ retension. Cardiac work (HR*SBP) and exercise capacitance MI Prophylaxis in high risk cardiovascular risk subjects Diabetic nephropathy prevents or delays ESRD Hypertensive nephropathy Scleroderma crisis-BP and renal function is caused by AII. ACE inhibitors Dramatic improvement and life saving 50mg OD, in liver disease 25mg OD
relax smooth muscle and thereby promote vasodilationarterioles dilate and compliance of large arteries TPR increase renal salt and water excretion, reduce plasma volume Both systolic and diastolic BP falls. Magnitude of drop in BP depends on Na+ and renin status. decrease cellular hypertrophy.

CNS

CV reflexes intact and no effect on capacitance vessels. Reflex increase in aldosterone is abolished No penetration

132

Respiratory Other

Renal: renal blood flow is mentained. Anti-proliferative effects. in the treatment of patients with diabetes, slowing the development and progression of diabetic glomerulopathy. slowing the progression of other forms of chronic renal disease, such as glomerulosclerosis losartan is a competitive antagonist of the thromboxane A2 receptor and attenuates platelet aggregatio. EXP3179, an active metabolite of losartan, reduces COX-2 mRNA up-regulation and COX-dependent prostaglandin generation

Side effects/ adverse effects

Well tolerated
Hypotension: especially in those with high pre-treatment renin/angiotensin II eg vol depleted, CHF, cirrhosis, renovascular HT Hyperkalemia - when ACE inhibitors are used with other drugs that can cause K+ retention reduced renal function, including that associated with bilateral renal artery stenosis and stenosis in the artery of a solitary kidney Cough, an adverse effect of ACE inhibitors, is less frequent with angiotensin II receptor antagonists. Angioedema occurs very rarely. Teratogenicity Occasional Headache, dizziness, nausea and GI upset

Interaction s

Hyperkalemia with K supplements/K sparing diuretics

Pharmacokinetics Absorption Oral bioavailability 33% due to high first pass metabolism. Not affected by food. Distribution 98% plasma protein bound Metabolism Partially carboxylated in liver to an active metabolite (EXP3179) which is 10-30 times more potent non-competitive AT1 antagonist. The metabolism of losartan to EXP 3174 and to inactive metabolites is mediated by CYP2C9 and CYP3A4. Peak plasma levels occur in 1 hr. T1/2 of losartan 2hr, T1/2 of metabolite 6-9hrs. No dose adjustment required in renal insufficiency but required in hepatic dysfunction Excretion Excreted in urine

133

Outline the pharmacology of hydrallazine and the potassium channel activators (nicorandil and minoxidil)

Hydrallazine Physicochemical Structure

Presentatio n MOA

25-50 mg tab Parenteral: 20 mg/mL for injection Pharmacodynamics Direct relaxation of arteriolar smooth muscle - mechanisms unclear - may involve a fall in intracellular calcium conc. Not a first-line drug in the treatment of hypertension. used in combination in the treatment of severe hypertension, CHF and in the treatment of hypertensive emergencies in pregnant women (especially preeclampsia) Usual dosage ranges from 40 mg/d to 200 mg/d. Dosing two or three times daily provides smooth control of blood pressure. Dilates arterioles but not veins/coronaries - powerful stimulation of the sympathetic nervous system, II to baroreceptor-mediated reflexes HR and contractility, plasma renin activity, and fluid retention counteract the antihypertensive effect of hydralazine. tachyphylaxis to its antihypertensive effects developed rapidly.

Use

Dose

CVS

CNS Respiratory Other Side effects/ adverse effects Most common adverse effects: headache, nausea, anorexia, palpitations, sweating, and flushing The higher dosage (>400mg/day) lupus erythematosuslike syndrome - arthralgia, myalgia, pleuritis, pericarditis, skin rashes, and fever Uncommon - pyridoxine-responsive polyneuropathy and drug fever Coronary steal

134

Interaction s Absorption Distribution Metabolism Pharmacokinetics Well absorbed but high first pass oral bioavailability ~25% Metabolized in part by acetylation in bowel/liver. rapid acetylators have greater first-pass metabolism, lower bioavailability, and less antihypertensive benefit. T1/2~ 1.5 to 3 hours, but vascular effects persist longer. peak hypotensive effect of the drug occur within 30 to 120 minutes of ingestion. Biliary excretion

Excretion

Nicorandil Structure Presentatio n MOA Physicochemical nicotinamide ester ester 5, 10mg tab, 2mg/vial inj Pharmacodynamics Activates ATP sensitive K channels- hyperpolarizing vascular smooth muscle vasodilatation Also donates NO which increase cGMP and cause vasodilatation both arterial and venous Angina HT MI CHF 5-20mg BD vasodilating properties in normal coronary arteries but more complex effects in patients with angina. Reduces both preload and afterload. No significant effects on contractility or conduction Provides some myocardial protection via preconditioning by activation of cardiac KATP channels. Mitochondrial K+ channel opening exerts myocardial protection by process of ischaemic preconditioning which reduces myocardial stunning, arrhythmias and infarct size when coronary artery is suddenly blocked

Use

Dose CVS

CNS Respiratory

135

Other Side effects/ adverse effects Interaction s Pharmacokinetics Absorption Absolute bioavailability is 75 23%. peak plasma levels occur within 0.30 to 1.0 hours after dosing Distribution weakly bound to human plasma proteins (free fraction greater than 75%) Metabolism Main biotransformation pathways are de-nitration and then introduction into the nicotinamide metabolism. Excretion Flushing Palpitation Weakness Headache Dizziness Mouth ulcers, nausea and vomiting

Minoxidil

Physicochemical
Structure

Presentation

Oral: 2.5, 10 mg tablets Topical: 2% lotion Pharmacodynamics Metabolized by hepatic sulfotransferase to the active molecule, minoxidil sulfate. Minoxidil sulfate opens the ATP-modulated K+ channel K+ efflux hyperpolarization relaxation of smooth muscle Reserved for poorly responding severe hypertension, especially in male patients with renal insufficiency. Used concurrently with a diuretic to avoid fluid retention and with a sympatholytic drug (usually a receptor antagonist) to control reflex cardiovascular effects. Alopecia The initial daily dose of minoxidil may be as little as 1.25 mg, which

MOA

Use

Dose

136

can be increased gradually to 40 mg in one or two daily doses.


CVS

Produces arteriolar vasodilation. no effect on the capacitance vessels. Increases blood flow to skin, skeletal muscle, the gastrointestinal tract, and the heart more than to the CNS. Strong reflex increase in heart rate, myocardial contractility and in cardiac output.

CNS Respiratory Other

renal artery vasodilator, but systemic hypotension can RBF. Renal function usually improves in patients who take minoxidil for the treatment of hypertension. Very potent stimulator of renin secretion.
Tachycardia, palpitations, angina, and edema Headache, sweating, and hirsutism

Side effects/ adverse effects Interaction s Absorption

fluid and salt retention

Pharmacokinetics Well absorbed. peak conc. occur 1 hour after oral administration, the maximal hypotensive effect of the drug occurs later

Distribution Metabolism Metabolized by hepatic sulfotransferase to the active molecule, minoxidil sulfate. Minoxidil has a half-life in plasma of 3 to 4 hours, but its duration of action is 24 hours or occasionally even longer. Excretion 20% of the absorbed drug is excreted unchanged in the urine, and the main route of elimination is by hepatic metabolism
physiological and pharmacological basis of antiarrhythmic therapy

137

A schematic representation of Na+ channels cycling through different conformational states during the cardiac action potential. Transitions between resting, activated, and inactivated states are dependent on membrane potential and time. The activation gate is shown as m and the inactivation gate as h. Potentials typical for each state are shown under each channel schematic as a function of time. The dashed line indicates that part of the action potential during which most Na+ channels are completely or partially inactivated and unavailable for reactivation

138

Inactivation gate (h) have voltage dependent function. They begin to close between -70 to - 55 mv and begin to recover from -55 to -70 mv Refractory period The time between phase 0 and sufficient recovery of sodium channels in phase 3 to permit a new propagated response to an external stimulus is the refractory period. Less negative resting membrane potential results in prolongation of refractory time. Since inactivation gates of sodium channels close between -70 to - 55 mv, hence, fewer sodium channels are available for diffusion of sodium ions when an AP is evoked from a resting potential of -60mv than when it is evoked from resting potential of 80 mv

SA node and AV node have resting membrane potential in the range of 50 to -70 mv hence all na channels are inactivated. Such depolarized cells exhibit slow responses slow upstroke velocity and slow conduction which depends on calcium inward current.

139

Other relatively depolarized cells exhibiting slow depolarization & conduction include cells exposed to hyperkalemia, sodium pump blockade, or ischemic cells.

Mechanisms of arrhythmias

Abnormal automaticity Non-pacemaker cells begin to spontaneously and abnormally initiate an impulse, believed to be the result of reduced (more positive) RMP bringing it closer to the threshold potential. Eg. Ischemia and electrolyte imbalances Acceleration of pacemaker discharge, brought about by increased phase 4 depolarization slope. Eg. hypokalemia, stimulation, positive chronotropic drugs, fibre stretch, acidosis and partial depolarization by currents of injury. After depolarization (or triggered activity)

140 Spontaneous depolarizations requiring a preceding impulse (a triggering beat) Early after depolarizations (EAD): After depolarizations originating during phase 2 or 3 of the AP Delayed afterdepolarization (DAD): After depolarizations originating during phase 4 of AP Early after depolarization

Prolonged action potential eg. Prolongation of QT interval (repolarization) by inhibition of delayed rectifier potassium current (sotalol, quinidine, dofetilide and procainamide) Torsade de pointe (TdP), a potentially lethal polymorphic ventricular arrhythmia, is an example of EAD, precipitated by K+channel blockers

Delayed after depolarization

Ventricular arrhythmias secondary to digoxin toxicity is an example of delayed afterdepolarization. Digoxin mediated increased intracellular Ca++ is believed to be the mechanism of this type of arrhythmia

Disorders of impulse conduction Most common mechanism of arrhythmias

141 Can result in conduction block and reentry

Re-entry Impulse recirculates in the heart and cause repititive activation Pre-requisites: Propagating impulse encounters electrophysiologically inhomogeneous tissue with unidirectional block allowing retrograde conduction retrograde conducting impulse encounters excitable tissue

Lengthening of the refractory period and / or slowing of the velocity of conduction may help terminate reentry. Examples of reentrant arrhythmias: AV nodal reentrant tachycardia (AVNRT) Atrioventricular reentrant tachycardia (AVRT) Atrial flutter Atrial fibrillation Ventricular tachycardia.

Mechanism of action of anti-arrhythmics Aim of therapy: Reduce ectopic pacemaker activity Modify conduction or refractoriness in reentry circuits to disable circus movement Major mechanisms to accomplish these goals: 1. Sodium channel blockade 2. Blockade of sympathetic autonomic effects in heart 3. Prolongation of effective refractory period 4. Calcium channel blockade Abnormal automaticity State dependent action of Na+ blockers: binding preference to the activated & inactivated channels than the resting channels Hence arrhythmic cells with rapid activity and depolarization of resting potential will have more channels in A and I stage and hence will be favorably blocked by the drug. In cells with abnormal automaticity most of the drugs reduce phase 4 slope by blocking either sodium and calcium channel thereby reducing the ratio of sodium to potassium permeability. Reentry arrhythmias Slow conduction by: Steady state reduction in the number of the unblocked channels which reduces the excitatory current to a level below that required for propagation

142 Prolongation of refractory time. (conversion to bi-direction block) Excessive slowing promotes reentry by allowing time for unidirectional block to recover. Hence powerful Na+ channel blockers e.g. Flecainide, Propafenone may actually promote ventricular tachyarrhythmias

Classify antiarrhythmic agents by their electro-physiological activity and mechanisms of action


The widest employed Vaughan-Williams classification. Major drawbacks: Over simplification of the effect The classification relies on the effect these agents have on normal tissue Major effects of an agent from one group overlaps with the effects of agents from other groups A more comprehensive classification of anti arrhythmic agents is available and has been called the Sicilian Gambit. Vaughan-Williams classification (1969) involves 4 classes. In 1979 harrison divided class I into subclass A B & C.

143

describe the pharmacology, with particular reference to the antiarrhythmic properties, of the sodium channel blocking agents (eg. lignocaine and flecainide)
Subclass IA Intermediate kinetics of binding & dissociation. Combined Na+ & K+ blockade

144

Supress AV conduction, prolong PR, QRS, QT and APD

145

Subclass IB
Rapid kinetics of binding and dissociation

Do not delay channel recovery or depress AV conduction or prolong APD, ERP & QT

Subclass IC

146

Slow kinetics of binding and dissociation

Most potent Na+ channel blockers with more prominent action on open state and longest recovery times. Markedly delays conduction, prolong PR, broaden QRS but have variable effect on APD.

Lignocaine
Physicochemical

147

Structure

Presentation MOA

20mg/ml inj Pharmacodynamics Blocks preferentially inactivated Na+ channels. Hence selective for partially depolarized cells & those with long AP (eg. PF & ventricular). Automaticity and after depolarizations in these cells supressed PF and Ventricular muscle: decreases APD duration (no class III action), and ERP to a lesser extent No action on APD or ERP of Atrial fibres. SA and AV node not affected Only in ventricular tachyarrythmias especially in digitalis toxicity setting where it does not effect AV node Used to be given prophylactially by infusion in acute MI reduces incidence of VF Current status: not used as prophylactic as failed to show survival and also increase short term mortality, possibly by increasing the incidence of asystole loading dose of 150200 mg over 15 minutes followed by a maintenance infusion of 24 mg/min. Therapeutic plasma conc. 2-5 microgram/ml. steady-state concentrations may be achieved in 810 hours in normal patients . In liver disease, the maintenance dose should be decreased, but usual loading doses can be given. In patients with heart failure, lidocaine's volume of distribution and total body clearance may both be decreased. Thus, both loading and maintenance doses should be decreased. No change in renal disease


Use

Dose

CVS

CNS Respiratory Other Side effects/ adverse

High dose can cause seizures

Cardiac

least cardiotoxic of the currently used sodium channel

148

effects

blockers Only in toxic doses:

Proarrhythmic effects (uncommon) - sinoatrial node arrest, worsening of impaired conduction, and ventricular arrhythmias May cause hypotensionpartly by depressing myocardial contractility.

Extra-cardiac Interaction s Seizures Nystagmus Paresthesia Blurred vision Disorientation Drowsiness Nausea Hypotension at high doses Drugs that decrease liver blood flow (eg, propranolol, cimetidine) reduce lidocaine clearance and so increase the risk of toxicity unless infusion rates are decreased. Pharmacokinetics High hepatic 1st pass metabolism: 3% oral bioavailability

Absorption Distribution Metabolism

Duration of action 10 to 20 min because of rapid redistribution Hydrolyzed, deethylated and conjugated. Metabolites excreted in urine T1/2 of early distribution phase is 8 min while late elimination is 2 hours

Excretion

Flecainide
Physicochemical

149

Structure

Presentation MOA

Oral: 50, 100, 150 mg tablets

Use

Pharmacodynamics Flecainide blocks Na+ current and delayed rectifier K+ current (IKr). very long recovery from Na+ channel block also blocks Ca2+ currents APD in Purkinje cells in ventricular cells, probably owing to block of delayed rectifier current Flecainide does not cause EADs in vitro or torsades de pointes. In atrial tissue, flecainide disproportionately prolongs action potentials at fast rates Flecainide prolongs the duration of PR, QRS, and QT intervals even at normal heart rates. maintenance of sinus rhythm in patients with supraventricular arrhythmias, including atrial fibrillation, in whom structural heart disease is absent, WPW
100200 mg twice a day

Dose CVS

CNS Respiratory Other Side effects/ adverse effects


Dose-related blurred vision can exacerbate CHF in pts with LV dysfunction Pro-arrythmogenic: acceleration of ventricular rate in patients with atrial flutter, increased frequency of episodes of re-entrant ventricular tachycardia, and increased mortality in patients convalescing from MI

Interaction s Pharmacokinetics Absorption Well absorbed. Distribution Metabolism T1/2~20hrs. Elimination occurs by both renal excretion of unchanged drug and hepatic metabolism by CYP2D6 to inactive metabolites. Excretion Both biliary and urinary excretion

150

The beta blockers

These drugs diminish phase 4 depolarization thus depressing automaticity, prolonging AV conduction, and decreasing HR and contractility Useful in treating tachyarrhythmias caused by increased sympathetic activity. They are also used for atrial flutter and fibrillation and for AV nodal reentrant tachycardia

151

All betablockers covered above

Class III agents


Class III: Potassium channel blockers Action is manifest by prolongation of the APD Most drugs with this action block the rapid component of the delayed rectifier potassium current, IKr. Undesirable property of "reverse use-dependence": action potential prolongation is least marked at fast rates (where it is desirable) and most marked at slow rates, where it can contribute to the risk of torsade de pointes.

152

153

Amiodarone Flecainide
Physicochemical
Structure

Presentation

400mg, 200mg tab 150mg/3ml Inj

MOA

Pharmacodynamics Markedly prolongs the APD (and the QT interval on the ECG) by blockade of IKr. IKs is blocked during chronic administration. Does not have reverse use-dependent action. Also significantly blocks inactivated sodium channels & decreases conduction velocity. Weak adrenergic and calcium channel blocking actions. Effects: slowing of the HR & AV node conduction. The broad spectrum of actions may account for its relatively high efficacy and low incidence of torsade de pointes despite significant QT interval prolongation. Extracardiac Effects: peripheral vasodilation, following intravenous administration. May be related to the action of the vehicle. (Polysorbate 80 and benzyl alcohol) Low doses (100200 mg/d) of amiodarone are effective in maintaining normal sinus rhythm in patients with atrial fibrillation. Effective in the prevention of recurrent ventricular tachycardia. Its use is not associated with an increase in mortality in patients with coronary artery disease or heart failure. In many centers, the implanted cardioverterdefibrillator (ICD) has succeeded drug therapy as the primary treatment modality for ventricular tachycardia, but amiodarone may be used for ventricular tachycardia as adjuvant therapy to decrease the frequency of uncomfortable ICD discharges. The drug increases the pacing and defibrillation threshold and these devices require retesting after a maintenance dose has been achieved.

Use

154

Dose

A total loading dose of 10 g is usually achieved with 0.81.2 g daily doses. The maintenance dose is 200 400 mg daily. Intravenous: 300 mg intravenously, over 1060 min depending on the circumstances and haemodynamic stability of the patient, followed by an infusion of 900 mg over 24 h. Additional infusions of 150 mg can be repeated. Maximum total daily dose of 2 g. Can cause thrombophlebitis if given by peripheral vien. Can be given by large bore peripheral line in emergency. Shock resistant VF: 300 mg followed by 150 mg if required. Therapeutic plasma range - 0.5 to 2.5 g/mL. Measured levels do not correlate well with efficacy or adverse effects Pharmacologic effects achieved rapidly by intravenous loading. With this route QT-prolonging effect is modest whereas bradycardia and atrioventricular block may be significant.

CVS

CNS Respiratory Other Side effects/ adverse effects

Cardiac: may produce symptomatic bradycardia & heart block in patients with preexisting sinus or AV node disease. Extracardiac: Accumulates in many tissues, including the heart (1050 times greater than plasma), lung, liver, and skin, and is concentrated in tears. The most common are tremor and ataxia (3%-35%, depending on dose and duration of therapy). Peripheral neuropathy is uncommon (0.3% annually) but may be severe, requiring dose reduction or discontinuation of therapy. Insomnia, memory disturbances, and delirium also have been reported. Asymptomatic corneal microdeposits (>90%), drug discontinuation is usually not required. Optic neuropathy/ neuritis (<1%-2%)

Neuropsychiatric

Eye

155

Thyroid Lungs Dose-related pulmonary toxicity is the most important adverse effect. Even on a low dose of 200 mg/d, fatal pulmonary fibrosis may be observed in 1% of patients. Risk factors for pulmonary fibrosis include: Underlying lung disease Dosages greater than 400 mg/d recent pulmonary insults Skin

Contains 37.3% iodine by weight Inhibits peripheral conversion of T4 to T3 Hypothyroidism (Wolff-Chaikoff effect)(6%) Hyperthyroidism ( Jodbasedow effect)(0.9%-2%)

Skin deposits result in a photodermatitis and a grayblue skin discoloration (4%-9%), in sun-exposed areas, eg, the malar regions.

Interaction s

Drugs that increase amiodarone levels Drugs that decrease amiodarone levels Amiodarone increases the following drug levels

Cimetidine Protease inhibitors Cholestyramine Rifampin Cyclosporine Digoxin (reduce digoxin dose by 50% and monitor levels) Theophylline Flecainide Lidocaine Procainamide Quinidine

others

Azole antifungals (prolong QTc) B-blockers (sinus bradycardia or AV block) Calcium channel blockers (sinus bradycardia or AV block) Fentanyl (bradycardia, sinus arrest, and hypotension) Fluoroquinolones (prolong QTcuse of sparfloxacin contraindicated) Statins (lovastatin, simvastatin) (increase the

156

risk of rhabdomyolysis or myopathy) Macrolide antibiotics (prolong QTc) Warfarin (potentiates effectreduce warfarin dose by 25%-33% & monitor INR) Absorption Distribution Metabolism Excretion Pharmacokinetics Variable absorbtion - bioavailability of 3565%. Highly lipophilic, with a large volume of distribution (66 L/kg) Hepatic metabolism (CYP3A4)- major active metabolite: desethylamiodarone. Elimination t1/2 is complex: rapid component of 310 days (50% of the drug) and a slower component of several weeks. Following discontinuation, effects are maintained for 1 3 months & measurable tissue levels up to 1 year.

Metabolites excreted in feces

Sotalol discussed above

Ibutilide
Physicochemical
Structure

Presentation MOA

0.1 g/mL solution for IV infusion

Use

Dose

Pharmacodynamics IKr blocker + activates an inward Na+ current. The action potential-prolonging effect of the drug may arise from either mechanism. The drug's efficacy rate is higher in patients with atrial flutter (50% to 70%) than in those with atrial fibrillation (30% to 50%). In atrial fibrillation, the conversion rate is lower in those in whom the arrhythmia has been present for weeks or months compared with those in whom it has been present for days. Ibutilide is administered as a rapid infusion (1 mg over 10 minutes) for the immediate conversion of atrial fibrillation or flutter to sinus rhythm.

157

CVS

CNS Respiratory Other Side effects/ adverse effects Interaction s Absorption Distribution Metabolism Excretion Pharmacokinetics The drug undergoes extensive first-pass metabolism and so is not used orally. It is eliminated by hepatic metabolism and has a half-life of 2 to 12 hours (average of 6 hours).
metabolites are excreted by the kidney

The major toxicity with ibutilide is excessive QT prolongation and torsades de pointes, which occurs in up to 6% of patients and requires immediate cardioversion in up to one-third of these.

Calcium channel antagonists

158

Electrophysiological actions of CCB

159

Verapamil: pharmacodynamics Cardiac Effects Blocks both activated and inactivated L-type calcium channels. Decreased slope of phase 4 depolarization & phase 0 in cal. Dependent slow fibres (AV, SA, depolarized tissues). AV nodal conduction time and effective refractory period are invariably prolonged Usually slows the SA node by its direct action, but its hypotensive action may occasionally result in a small reflex increase of sinoatrial nodal rate. Suppress both early & delayed afterdepolarizations. May antagonize slow responses arising in severely depolarized tissue. Extracardiac Effects: peripheral vasodilation, and smooth muscle relaxation Pharmacokinetics & Dosage T1/2 = 7 hours. Metabolized by the liver. Oral bioavailability =20%. Caution in patients with hepatic dysfunction. Verapamil dosage is an initial bolus of 5 mg administered over 25 minutes, followed a few minutes later by a second 5 mg bolus if needed. Thereafter, doses of 510 mg can be administered every 46 hours, or a constant infusion of 0.4 mcg/kg/min may be used. Effective oral dosages range from 120 to 640 mg daily, divided into three or four doses

Adverse effects Cardiac dose-related and usually avoidable. A common error: intravenous verapamil to a patient with ventricular tachycardia misdiagnosed as supraventricular tachycardia. In this setting, hypotension and ventricular fibrillation can occur. Should be avoided in wide complex tachy. Negative inotropic effects- may limit its clinical usefulness in diseased hearts

160 AV block in large doses or in AV nodal disease. In sinus node disease, verapamil can precipitate sinus arrest. Extracardiac Adverse effects include constipation, lassitude, nervousness, and peripheral edema.

Uses SVT is the major arrhythmia indication for verapamil. Verapamil can also reduce the ventricular rate in atrial fibrillation and flutter. MAT Diltiazem Hepatic metabolism T1/2 = 3.5 hrs Oral bioavailabiltiy 40 % (extensive 1st pass) Dose: loading 0.25mg/kg, followed by 0.35mg/kg. IV maintenance: 5-15 mg/hr. Oral maintenance: 60 -120 mg 6-8 hourly

Digoxin
Physicochemical
Structure

Molecule consist of aglycone to which are attached one or more sugars. Pharmacological properties reside in aglycone and attached sugars modify solubility and and cell permeability.

Presentation

0.25mg tab 0.5mg/2ml Inj

Pharmacodynamics
MOA
positive inotropic effects results from increased intracellular Ca2+ . Binding of cardiac glycosides to the sarcolemmal Na+,K+ATPase and inhibition of cellular Na+ pump activity results in a reduction in the rate of active Na+ extrusion and a rise in cytosolic Na+. This increase in intracellular Na+ reduces the transmembrane Na+ gradient that drives the extrusion of intracellular Ca2+ during

161 myocyte repolarization. With reduced Ca2+ efflux and repeated entry of Ca2+ with each action potential, Ca2+ accumulates in the myocyte: Ca2+ uptake into the SR is increased; this increased Ca2+ becomes available for release from the SR onto troponin C and other Ca2+-sensitive proteins of the contractile apparatus during the next cycle of excitation-contraction coupling, thereby augmenting myocyte contractility.

Cardiac glycosides increase phase 4 slope (i.e., increase the rate of automaticity), especially if [K]o is low. Prominent vagotonic actions, resulting in inhibition of Ca2+ currents in the AV node and activation of acetylcholine-mediated K+ currents in the atrium. Thus the major "indirect" electrophysiological effects of cardiac glycosides are hyperpolarization, shortening of atrial action potentials, and increases in AV nodal refractoriness. In heart transplant patients--ineffective for rate control.

Use

CHF Rate control in AF and AFL and PSVT

Dose

The antiarrhythmic effects of digoxin can be achieved with intravenous or oral therapy. However, digoxin undergoes relatively slow distribution to effector site(s); therefore, even with intravenous therapy, there is a lag of several hours between drug administration and the development of measurable antiarrhythmic effects

To avoid intoxication, a loading dose of approximately 1 to 1.5 mg digoxin is administered over 24 hours. Measurement of postdistribution serum digoxin concentration and adjustment of the

162

daily dose (0.125 to 0.375 mg) to maintain concentrations of 0.5 to 2 ng/ml are useful during chronic digoxin therapy.
CVS

Force of contraction: dose dependant increase in force of contraction, increased velocity of tension development, increased CO. systole is shortened and diastole is prolonged. Tone: defined as maximum length of fibre at a given filling pressure. Tone is not changed, end diastolic volume is reduced due to increased ejection fraction Rate: decreased by digitalis. In CHF sympathetic overactivity is also abolished due to improved CO. In addition digitalis slows HR by vagal and extra vagal effects Vagal tone is increased: Refelexly through nodose ganglion and sensitization of baroreceptors: amplify parasympathetic discharge to the heart Direct stimulation of vagal centre Sensitization of SA node to Ach. Extravagal: direct action on SA and AV nodes, probably an antiadrenergic action exerted beyond receptor level. Electrophysiological action: Action potential: RMP is shifted towards isoelectric level increased excitability Rate of phase 0 depolarization reduced Phase 4 slope is increased in PF- ectopic automaticity increased At high doses coupled beats occur due to oscillations in phase 4 resulting in DAD APD and amplitude is reduced

163

ERP

ECG: Decreased amplitude or inversion of t wave Increased PR interval Shortening of QT interval Depression of ST segment Blood vessels: mild vasoconstrictor CNS

Atrium decreased by vagal action, increased by direct action AV node and bundle of his increased by direct, vagomimetic and antiadrenergic actions. Conduction hence reduced. Ventricle abbreviated by direct action

Higher doses cause CTZ activation nausea vomiting. Very high doses produce hyperapnoea, central sympathetic stimulation, confusion and visual disturbances Kidney increased dieresis due to improved renal perfusion
Increased sympathetic activity, electrolyte abnormalities (e.g., hypokalemia, hypomagnesemia, and hypercalcemia) and hypoxia increase the risk of digitalis toxicity. Digitalis exerts direct arterial vasoconstrictor effects, which can be especially deleterious in patients with advanced atherosclerosis who receive intravenous drug; mesenteric and coronary ischemia have been reported. low therapeutic index 1.5 - 3 Toxicity: Extracardiac manifestations: fatique, nausea, vomiting, abdominal pain, disturbances of cognitive function, and blurred or yellow vision Cardiac: characteristic digitalis arrhythmias: DAD-related tachycardias along with impairment of sinus node or AV nodal function. Atrial tachycardia with AV block is classic,

Respiratory Other Side effects/ adverse effects

164 but ventricular bigeminy (sinus beats alternating with beats of ventricular origin), "bidirectional" ventricular tachycardia (a very rare entity), AV junctional tachycardias, and various degrees of AV block also can occur. With severe intoxication (e.g., with suicidal ingestion), severe hyperkalemia owing to poisoning of Na+,K+-ATPase and profound bradyarrhythmias, which may be unresponsive to pacing therapy, are seen. Treatment of toxicity: For tachyarrythmias: infuse KCL 20mmol/hr to max 100mmol. K antagonizes digitalis induced automaticity and decreases binding of digoxin to Na-K ATPase. K contraindicated in presence of AV blocks. Avoided in presence of very high plasma K. Ventricular arrhythmias: Lignocaine is drug of choice, repeated as required. Supraventricular arrhythmias: propanolol may be given. AV blocks and bradycardia: atropine may work temporarily. Temporary pacing may be required. Cardioversion should be avoided because severe conduction defects may be unmasked. Hemodylasis is not very effective owing to large molecule Digoxin antibody (DIGIBIND): the Fab fragment ultimate antidose

Sinus bradycardia and AV block often respond to intravenous atropine, but the effect is transient. Mg2+ has been used successfully in some cases of digitalis-induced tachycardia. Any serious arrhythmia should be treated with antidigoxin Fab fragments (DIGIBIND)

Caution

Hypokalemia enhances toxicity Elderly, renal dysfunction/ severe hepatic dysfunction more sensitive MI-arrythmogenic dose is reduced. Should be used after MI only when heart failure is accompanied by AF and rapid ventricular rate Thyrotoxicosis reduces responsiveness + increased

165 incidence of arrhythmias Myxoedema reduced digitalis clearance Acute myocarditis poor response + more arrythmogenic WPW digitalis contraindicated Amiodarone, quinidine, verapamil, diltiazem, cyclosporine, itraconazole, propafenone, and flecainide decrease digoxin clearance, likely by inhibiting P-glycoprotein, the major route of digoxin elimination. New steady-state digoxin concentrations are approached after 4 to 5 half-lives, i.e., in about a week. Hypokalemia, which can be caused by many drugs (e.g., diuretics, amphotericin B, and corticosteroids), will potentiate digitalis-induced arrhythmias. Succinylcholine causes arrhythmias in digitalized pts Metoclopramide, sucralfate neomycin, erythromycin reduce absorption

Interaction s

Pharmacokinetics Absorption
Oral bioavailability 60-80%

Distribution Plasma protein binding ~ 25%


Volume of distribution 4 to 7 liters/kg. The principal tissue reservoir is skeletal muscle

Metabolism

intestinal microflora may metabolize digoxin, markedly reducing bioavailability. Destruction of flora by antibiotics increases bioavailability. Toxicity can occur with drugs like tetracycline/erythromycin The elimination half-life of digoxin ordinarily is approximately 36 hours Onset of action 15-30min Peak 2-5hrs Duration 2-6 days

Excretion

Renal elimination of unchanged drug accounts for less than 80% of

166 digoxin elimination.

Digoxin doses should be reduced (or dosing interval increased) and serum concentrations monitored closely in patients with impaired excretion owing to renal failure or in patients who are hypothyroid

adenosine
mechanism of action Nucleoside composed of a molecule of adenine attached to a ribose sugar molecule (ribofuranose) Activates acetylcholine-sensitive K+ current in the atrium and sinus and AV nodes, resulting in shortening of action potential duration, hyperpolarization, and slowing of normal automaticity. Inhibits the electrophysiological effects of increased intracellular cyclic AMP that occur with sympathetic stimulation. Because adenosine thereby reduces Ca2+ currents, it can be antiarrhythmic by increasing AV nodal refractoriness and by inhibiting DADs elicited by sympathetic stimulation. Intravenous bolus transiently slows sinus rate and AV nodal conduction velocity and increases AV nodal refractoriness. Pharmacokinetics Half-life of seconds by carrier-mediated uptake, in most cell types, and subsequent metabolism by adenosine deaminase. Dose: 61212 mg, bolus by large bore IV access preferably central. potentiated by dipyridamole, an adenosine-uptake inhibitor, and in patients with cardiac transplants owing to denervation hypersensitivity. Methylxanthines block adenosine receptors; therefore, larger than usual doses are required Adverse Effects. Transient asystole (lack of any cardiac rhythm whatsoever) is common but usually lasts less than 5 seconds and is in fact the therapeutic goal. sense of chest fullness and dyspnea

167

Rarely can precipitate bronchospasm or atrial fibrillation presumably by heterogeneously shortening atrial action potentials. Magnesium Blocks voltage dependent L type Ca2+ channels Necessary co-factor for membrane enzyme Na+/K+ ATPase. By reducing the availability of intracellular Na+, reduces the ca inward current and triggered activity. Hence best classified as Na/K pump agonist Decreases RMP -> reduced automaticity. APD is increased thereby increasing absolute refractory period and reducing relative refractory period. Electrophysiological properties enhanced in the setting of increased extracellular K, hence, utility appears greatest in setting of ischemia (loss of intracellular K) Uses Acute rate control of AF Preventive post op SVT Acute control of MAT Ventricular arrythmias associated with triggered activity eg. Torsades & digoxin toxicity Polymorphic VT induced by class I agents Very effective at controlling transient ventricular arrythmia in setting of ischemia eg post infarct and cardiac surgery Dose AF rate and MAT control: 0.15 mmol/kg slow IV push. Subsequent dose recommendation has varied from 60 mmol to 0.1 mmol/kg/hr for 24 hrs SVT prophylaxis following cardiac surgery: 20-25 mmol / day for 4 days Transient ventricular arrythmia: 10 mmol slow iv push LIMIT-2 post MI dose: 8 mmol bolus over 5 min, followed by 65 mmol over 24 hrs Plasma levels for potent anti-arrythmic action are atleast 1.8mmol/l Adverse effects Rapid bolus hypotension by vasodilatation Skeletal muscle weakness

168

Excessive action in the setting of hyperkalemia bradycardia and heart block

Proarrythmic effects of anti-arrythmic drugs quinidine syncope due to VF and polymorphic VT at therapeutic concentration (also seen with disopyramide) CAST trial which involved flecainide, encainide and morizicine was terminated early because of adverse outcome in flecainide and encainide groups (RR of arrythmic death or non-fatal cardiac arrest of 3.6) Most pro-arrythmic events occur soon after starting the drug Proarrythmia appears to be correlated with the degree of drug induced QT prolongation or characteristics of sodium channel blockade Agents with short time constant of sodium channel blockade where sodium channel blockade is more pronounced at fast hr are less pro-arrythmic than drugs with long time constants (eg flecanide and propafenone) Class III drugs and quinindine pr-arrythmia correlate with the degree of QT prolongation Reentry is more likely to occur with shorter refractory period and reduced conduction velocity. Class Ib shorten RP. While class 1A and III prolong and hence would be beneficial in re-entry The scale of potency of pro-arrythmia has been found to be: flecainide > propafenone > quinidine > disopyramide > procainamide > mexiletine > lidocaine > sotalol Anti-arrythmic drugs are effective at suppressing abnormal automaticity with the exception of triggered automaticity due to EAD. Class IA and III can produce proarrythmia due to EAD Digoxin can be pro-arrythmic via production of triggered activity due to DAD
INOTROPES & VASOPRESSORS

169

Adrenaline
Physicochemical
Structure

Presentation

First, epinephrine is unstable in alkaline solution; when exposed to air or light, it turns pink from oxidation to adrenochrome and then brown from formation of polymers. Epinephrine injection is available in 1 mg/ml (1:1000), 0.1 mg/ml (1:10,000), and 0.5 mg/ml (1:2,000) solutions.

Pharmacodynamics
MOA Use

potent stimulant of 1, 1 and 2 receptors respiratory distress due to bronchospasm rapid relief of hypersensitivity reactions, including anaphylaxis Prolong the action of local anesthetics, presumably by decreasing local blood flow cardiac arrest

170

Dose

topical hemostatic agent on bleeding surfaces such as in the mouth or in bleeding peptic ulcers during endoscopy inhalation of epinephrine may be useful in the treatment of postintubation and infectious croup. Refractory shock especially where both inotropic & vasoconstrictive effects are desired Usual S/C dose 0.3 to 0.5 mg. Dose in cardiac arrest 1mg IV boluses Anaphylaxis, 1ml boluses of 1:10000 soln 1% (10 mg/ml; 1:100) formulation is available for inhalation. IV infusions 2-5g/min for shock.

CVS

Blood Pressure. IV blous of normal dose systolic > diastolic, pulse pressure. As the response wanes, the mean pressure may fall below normal before returning to control levels. mechanism: (1) positive inotropic action (2) positive chronotropic action and (3) vasoconstriction in many vascular beds especially in the precapillary resistance vessels of skin, mucosa, and kidneyalong with marked constriction of the veins. HR BPvagal discharge HR IV bolus of low dose (0.1 g/kg) BP. Reason greater sensitivity to 2 receptors than receptors Slow IV infusion or S/C adrenaline peripheral resistance owing to dominant action on 2 receptors of vessels in skeletal muscle diastolic BP & blood flow. systolic pressure due to HR, SV, and CO blood flow venous return, RAP. At higher doses of infusion actions predominate and TPR. Vascular Effects. Marked cutaneous blood flow. constricting precapillary vessels and small venules renal blood flow by as much as 40% Blood flow to skeletal muscles by therapeutic doses. > effect. At higher doses effect predominates.

effect

Arterial and venous pulmonary pressures due to direct pulmonary vasoconstriction + redistribution of blood from the

171 systemic to the pulmonary circulation pulmonary edema can occur Coronary blood flow under physiological conditions. Cause: diastolic time, blood pressure, myocardial O2 demand metabolic vasodilatation mediated by adenosine. Cerebral circulation does not constrict in response to arenaline. Autoregulation limits in CBF Cardiac Effects. predominant 1 stimulation. HR, SV, CO, work & O2 consumption. Cardiac systole is shorter. Cardiac efficiency (work done relative to oxygen consumption) Pro-arrythmic : Activates latent pacemakers, automaticity. slope of phase 4 depolarization, amplitude and rate of phase 0 depolarization seen in conducting fibres. Not in atrial and ventricular muscle fibres. refractory period of AV node by direct effects. However reflex vagal discharge may indirectly tend to prolong it.

CNS

Poor penetration. At therapeutic doses restlessness, apprehension, headache, and tremor due to effects on cardiovascular system Relaxes bronchial muscle (2 effect). Inhibition of antigen-induced release of inflammatory mediators from mast cells ( effect), diminution of bronchial secretions and congestion within the mucosa ( effect).

Respiratory

Other

Smooth Muscles. Gastrointestinal smooth muscle relaxed. Pyloric and ileocecal sphincters are contracted, but these effects depend on the preexisting tone of the muscle. If tone already is high, epinephrine causes relaxation; if low, contraction. During the last month of pregnancy and at parturition, epinephrine inhibits uterine tone and contractions ( 2 effect) Urinary bladder: relaxes detrusor muscle ( effect) and contracts

172 the trigone and sphincter muscles ( effect). Metabolic Effects. 20% to 30% in oxygen consumption Hyperglycemia lactate insulin secretion due to 2 receptors , insulin secretion due to receptors. Predominant effect is inhibition. Glucagon secretion ( receptors on cells of pancreatic islets). uptake of glucose by peripheral tissues glycogenolysis ( receptors) free fatty acid conc. ( receptors in adipocytes)

Renal: RBF. Since the glomerular filtration rate is only slightly and variably altered, the filtration fraction is consistently increased. Excretion of Na+, K+, and Cl- is decreased. The secretion of renin is increased as a consequence of a direct action of epinephrine on 1 receptors in the juxtaglomerular apparatus.

Miscellaneous number of circulating polymorphonuclear leukocytes accelerates blood coagulation and promotes fibrinolysis. Stimulates lacrimation and a scanty mucus secretion from salivary glands. Mydriasis lowers intraocular pressure increase physiological tremor, at least in part due to receptormediated enhancement of discharge of muscle spindles. Epinephrine promotes a fall in plasma K+, largely due to stimulation of K+ uptake into cells.

Side

restlessness, throbbing headache, tremor, and palpitations.

173

effects/ adverse effects

cerebral hemorrhage and cardiac arrhythmias. Angina may be induced by epinephrine in patients with coronary artery disease. Contraindicated in patients who are receiving nonselective receptor blocking drugs, since its unopposed actions on vascular 1 receptors may lead to severe hypertension and cerebral hemorrhage.

Interaction s Pharmacokinetics Absorption


Not effective orally. Rapidly conjugated and oxidized in the GI mucosa and liver. Absorption from subcutaneous tissues occurs relatively slowly, more rapid after intramuscular injection. In emergencies, it may be necessary to administer epinephrine intravenously.

Distribution Metabolism
Rapidly inactivated. The liver is rich in both of the enzymes responsible for destroying circulating epinephrine (COMT and MAO).

Excretion

only small amounts appear in the urine

Noradrenaline
Physicochemical
Structure

Presentation MOA

1 mg/mL for injection

Pharmacodynamics
Major chemical mediator liberated by mammalian postganglionic sympathetic nerves

174

Sympathomimetic Acts on both & 1- receptors but has relatively little effect on 2-receptors Like adrenaline: low dose predominantly beta effect; higher dose alpha Causes i) vasoconstriction [ -receptor] ii) Inotrope [ 1-receptors] iii) Chronotrope [ 1-receptors]
Clinical effects offset by increased afterload

Consequently increases systemic BP and coronary artery blood flow

Use Dose CVS

In the treatment of low blood pressure, the dose is titrated to the desired pressor response. 0.06-0.15g/kg/min Systolic, diastolic and pulse pressure CO - unchanged or decreased TPR vagal reflex activity HR renal blood flow Constricts mesenteric vessels and splanchnic and hepatic blood flow. Coronary flow

CNS Respiratory Other

hyperglycemia and other metabolic effects similar to those produced by epinephrine but these are observed only when large doses are given because norepinephrine is not as effective a "hormone" as epinephrine. severe hypertension necrosis and sloughing can occur at the site of intravenous injection owing to extravasation of the drug. The infusion should be made high in the limb, preferably through a long plastic cannula extending centrally. Impaired circulation at injection sites, with or without extravasation of norepinephrine, may be relieved by infiltrating the area with phentolamine, an receptor antagonist. Caution:

Side effects/ adverse effects

Should not be mixed in saline alone [water O.K.] Risk of arrhythmias with volatile anaesthetics Give with caution to patients on MAO or tricyclics as will prolong action

175

Interaction s

Should not be mixed in saline alone [water O.K.] Risk of arrhythmias with volatile anaesthetics Give with caution to patients on MAO or tricyclics as will prolong action

Absorption

Pharmacokinetics ineffective when given orally and is absorbed poorly from sites of subcutaneous injection

Distribution Metabolism Rapidly inactivated. The liver is rich in both of the enzymes
responsible for destroying circulating epinephrine (COMT and MAO).

Excretion

Small amounts normally are found in the urine Evidence

some survival benefit in septic shock, compared with high-dosage dopamine and epinephrine

epinephrine Heart rate Stroke volume Cardiac output Arrhythmias Coronary blood flow Systolic BP MAP Diastolic BP Mean PAP TPR Cerebral blood flow Muscle blood flow Skin blood flow + ++ +++ ++++ ++ +++ + +/0/++ -/+ ++ +++ --

norepinephrine ++ 0/++++ ++ +++ ++ ++ ++ ++ 0/0/--

176

Renal blood flow Splanchnic blood flow Oxygen demand Blood glucose Blood lactate

++ +++ +++

0/0/+ 0/+ 0/+

Vasopressin
Physicochemical
Structure Presentation MOA
nonapeptide with a 6-amino-acid ring and a 3-amino-acid side chain.

Inj 20 U/ml

Pharmacodynamics
Vasopressin is a peptide hormone released by the posterior pituitary in response to rising plasma tonicity or falling blood pressure. Vasopressin possesses antidiuretic and vasopressor properties. Acts via G protein coupled V1 and V2 receptors. It exerts its circulatory effects through V1 (V1a in

vascular smooth muscle, V1b in the pituitary gland) and V2 receptors (renal collecting duct system; Table). V1a stimulation mediates constriction of vascular smooth muscle, whereas V2 receptors mediate water reabsorption by enhancing renal collecting duct permeability.

Use

Dose CVS

Cardiac arrest Vasopressor sparing/catecholamine hyposensitivity in shock states Bleeding esophageal varices 0.1 to 0.05 units/min
40 U boluses in cardiac arrest

CNS

Vasopressin causes less direct coronary and cerebral vasoconstriction than catecholamines and has a neutral or inhibitory impact on CO, depending on its dose-dependent increase in SVR and the reflexive increase in vagal tone. A vasopressin-modulated increase in vascular sensitivity to norepinephrine further augments its pressor effects. The agent may also directly influence mechanisms involved in the pathogenesis of vasodilation, through inhibition of ATPactivated potassium channels, attenuation of nitric oxide production, and reversal of adrenergic receptor downregulation. The pressor effects of vasopressin are relatively preserved during hypoxic and acidotic conditions, which commonly develop during shock of any origin. does not penetrate BBB

177

Respiratory Other Side effects/ adverse effects Allergic reactions & anaphylaxis Cardiac arrest, circumoral pallor, arrhythmias, decreased cardiac output, angina, myocardial ischemia, peripheral vasoconstruction and gangrene. Gastrointestinal: abdominal cramps, nausea, vomiting. Nervous System: tremor, vertigo, palpitations Respiratory: bronchial constriction. Skin and Appendages: sweating, urticaris, cutaneous gangrene Interaction s Drugs which potentiate antidiuretic effect: carbamazepine; chlorpropamide; clofibrate; urea; fludrocortisone; tricyclic antidepressants. Drugs which decrease the antidiuretic effect of vasopressin: demeclocyline; norepinephrine; lithium; heparin, alcohol. Ganglionic blocking agents may produce a marked increase in sensitivity to the pressor effects of vasopressin. Pharmacokinetics Orally destroyed by trypsin

Absorption Distribution Metabolism T1/2 10 to 20 minutes. Metabolized by enzymatic cleavage in many organs especially liver and kidney Excretion

Dobutamine
Physicochemical
Structure

Presentation MOA

12.5mg/ml Injection

Pharmacodynamics

strong +ve inotropy due to beta1 agonist effects and alpha1 agonism - mild +ve chronotropy due (+) isomer effect on beta receptors - weaker alpha receptor blockade and beta2 stimulation, produced by (+) isomer and alpha1 agonism produced by

178

(-) isomer - overall peripheral effect should be an increase in blood flow to skeletal muscle (beta2 agonism) and some reduction in skin blood flow (alpha1 agonism balanced by some alpha blockade). These effects are weak compared to the myocardial effects - net effects are an increase in SV and CO. SVR may be unchanged or moderately decreased and arterial pressure may thus rise, fall slightly or remain unchanged - at doses > 15 mcg/kg/min tachycardia and arrhythmias are more likely - tolerance may be seen after 48-72 hrs, presumably due to down-regulation of beta receptors. May necessitate an increase in dose. Dose required to produce toxic effects seems to be increased equivalently
Use Dose CVS

CNS Respiratory Other Side effects/ adverse effects Interaction s Pharmacokinetics Absorption Distribution Metabolism

tachycardia and tachyarrhythmias less frequent than with dopamine - enhances AV conduction and may precipitate AF in predisposed patients

Excretion

Onset of action within 2 min and maximal effect associated with a given infusion rate occurs approximately 10 min after starting the infusion. metabolized by COMT to. t1/2 2.3 min inactive metabolites which are excreted in the urine

Dopamine
Physicochemical

179

Structure

Presentation MOA

200mg ampule

Use

Dose

CVS

Pharmacodynamics Acts on D1, D2, and 1 receptors D1 receptors in renal and mesenteric blood vessels are most sensitive-at low doses-increased blood flow->GFRNa excretion Higher doses stimulate 1 receptors inotropic actions Large doses have predominant actions vasoconstriction CHF, particularly in patients with oliguria and low or normal peripheral vascular resistance. Cardiogenic and septic shock. Dopamine hydrochloride is used only intravenously. The drug initially is administered at a rate of 2 to 5 g/kg per minute; this rate may be increased gradually up to 20 to 50 g/kg per minute or more as the clinical situation dictates At low doses (1 to 3 g/kg per min) Vascular D1 receptors in in the renal, mesenteric, and coronary beds activates adenylyl cyclase, cAMP vasodilation GFR, renal blood flow, and Na+ excretion. Total peripheral resistance usually is unchanged At higher conc. (3 to 10 g/kg per min) 1 receptors +ve inotropic actions, release of norepinephrine from nerve terminals SBP & pulse pressure, minimal effect on DBP. At ever higher conc. (10 to 20 g/kg per min) predominant activation of vascular 1 receptors generalized vasoconstriction Poor penetration no effect

CNS Respiratory Other Side effects/ adverse effects Interaction s Absorption

Nausea, vomiting, tachycardia, anginal pain, arrhythmias, headache, hypertension, and peripheral vasoconstriction Extravasation ischemic necrosis and sloughing. Rarely, gangrene of the fingers or toes

MAO inhibitor, TCA Pharmacokinetics ineffective when administered orally

180

Distribution Metabolism Excretion

Dopamine is a substrate for both MAO and COMT

evidence
Published data in sepsis suggest that dopamine may impair hepatosplanchnic No benefit of renal dose dopamine

Isoprenaline
Physicochemical
Structure

Presentation MOA Use Dose CVS

1000 microgram/5mL

Pharmacodynamics activating 1 and 2 receptors equally To treat shock, cardiac arrest, bradyarrythmias, bronchospasm 0.05 - 0.5 microgram/kg/minute positive chronotropic, dromotropic, and inotropic effect SBP. In skeletal muscle arterioles it produces vasodilatation DBP relaxation of bronchial smooth muscle Tachycardia continuously monitor heart rate. Cardiac dysrhythmia, hypertension, hypotension, vomiting, tremor.

CNS Respiratory Other Side effects/ adverse effects Interaction s Absorption Distribution Metabolism

Pharmacokinetics

T1/2 ~2hrs

181

Excretion

milrinone
Physicochemical
Structure

Class Presentation MOA

Phosphodiesterase inhibitor
Injection, solution: 1 mg/mL (10 mL, 20 mL, 50 mL)

Use Dose

Pharmacodynamics phosphodiesterase III inhibitors potentiates the effect of cyclic adenosine monophosphate (cAMP). Milrinone also enhances relaxation of the left ventricle by increasing Ca2+-ATPase activity on the cardiac sarcoplasmic reticulum. This increases calcium ion uptake. Cardiogenic shock Pulmonary hypertension eg with PE
Adults: I.V.: Loading dose (optional): 50 mcg/kg administered over 10 minutes followed by a maintenance dose titrated according to the hemodynamic and clinical response; Maintenance dose: I.V. infusion: 0.375-0.75 mcg/kg/minute. If hypotension is a problem, loading doses may be omitted and maintenance infusions initiated. There is some delay in hemodynamic effects, but it is minimal (1-3 hours).

Monitoring Parameters
Platelet count, CBC, electrolytes (especially potassium and magnesium), liver function and renal function tests; ECG, CVP, SBP, DBP, heart rate; infusion site

CVS

+ve inotropic and vasodilatation

CNS Respiratory Other Side effects/ adverse effects


>10%: Cardiovascular: Ventricular arrhythmia (ectopy 9%, NSVT 3%, sustained ventricular tachycardia 1%, ventricular fibrillation <1%)

182

1% to 10%: Cardiovascular: Supraventricular arrhythmia (4%), hypotension (3%), angina/chest pain (1%) Central nervous system: Headache (3%) <1% (Limited to important or life-threatening): Atrial fibrillation, hypokalemia, MI, thrombocytopenia, tremor, ventricular fibrillation Postmarketing and/or case reports: Anaphylaxis, bronchospasm, injection site reaction, liver function abnormalities, rash, torsade de pointes

Interaction s Absorption Distribution

no known significant interactions

Pharmacokinetics Good Oral absorption


Onset of action: I.V.: 5-15 minutes Distribution: Vdss: 0.32-0.45 L/kg Protein binding, plasma: ~70%

Metabolism

Metabolism: Hepatic (12%) Half-life elimination: Normal renal function: ~2.5 hours; CVVH: 20.1 hours (Taniguchi, 2000)

Excretion

Excretion: Urine (85% as unchanged drug) within 24 hours; active tubular secretion is a major elimination pathway for milrinone

levosimendan
Physicochemical

183

Structure

Class Presentation

Calcium sensitizer
2.5 mg per 5 mL per ampoule and 10mL per ampoule

Pharmacodynamics
MOA
does not increase intracellular concentrations of free calcium. It binds to cardiac troponin C in a calcium-dependent manner and stabilises troponin C. This causes actin-myosin cross-bridges, without increasing myocardial

consumption of adenosine triphosphate (ATP). Also opening of ATPdependent potassium (K_) channels causes vasodilatation

Use Dose CVS

indicated for inotropic support in acutely-decompensated severe congestive heart failure


6 to 12 g/kg loading dose over 10 minutes followed by

0.05 to 0.2 g/kg/min as a continuous infusion enhance cardiac contractility. No arrythmogenic potential. venous, arterial and coronary vasodilation, probably by opening ATPsensitive potassium channels in smooth muscle.

CNS Respiratory Other Side effects/ adverse effects Interaction s Absorption


Tachycardia, enhanced AV conduction Hypotension

No significant interactions Pharmacokinetics


well

absorbed orally 5% dose is converted in intestines to highly active metabolite with t1/2 75-80h. Effects after infusion hence may persist 7-9 days after stopping infusion. T1/2 of levosimendan otherwise is 1hr.

Distribution 98% bound to plasma proteins Metabolism Excretion

184

Evidence Some of the Phase-III studies in the extensive clinical program were the trials LIDO (200 patients), RUSSLAN (500), CASINO (250), REVIVE-I (100), REVIVE-II (600) and finally SURVIVE (1350),[1] a head-to-head trial between levosimendan and dobutamine in acute decompensated heart failure. levosimendan did not significantly reduce all-cause mortality at 180 days In the randomised Levosimendan Infusion versus Dobutamine (LIDO) trial in 203 patients with severe, low-output decompensated CHF,[10] significantly more patients in the levosimendan group achieved the primary efficacy end-point of an increase from baseline in cardiac index 30% and a decrease in PCWP 25% than in the dobutamine group (5 to 10 g/kg/min for 24 hours) [28 and 15%, respectively].

You might also like